You are on page 1of 142

G.R. No.

137705

August 22, 2000

SERG'S PRODUCTS, INC., and SERGIO T. GOQUIOLAY, petitioners,


vs. PCI LEASING AND FINANCE, INC., respondent.
FACTS:
PCI Leasing filed a complaint for a sum of money with an
application for a writ of replevin. Writ of Replevin was granted
directing its sheriff to seize and deliver the machineries and
equipment to PCI Leasing after 5 days and upon the payment of the
necessary expenses.
Serg's Products Inc. filed a motion for special protective order
but the motion was opposed by PCI Leasing, on the ground that the
properties were still personal and therefore still subject to seizure and
a writ of replevin.
Serg's Products Inc. asserted that the properties sought to be
seized were immovables. They argued that to give effect to the
agreement would be prejudicial to innocent third parties.
Appellate court held that the subject machines were personal
property, and that they had only been leased, not owned, by
petitioners.
ISSUE:
Whether or not the machineries purchased and imported by
SERGS became real property by virtue of immobilization.
RULING:
Court held that the machines were placed by petitioners in the
factory built on their own land. Indisputably, they were essential and
principal elements of their chocolate-making industry. Although each
of them was movable or personal property on its own, all of them
have become "immobilized by destination" because they are essential
and principal elements in the industry. Court agreed with petitioners
that said machines are real, not personal pursuant to Article 415 (5) of
the Civil Code.
However, the Court held that contracting parties may validly
stipulate that a real property be considered as personal. After

agreeing to such stipulation, they are consequently estopped from


claiming otherwise.
In this case, the Lease Agreement clearly provides that the
machines in question are to be considered as personal property.
Hence, the petitioners are estopped from denying the
characterization of the subject machines as personal property. Under
the circumstances, they are proper subjects of the Writ of Seizure.
In treating the machines as personal property pursuant to the
Lease Agreement is good only insofar as the contracting parties are
concerned. Third persons acting in good faith are not affected by its
stipulation characterizing the subject machinery as personal.

G.R. No. L-18456

November 30, 1963

CONRADO P. NAVARRO, plaintiff-appellee, vs.


RUFINO G. PINEDA, RAMONA REYES, ET AL., defendantsappellants.
FACTS:
To secure a loan, Pineda and his mother executed real estate
and chattel mortgages in favor of Navarro. The real estate mortgage
covered a parcel of land owned by the mother while the chattel
mortgage covered a residential house. The defendants failed to pay
when the mortgage debt became due and payable. They asked and
granted extensions to pay for the loan.
On the second extension, Pineda executed a PROMISE wherein
in case of default in payment, he wouldnt ask for any additional
extension and there would be no need for any formal demand. In
spite of this, they still failed to pay.
Navarro then filed a complaint for foreclosure of the mortgage
and for damages. The trial court decided in his favor.
ISSUE:
Whether or not the deed of real estate mortgage and chattel
mortgage appended to the complaint is valid notwithstanding the fact
that the house was made subject of chattel mortgage for the reason
that it is erected on a land that belongs to a third person.
RULING:
Where a house stands on a rented land belonging to
another person, it may be the subject matter of a chattel mortgage as
personal property if so stipulated in the document of mortgage, and
in an action by the mortgagee for the foreclosure, the validity of the
chattel mortgage cannot be assailed by one of the parties to the
contract of mortgage.
Furthermore, although in some instances, a house of mixed
materials has been considered as a chattel between the parties and
that the validity of the contract between them, has been
recognized, it has been a constant criterion that with respect to
third persons, who are not parties to the contract, and specially in
execution proceedings, the house is considered as immovable
property.
3

G.R. Nos. L-10837-38

May 30, 1958

ASSOCIATED INSURANCE and SURETY COMPANY, INC., plaintiff,


vs. ISABEL IYA, ADRIANO VALINO and LUCIA VALINO,
defendants.
FACTS:
Adriano Valino and Lucia A. Valino, husband and wife,
purchased a house on installment basis from the Philippine Realty
Corporation. To enable her to purchase on credit rice from NARIC,
Valino filed a bond (P11,000) subscribed by Associated Insurance and
Surety Co Inc, and as a counter-guaranty, Valino executed an alleged
chattel mortgage on the aforementioned house in favour of the surety
company.
To secure payment of an indebtedness in the amount of
P12,000.00, the Valinos executed a real estate mortgage over the lot
and the house in favor of Isabel Iya.
As Valino failed to satisfy her obligation to the NARIC, the
company foreclosed the chattel mortgage over the house. A public
sale was conducted wherein the property was awarded to the surety
company, and then caused the said house to be declared in its name
for tax purposes.
The surety company learned of the existence of the real estate
mortgage over the lot and the improvements thereon; thus, they
prayed for the exclusion of the residential house from the real estate
mortgage and the declaration of its ownership in virtue of the award
given during bidding.
Isabel Iya filed her answer alleging among other things, that in
virtue of the real estate mortgage executed by her co-defendants, she
acquired a real right over the lot and the house constructed thereon;
that the auction sale as a result of the foreclosure of the chattel
mortgage on the house was null and void for non-compliance with the
form required by law. She prayed for the dismissal of the complaint
and annulment of the sale.
Surety Company argued that as the lot on which the house was
constructed did not belong to the spouses at the time the chattel
mortgage was executed, the house might be considered as personal
property, and they prayed that the said building be excluded from the
real estate mortgage.
4

ISSUE:
As the building constructed thereon has been the subject of 2
mortgages; controversy arise as to which of these encumbrances
should receive preference over the other.
RULING:
A building certainly cannot be divested of its character of realty
by the fact that the land on which it is constructed belongs to another.
In the case at bar, as personal properties could only be the subject of a
chattel mortgage and as obviously the structure in question is not
one, the execution of the chattel mortgage covering said building is
clearly invalid and a nullity.
While it is true that said document was correspondingly
registered in Chattel Mortgage Registry of Rizal, this act produced no
effect whatsoever, for where the interest conveyed is in the nature of
real property, the registration of the document in the registry of
chattels is merely a futile act. Thus, the registration of the chattel
mortgage of a building of strong materials produced no effect as far as
the building is concerned. The building is subject to the real estate
mortgage, in favour of Iya. Iyas right to foreclose not only the land
but also the building erected thereon is recognized.

G.R. No. L-40411

August 7, 1935

DAVAO SAW MILL CO., INC., plaintiff-appellant, vs. APRONIANO


G. CASTILLO and DAVAO LIGHT & POWER CO., INC., defendantsappellees.
FACTS:
The Davao Sawmill Co. has operated a sawmill in the Province
of Davao. However, the land upon which the business was conducted
belonged to another person. On the land the sawmill company
erected a building which housed the machinery used by it. Some of
the implements thus used were clearly personal property, the conflict
concerning machines which were placed and mounted on foundations
of cement.
In the contract of lease, Davo Sawmill agreed to turn over free
of charge all improvements and buildings erected by it on the
premises with the exception of machineries, which shall remain with
the Davao Sawmill. In an action brought by the Davao Light and
Power Co., judgment was rendered against Davao Sawmill. A writ of
execution was issued and the machineries placed on the sawmill were
levied upon as personalty by the sheriff. Davao Light and Power Co.
proceeded to purchase the machinery and other properties auctioned
by the sheriff.
ISSUE:
Whether or not the machineries are personal in nature.
RULING:
YES. It is machinery which is involved; moreover, machinery
not intended by the owner of any building or land for use in
connection therewith, but intended by a lessee for use in a building
erected on the land by the latter to be returned to the lessee on the
expiration or abandonment of the lease.
It was held that machinery which is movable in its nature only
becomes immobilized when placed in a plant by the owner of the
property or plant, but not when so placed by a tenant, a usufructuary,
or any person having only a temporary right, unless such person
acted as the agent of the owner.
In this case, the machinery placed in the sawmill is a movable
property.
6

G.R. No. 168557

February 16, 2007

FELS ENERGY, INC., Petitioner, vs. THE PROVINCE OF


BATANGAS and THE OFFICE OF THE PROVINCIAL ASSESSOR OF
BATANGAS, Respondents.
FACTS:
NPC entered into a lease contract with Polar Energy, Inc. over
3x30 MW diesel engine power barges moored at Balayan Bay in
Calaca, Batangas. The contract was for a period of five years.
In the agreement, NPC was made to shoulder any tax expenses.
Subsequently, Polar Energy, Inc. assigned its rights under the
Agreement to FELS. FELS received an assessment of real property
taxes on the power barges. FELS referred the matter to NPC,
reminding it of its obligation under the Agreement to pay all real
estate taxes. It then gave NPC the full power and authority to
represent it in any conference regarding the real property assessment
of the Provincial Assessor. NPC sought reconsideration to Provincial
Assessor but was denied. LBAA affirmed provincial assessor while
CBAA found the power barges exempt from real property tax,
consequently reversed its own ruling. FELS & NPC separately filed a
petition for review before CA.
ISSUE:
Whether or not power barges, which are floating and movable,
are personal properties and therefore, not subject to real property tax.
RULING:
Power barges are real property and are thus subject to real
property tax. Tax assessments by tax examiners are presumed correct
and made in good faith, with the taxpayer having the burden of
proving otherwise.
Article 415 (9) of the New Civil Code provides that "docks and
structures which, though floating, are intended by their nature and
object to remain at a fixed place on a river, lake, or coast" are
considered immovable property. Thus, power barges are categorized
as immovable property by destination, being in the nature of
machinery and other implements intended by the owner for an
industry or work which may be carried on in a building or on a piece
of land and which tend directly to meet the needs of said industry or
work.

G.R. No. 179987

September 3, 2013

HEIRS OF MARIO MALABANAN, (Represented by Sally A.


Malabanan), Petitioners, vs. REPUBLIC OF THE PHILIPPINES,
Respondent.
FACTS:
The property subject of the application for registration is a
parcel of land situated in Barangay Tibig, Silang Cavite. Mario
Malabanan filed an application for land registration covering the
property in the RTC. Malabanan claimed that he had purchased the
property from Eduardo Velazco, and that he and his predecessors-ininterest had been in open, continuous, uninterrupted, public and
adverse possession and occupation of the land for more than 30
years.
Velazco testified that the property was originally belonged to a
twenty-two hectare property owned by his great-grandfather, Lino
Velazco. Upon Linos death, his four sons inherited the property and
divided it among themselves. But by 1966, Estebans wife, Magdalena,
had become the administrator of all the properties inherited by the
Velazco sons from their father, Lino. After the death of Esteban and
Magdalena, their son Virgilio succeeded them in administering the
properties, including Lot 9864-A, which originally belonged to his
uncle, Eduardo Velazco. It was this property that was sold by Eduardo
Velazco to Malabanan.
Among the evidence presented by Malabanan during trial was a
Certification dated 11 June 2001, issued by the CENRO-DENR, which
stated that the subject property was verified to be within the
Alienable or Disposable land per Land Classification Map No. 3013
established under Project No. 20-A and approved as such under FAO
4-1656 on March 15, 1982. On 3 December 2002, the RTC approved
the application for registration.
The Republic interposed an appeal to the Court of Appeals,
arguing that Malabanan had failed to prove that the property
belonged to the alienable and disposable land of the public domain,
and that the RTC had erred in finding that he had been in possession
of the property in the manner and for the length of time required by
law for confirmation of imperfect title. On 23 February 2007, the
Court of Appeals reversed the RTC ruling and dismissed the
application of Malabanan.

ISSUE:
Whether or not a parcel of land classified as alienable and
disposable be deemed private land and therefore susceptible to
acquisition by prescription in accordance with the Civil Code.
RULING:
As a general rule and pursuant to the Regalian Doctrine, all
lands of the public domain belong to the State and are inalienable.
Lands that are not clearly under private ownership are also presumed
to belong to the State and, therefore, may not be alienated or
disposed.
In complying with Section 14(2) of the Property Registration
Decree, consider that under the Civil Code, prescription is recognized
as a mode of acquiring ownership of patrimonial property. However,
public domain lands become only patrimonial property not only with
a declaration that these are alienable or disposable. There must also
be an express government manifestation that the property is already
patrimonial or no longer retained for public service or the
development of national wealth, under Article 422 of the Civil Code.
And only when the property has become patrimonial can the
prescriptive period for the acquisition of property of the public
dominion begin to run.
The petitioners failed to present sufficient evidence to establish
that they and their predecessors-in-interest had been in possession of
the land since June 12, 1945. Without satisfying the requisite
character and period of possession - possession and occupation that
is open, continuous, exclusive, and notorious since June 12, 1945, or
earlier - the land cannot be considered ipso jure converted to private
property even upon the subsequent declaration of it as alienable and
disposable. Prescription never began to run against the State, such
that the land has remained ineligible for registration under Section
14(1) of the Property Registration Decree. Likewise, the land
continues to be ineligible for land registration under Section 14(2) of
the Property Registration Decree unless Congress enacts a law or the
President issues a proclamation declaring the land as no longer
intended for public service or for the development of the national
wealth.

G.R. No. 186961

February 20, 2012


9

REPUBLIC OF THE PHILIPPINES, Petitioner, vs. EAST


SILVERLANE
REALTY
DEVELOPMENT
CORPORATION,
Respondent.
FACTS:
The respondent filed with the RTC an application for land
registration, covering a parcel of land identified as Lot 9039 of
Cagayan Cadastre, situated in El Salvador, Misamis Oriental and with
an area of 9,794 square meters.
The respondent purchased the portion of the subject property
consisting of 4,708 square meters (Area A) from Francisca Oco and
the remaining portion consisting of 5,086 square meters (Area B)
from Rosario U. Tan Lim, Nemesia Tan and Mariano U. Tan.
It was claimed that the respondents predecessors-in-interest
had been in open, notorious, continuous and exclusive possession of
the subject property since June 12, 1945.
The RTC granted the application and ordered the Land
Registration Authority to issue a decree in the name of the applicant
East Silverlane Realty Development Corporation. The CA likewise
affirmed the RTC's decision.
The petitioner assails the foregoing, alleging that the
respondent failed to prove that its predecessors-in-interest possessed
the subject property in the manner and for the length of time required
under Section 48 (b) of Commonwealth Act No. 141, otherwise known
as the "Public Land Act" (PLA), and Section 14 of Presidential Decree
No. 1529, otherwise known as the "Property Registration Decree"
(P.D. No. 1529). According to the petitioner, the respondent did not
present a credible and competent witness to testify on the specific
acts of ownership performed by its predecessors-in-interest on the
subject property.
ISSUE:
Whether the respondent has proven itself entitled to the
benefits of the PLA and P.D. No. 1529 on confirmation of imperfect or
incomplete titles.

RULING:
10

The Court held that the evidence submitted by the respondent


fell short of proving that it has acquired an imperfect title over the
subject property under Section 48 (b) of the PLA.
Court ruled that possession and occupation of an alienable and
disposable public land for the periods provided under the Civil Code
do not automatically convert said property into private property or
release it from the public domain. There must be an express
declaration that the property is no longer intended for public service
or development of national wealth.
Without such express declaration, the property, even if
classified as alienable or disposable, remains property of the State,
and thus, may not be acquired by prescription.
The respondent cannot register the subject property in its name
since it was not established by the required quantum of evidence that
the respondent and its predecessors-in-interest had been in open,
continuous, exclusive and notorious possession of the subject
property for the prescribed statutory period.

G.R. No. 147266

September 30, 2005


11

LUDO & LUYM DEVELOPMENT CORPORATION AND/OR CPC


DEVELOPMENT CORPORATION, Petitioners, vs. VICENTE C.
BARRETO as substituted by his heirs, namely: MAXIMA L.
BARRETO, PEREGRINA B. UY, ROGELIO L. BARRETO, VIOLETA
L. BARRETO, FLORENDA B. TEMPLANZA, EDUARDO L.
BARRETO, EVELYN B. BERSAMIN, CECILIA B. AQUINO and
NELSON NILO L. BARRETO, Respondent.
FACTS:
Vicente C. Barreto, as tenant of landowner Antonio Bartolome,
worked on and cultivated two hectares of land devoted to sugarcane
plantation. Antonio Bartolome sold the entire estate to LUDO with
the latter absorbing all the farmworkers of the former. Vicente C.
Barreto was designated as a co-overseer with Bartolome on the sixhectare coco land portion of the estate, pending the development of
the entire estate into a residential-commercial complex. . It was
agreed that the new owner, LUDO, Antonio Bartolome and
complainant Vicente C. Barreto will share in the harvests.
Ludo decided to convert the entire estate into a residentialcommercial complex and disturbance compensation was given to
some of the farmworkers; some who refused to accept the same were
eventually settled by compromise agreements. DAR then issued a
conversion permit authorizing the conversion of the entire state into a
residential/commercial lot.
Ten years later, CPC, the developer of the subject property,
wrote the Secretary of the DAR to ask for the renewal of the
conversion permit earlier issued to the owner. Barreto fervently
opposed the move. CPC then formally informed Vicente C. Barreto of
the termination of his employment as a co-overseer of the subject
landholding.
In its decision, DARAB Regional Office found that there was no
tenancy relationship existing between respondent LUDO and
complainant Vicente C. Barreto, thus, no disturbance compensation
was due the latter for having been dispossessed of the six-hectare
landholding he had been tilling.
Barreto appealed the decision to the DARAB, who thereafter
dismissed the appeal. During the pendency of the case, Barreto
passed away and was substituted by his wife and children in the
appeal. They subsequently filed a petition for review on certiorari
before the Court of Appeals. The appellate court ruled in favor of
12

petitioners-appellants heirs of Vicente C. Barreto and annulled and


set aside the DARABs decision. CPC were ordered to pay the
Barretos disturbance compensation.
ISSUE:
Whether or not there existed a tenancy relationship between
petitioner LUDO and Vicente C. Barreto.
RULING:
YES. The issue of whether or not there exists a tenancy
relationship between parties is best answered by law, specifically, The
Agricultural Tenancy Act of the Philippines. Court held that the
essential requisites of tenancy relationship are: the parties are the
landholder and the tenant; the subject is agricultural land; there is
consent; the purpose is agricultural production; and there is
consideration. All of the requisites are indispensable in order to
create or establish tenancy relationship between the parties. The
intention of a tenant to surrender the landholding cannot be
presumed by implication alone. Tenancy relations cannot be
bargained away except for the strong reasons provided by law which
must be convincingly shown by evidence.
In the case at bar, no one has denied the existence of the
tenancy status of deceased Vicente C. Barreto over the subject thirtysix-hectare landholding with respect to its former owner, Antonio
Bartolome. There being no waiver executed by deceased tenant
Barreto, no less than the law clarifies that the existence of an
agricultural tenancy relationship is not terminated by mere changes
of ownership, in cases of sale or transfer of legal possession as in
lease.
When petitioner LUDO became the owner of the subject
landholding, it became subrogated to the rights and obligations of its
predecessor-in-interest, Antonio Bartolome, his obligation under the
law to the deceased tenant, Vicente C. Barreto, continues and subsists
until terminated as provided for by law.
The Court also held that the subject landholding was just
merely reclassified and not converted. Reclassification is very much
different from conversion. Conversion is the act of changing the
current use of a piece of agricultural land into some other use as
approved by the DAR. Reclassification, in contrast, is the act of
specifying how agricultural lands shall be utilized for non-agricultural
uses such as residential, industrial or commercial, as embodied in the
13

land use plan, subject to the requirements and procedure for land use
conversion.
Accordingly, a mere reclassification of agricultural land does
not automatically allow a landowner to change its use and thus cause
the ejectment of the tenants. Parties can still continue with their
tenurial relationship even after such reclassification. He has to
undergo the process of conversion before he is permitted to use the
agricultural land for other purposes.

G.R. No. 157285

February 16, 2007


14

WOODRIDGE SCHOOL, INC., and MIGUELA JIMENEZ-JAVIER,


Petitioners, vs. ARB CONSTRUCTION CO., INC., Respondent.
FACTS:
Woodridge is the usufructuary of a parcel of land in the name of
spouses Ernesto T. Matugas and Filomena U. Matugas. Its copetitioner, Miguela Jimenez-Javier, is the registered owner of the
adjacent lot to that of Woodridge.
On the other hand, ARB is the owner and developer of Soldiers
Hills Subdivision in Bacoor, Cavite, which is composed of four phases.
Phase I of the subdivision was already accessible from the Marcos
Alvarez Avenue. To provide the same accessibility to the residents of
Phase II of the subdivision, ARB constructed the disputed road to link
the two phases.
Petitioners' properties sit right in the middle of several estates.
Initially, petitioners offered to pay ARB P50,000 as indemnity for the
use of the road. ARB refused the offer and fenced the perimeter of the
road fronting the properties of petitioners. By doing so, ARB
effectively cut off petitioners' access to and from the public highway.
After failing to settle the matter amicably, Woodridge filed a
complaint to enjoin ARB from depriving them of the use of the
disputed subdivision road and to seek a compulsory right of way after
payment of proper indemnity.
Trial court rendered its decision in favor of Woodridge. The
appellate court reversed the decision of the lower court.
ISSUE:
Whether or not the road lots in a private subdivision are public
property.
RULING:
The road lots in a private subdivision are private property,
hence, the local government should first acquire them by donation,
purchase, or expropriation, if they are to be utilized as a public road.
Otherwise, they remain to be private properties of the ownerdeveloper.
The use of the subdivision roads by the general public does not
strip it of its private character. The road is not converted into public
15

property by mere tolerance of the subdivision owner of the public's


passage through it. The law is clear. The transfer of ownership from
the subdivision owner-developer to the local government is not
automatic but requires a positive act from the owner-developer before
the city or municipality can acquire dominion over the subdivision
roads. Therefore, until and unless the roads are donated, ownership
remains with the owner-developer.

G.R. No. 152115

January 26, 2005

16

NIMFA USERO, petitioner, vs. COURT OF APPEALS and SPS.


HERMINIGILDO & CECILIA POLINAR, respondents.
FACTS:
Petitioners and the private respondent are registered owners of
neighboring parcels of land wherein between the lots is a low-level
strip of land with stagnant body of water. Whenever there is a storm
or heavy rain, the water therein would flood thereby causing damage
to houses of the Polinars prompting them to build a concrete wall on
the bank of the strip of land about 3meters from their house and
riprapped the soil in that portion.
The Useros claimed ownership of the strip, demanded the halt
of the construction but the Polinars never heeded believing that the
strip is part of a creek. However, the Polinars offered to pay for the
land. As the parties still failed to settle, both filed separate complaints
for forcible entry. The Municipal Trial Court ruled in favor of the
petitioner, while the regional trial court reversed and ordered the
dismissal of the complaint and confirmed the existence of the creek
between the lots.
ISSUE:
Whether or not the disputed strip of land is part of the creek
hence part of public domain.
RULING:
Yes. Art. 420 of the Philippine New Civil Code (NCC) provides
for properties which are part of public domain. A creek is included in
the phrase "and others of similar character". A creek, which refers to a
recess or arm of a river, is a property belonging to the public domain,
therefore not susceptible of private ownership. Being public water, it
cannot be registered under the Torrens system under the name of any
individual.
The phrase "others of similar character" includes a creek which
is a recess or an arm of a river. It is property belonging to the public
domain which is not susceptible to private ownership. Being public
water, a creek cannot be registered under the Torrens System in the
name of any individual.

G.R. No. 155051

May 29, 2007


17

RURAL BANK OF ANDA, INC., Petitioner, vs. ROMAN CATHOLIC


ARCHBISHOP OF LINGAYEN- DAGUPAN, Respondent.
FACTS:
The lot in dispute is Cadastral Lot 736 (Lot 736) located in the
Poblacion of Binmaley, Pangasinan.
The Sangguniang Bayan of Binmaley, Pangasinan, passed and
approved Resolution Nos. 1045 and 105. Resolution No. 104
converted Lot 736 from an institutional lot to a commercial lot.
Resolution No. 105 authorized the municipal mayor to enter into a
contract of lease for 25 years with the Rural Bank of Anda over a
portion of Lot 736 with an area of 252 square meters.
Fr. Arenos, the director of the seminary, discovered that a
sawali fence was being constructed enclosing a portion of Lot 736.
Mayor Domalanta and Fr. Arenos agreed that the construction of the
building for the Rural Bank of Anda should be stopped. Respondent
requested Mayor Domalanta to remove the sawali fence and restore
the concrete fence. Later, Mayor Domalanta informed respondent
that the construction of the building of the Rural Bank of Anda would
resume but that he was willing to discuss with respondent to resolve
the problem concerning Lot 736.
Respondent filed a complaint for Abatement of Illegal
Constructions, Injunction and Damages with Writ of Preliminary
Injunction in the Regional Trial Court of Lingayen, Pangasinan.
The trial court rendered a decision in favor of the plaintiff. The
trial court found that Lot 736 is not covered by any Torrens title
either in the name of respondent or in the name of the Municipality of
Binmaley. The trial court held that Lot 736 is public in nature. Since
Lot 736 is property of public dominion, it is outside the commerce of
man. The CA affirmed the decision of the trial court.
ISSUE:
The issue in this case is whether Resolution Nos. 104 and 105 of
the Sangguniang Bayan of Binmaley are valid.

RULING:
18

The records show that Lot 736 is used as a pathway going to the
school, the seminary, or the church, which are all located on lots
adjoined to Lot 736.14 Lot 736 was also used for parking and
playground. In other words, Lot 736 was used by the public in
general.
Both respondent and the Municipality of Binmaley failed to
prove their right over Lot 736. Since Lot 736 has never been acquired
by anyone through purchase or grant or any other mode of
acquisition, Lot 736 remains part of the public domain and is owned
by the state.
It is well settled "that no public land can be acquired by private
persons without any grant, express or implied, from the government."
Municipal corporations cannot appropriate to themselves public or
government lands without prior grant from the government. Since
Lot 736 is owned by the state, the Sangguniang Bayan of Binmaley
exceeded its authority in passing Resolution Nos. 104 and 105. Thus,
Resolution Nos. 104 and 105 are void and consequently, the contract
of lease between the Municipality of Binmaley and the Rural Bank of
Anda over a portion of Lot 736 is also void.

G.R. No. 164584

June 22, 2009


19

PHILIP MATTHEWS, Petitioner, vs. BENJAMIN A. TAYLOR and


JOSELYN C. TAYLOR, Respondents.
FACTS:
Benjamin Taylor, a British, was married to Joselyn Taylor, a
Filipina. While their marriage was subsisting, Joselyn bought a
Boracay property in consideration of P129,000 in 1989.The sale was
allegedly financed by Benjamin. They constructed improvements
thereon and eventually converted the property to a vacation and
tourist resort known as the Admiral Ben Bow Inn.
However, Benjamin and Joselyn had a falling out. Joselyn
executed a Special Power of Attorney (SPA) in favor of Benjamin,
authorizing the latter to maintain, sell, lease, and sub-lease and
otherwise enter into contract with third parties with respect to their
Boracay property.
In 1992, Joselyn as lessor and petitioner Philip Matthews as
lessee, entered into an Agreement of Lease involving the Boracay
property for a period of 25 years. The agreement was signed by the
parties and executed before a Notary Public. Petitioner thereafter
took possession of the property and renamed the resort as Music
Garden Resort.
Benjamin opposed the agreement and claims that it was null
and void since it was entered into by Joselyn without his consent.
Benjamin claimed that his funds were used in the acquisition and
improvement of the Boracay property, and coupled with the fact that
he was Joselyns husband, any transaction involving said property
required his consent.
Matthews claimed good faith in transacting with Joselyn. Since
Joselyn appeared to be the owner of the Boracay property, he found it
unnecessary to obtain the consent of Benjamin. Moreover, as
appearing in the Agreement, Benjamin signed as a witness to the
contract, indicating his knowledge of the transaction and, impliedly,
his conformity to the agreement entered into by his wife. Benjamin
was, therefore, estopped from questioning the validity of the
Agreement.
RTC rendered a judgment in favor of Benjamin. The RTC
considered the Boracay property as community property of Benjamin
and Joselyn; thus, the consent of the spouses was necessary to
validate any contract involving the property. On appeal to the CA,
20

Matthews still failed to obtain a favorable decision. Hence, the


petition for review on certiorari before the SC.
ISSUE:
Whether or not the consent of Benjamin is required in the
Agreement of Lease.
RULING:
No. Sec. 7, Art. XII of the 1987 Constitution states that "aliens,
whether individuals or corporation, have been disqualified from
acquiring lands of the public domain."
The rule is clear and inflexible: aliens are absolutely not allowed
to acquire public or private lands in the Philippines. The Court held
that Benjamin has no right to nullify the Agreement of Lease between
Joselyn and Matthews. Benjamin, being an alien, is absolutely
prohibited from acquiring private and public lands in the Philippines
even if he provided the funds for such acquisition. Considering that
Joselyn appeared to be the designated "vendee" in the Deed of Sale of
said property, she acquired sole ownership thereto. By entering into
such contract knowing that it was illegal, no implied trust was created
in his favor; no reimbursement for his expenses can be allowed; and
no declaration can be made that the subject property was part of the
conjugal/community property of the spouses.

G.R. No. 161107

March 12, 2013


21

HON. MA. LOURDES C. FERNANDO, in her capacity as City Mayor


of Marikina City, JOSEPHINE C. EVANGELIST A, in her capacity as
Chief, Permit Division, Office of the City Engineer, and ALFONSO
ESPIRITU, in his capacity as City Engineer of Marikina City,
Petitioners, vs. ST. SCHOLASTICA'S COLLEGE and ST.
SCHOLASTICA'S ACADEMY-MARIKINA, INC., Respondents.
FACTS:
St. Scholasticas College (SSC) and St. Scholasticas AcademyMarikina, Inc. (SSA-Marikina) are educational institutions. SSC is the
owner of four (4) parcels of land. The property is enclosed by a tall
concrete perimeter fence built some thirty (30) years ago. Abutting
the fence along the West Drive are buildings, facilities, and other
improvements. The petitioners are the officials of the City
Government of Marikina. The city enacted an ordinance regulating
the construction of fences and walls. Section 3 of such ordinance
limits the height of fences or walls on the front yard which shall be no
more than one (1) meter in height. Fences in excess of one (1) meter
shall be of an open fence type, at least eighty percent (80%) see-thru.
The SSC argued that the ordinance contravenes Section 1,
Article III of the 1987 Constitution. That the implementation of the
ordinance on their property would be tantamount to an appropriation
of property without due process of law; and that the petitioners could
only appropriate a portion of their property through eminent domain.
They also pointed out that the goal of the provisions to deter lawless
elements and criminality did not exist as the solid concrete walls of
the school had served as sufficient protection for many years.
RTC rendered a decision in favor of SSC. RTC agreed with SSC
that the order of the city to demolish the fence and to move it back six
(6) meters would amount to an appropriation of property which could
only be done through the exercise of eminent domain. It further
found that the 80% see-thru fence requirement could run counter to
the respondents right to privacy, considering that respondents were
entitled to some sense of privacy in their affairs.
The CA affirmed the RTC decision. The CA reasoned out that
the objectives stated in Ordinance No. 192 did not justify the exercise
of police power, as it did not only seek to regulate, but also involved
the taking of the respondents property without due process of law.
Hence, the petitioners appealed before the SC.
ISSUE:
22

Whether or not the city ordinance is a valid exercise of police


power.
RULING:
No. As with the State, local governments may be considered as
having properly exercised their police power only if the following
requisites are met: (1) the interests of the public generally, as
distinguished from those of a particular class, require its exercise and
(2) the means employed are reasonably necessary for the
accomplishment of the purpose and not unduly oppressive upon
individuals. In short, there must be a concurrence of a lawful subject
and lawful method.
The Court joins the CA in finding that the real intent of the
setback requirement was to make the parking space free for use by
the public, considering that it would no longer be for the exclusive use
of the respondents as it would also be available for use by the general
public. Section 9 of Article III of the 1987 Constitution, a provision on
eminent domain, provides that private property shall not be taken for
public use without just compensation. Also, the implementation of
the setback requirement would be tantamount to a taking of a total of
3,762.36 square meters of the respondents private property for
public use without just compensation, in contravention to the
Constitution.
The ultimate goal of this objective is clearly the prevention of
crime to ensure public safety and security. The means employed by
the petitioners, however, is not reasonably necessary for the
accomplishment of this purpose and is unduly oppressive to private
rights. The petitioners have not adequately shown that an 80% seethru fence would provide better protection and a higher level of
security, or serve as a more satisfactory criminal deterrent, than a tall
solid concrete wall.
The State may not, under the guise of police power, infringe on
private rights solely for the sake of the aesthetic appearance of the
community. Similarly, the Court cannot perceive how a see-thru fence
will foster "neighborliness" between members of a community.
Compelling the respondents to construct their fence in accordance
with the assailed ordinance is, thus, a clear encroachment on their
right to property, which necessarily includes their right to decide how
best to protect their property.
G.R. No. 158687

January 27, 2006


23

FRISCO F. DOMALSIN, Petitioner, vs.


SPOUSES JUANITO VALENCIANO and AMALIA VALENCIANO,
Respondents.
FACTS:
The property subject of this action for forcible entry is a parcel
of land located at sitio Riverside, Camp 3, Tuba, Benguet. Frisco B.
Domalsin claims to be the lawful owner and possessor of said parcel
of land since 1979 up to the present. He declared it for taxation
purposes and allegedly introduced improvements thereon. He was in
continuous, adverse possession and in the concept of an owner for the
past nineteen (19) years.
On August 1, 1998, Sps. Valenciano allegedly entered the
premises to construct a building made of cement and strong materials
without the authority and consent of Domalsin by means of force and
strategy, and without a building permit from the DPWH.
Respondents claimed that the ongoing construction was with
the consent and conformity of the DPWH and in fact the
improvements found in the property were introduced by the residents
thereof, including its first residents, William and Gloria Banuca, and
not by Domalsin. The premises on which petitioners Sps. Valenciano
are constructing their house were leveled after the earthquake in 1990
by the Banuca spouses. Petitioners Sps. Valenciano are just starting
the construction because the permission was only given now by Gloria
Banuca.
What is being contested is the possession of a portion of the
road-right-of way of Kennon Road which is located in front of a parcel
of land that Domalsin bought by way of Deed of Waiver and
Quitclaim from Castillo Binay-an.
MCTC and RTC ruled that Frisco F. Domalsin is the actual
possessor of the lot in dispute and ordered the Sps. Valenciano to
vacate and deliver the physical possession thereof to the petitioner. It
held that petitioner had prior material possession over the subject
land.
CA, however, reversed the decision. Hence, this appeal.
ISSUE:
24

Whether or not petitioner is the rightful owner of the subject


property.
RULING:
Neither the petitioner nor the respondents can own nor possess
the subject property the same being part of the public dominion.
Properties of public dominion are owned by the general public. Public
use is "use that is not confined to privileged individuals, but is open to
the indefinite public." As the land in controversy is a portion of
Kennon Road which is for the use of the people, there can be no
dispute that same is part of public dominion. This being the case, the
parties cannot appropriate the land for themselves. Thus, they cannot
claim any right of possession over it.
The fact that the parties do not and cannot own the property
under litigation does not mean that the issue to be resolved is no
longer priority of possession. The determining factor for one to be
entitled to possession will be prior physical possession and not actual
physical possession. Since title is never in issue in a forcible entry
case, the Court of Appeals should have based its decision on who had
prior physical possession. The main thing to be proven in an action
for forcible entry is prior possession and that same was lost through
force, intimidation, threat, strategy and stealth, so that it behooves
the court to restore possession regardless of title or ownership.

G.R. No. 140798

September 19, 2006


25

MARCELITO D. QUEVADA, petitioner, vs. COURT OF APPEALS and


JUANITO N. VILLAVERDE, respondents.
FACTS:
Juanito is the lessor of a parcel of land with a residential house
in Sampaloc, Manila. Sometime in 1994, he and Quevada entered into
a Contract of Lease of a portion of the residential house (consisting of
96 square meters) which is located on the subject property for the
period from August 15, 1994 to August 15, 1995, at a monthly rental of
P2,500. After expiration of the lease, they entered into another
Contract of Lease, which was an extension of the previous date. After
the expiration of the extended Lease, Marcelito continued possessing
the premises, but without payment of any reasonable compensation.
Juanito made several demands to the petitioner to vacate the
premises but was refused. Because of the Marcelitos refusal to vacate
the premises, Juanito referred the matter to the barangay court for
conciliation, only for the former to repudiate the "agreement to vacate
as of December 31, 1997." A notice to vacate the leased property was
served upon Quevada. Quevada answered that he started building the
house on the lot which was finished in 1986 at which time he
occupied the house as his residence. Juanito in turn "advised" him
that he would go ahead and buy the lot but with an assurance that as
soon as Marcelito would be in a financial position to do payment, the
former will transfer the title to the latter. Thus, a Lease Contract, in
the meantime, was executed, for him to pay the rentals at P2,500 a
month, but only with respect to the land, since the house belonged to
him. Juanito did not give him a chance to pay the purchase price by
setting a deadline to do the payment; similarly, he respondent refused
to accept the monthly rental of the lot for P2,500.
Metropolitan Trial Court (MeTC) ruled in favor of Juanito. RTC
and CA affirmed the decision of MeTC.
ISSUE:
1) Whether the action for ejectment is proper; 2) whether such
action can be brought by private respondent who is not the titled
owner of the property; 3) whether petitioner can be reimbursed for
the value of the house on the property; and 4) whether there is an
implied trust.

RULING:
26

As a lessor, Juanito was unlawfully deprived possession of the


residential house after Marcelito's right to its possession as lessee had
expired on April 15, 1996. Despite several demands given by the
former to vacate the premises, the latter refused and even repudiated
the agreement to vacate, which was entered into before the barangay
court.
Petitioner's continued use and occupancy of the premises
without any contract between him and private respondent was by
mere tolerance or permission of the latter. Possessory acts, no matter
how long so continued, do not start the running of the period of
prescription. Possession by tolerance is lawful, but such possession
becomes unlawful when the possessor by tolerance refuses to vacate
upon demand made by the owner. A person who occupies the land of
another at the latter's tolerance or permission, without any contract
between them, is necessarily bound by an implied promise to vacate
upon demand, failing which, a summary action for ejectment is the
proper remedy.
Its filing was within the one-year period after Juanito had been
unlawfully deprived or withheld of its possession. The unlawful
deprivation or withholding of possession started not from the date the
lease contract expired, but from the date the written notice to vacate
was served. Juanito may bring the action for unlawful detainer, even
though he is not the titled owner of the leased property. The only
issue to be resolved in unlawful detainer or desahucio is the actual
physical or material possession of the property involved, independent
of any claim of ownership by any of the party litigants.
Marcelito is necessarily in prior lawful possession of the
property, but his possession eventually becomes unlawful upon
termination or expiration of his right to possess. His prior physical
possession of the leased property does not automatically entitle him
to continue in its possession and does not give him a better right to
the property.
Court held that petitioner should be paid for the value of the
portion of the house covered by the lease, to be offset against rentals
due. The fundamental doctrine of unjust enrichment is the transfer of
value without just cause or consideration. Therefore, to have a just
transfer of the leased portion of the house, its value should be offset
against the reasonable rent due for its continued use and occupancy
until the former vacates and surrenders it to the latter.
G.R. No. 131726

May 7, 2002
27

YOLANDA PALATTAO, petitioner, vs. THE COURT OF APPEALS,


HON. ANTONIO J. FINEZA, as Presiding Judge of the Regional Trial
Court of Caloocan City, Branch 131 and MARCELO CO, respondents.
FACTS:
Petitioner Yolanda Palattao entered into a lease contract
whereby she leased to private respondent a house and a 490-squaremeter lot located in 101 Caimito Road, Caloocan City. The duration of
the lease contract was for three years. The contract gave Marcelo,
respondent lessee, the first option to purchase the leased property.
During the last year of the contract, the parties began
negotiations for the sale of the leased premises. Yolanda offered to
sell to private respondent 413.28 square meters of the leased lot at
P7,800.00 per square meter. Marcelo manifested his desire to buy the
whole 490-square-meter leased premises and inquired from Yolanda
the reason why only 413.28 square meters of the leased lot were being
offered for sale.
Yolanda made a final offer to sell the lot at P7,500.00 per
square meter with a down payment of 50% upon the signing of the
contract of conditional sale. Marcelo accepted petitioners offer and
reiterated his request for clarification as to the size of the lot for sale.
Petitioner acknowledged private respondents acceptance of the offer
in his letter dated November 10, 1993. Petitioner gave private
respondent on or before November 24, 1993, within which to pay the
50% down payment in cash or managers check. Petitioner stressed
that failure to pay the down payment on the stipulated period will
enable petitioner to freely sell her property to others. Petitioner
likewise notified private respondent that she is no longer renewing
the lease agreement upon its expiration on December 31, 1993.
Marcelo did not accept the terms proposed by petitioner and
wrote a letter to petitioner manifesting his intention to exercise his
option to renew their lease contract for another three years. Yolanda
declined to renew the lease and demanded that Marcelo vacate the
premises but the latter refused.
Metropolitan Trial Court rendered a decision in favor of
Yolanda. RTC, on the other hand, reversed the decision of MeTC.
Petitioner filed a petition for review with the Court of Appeals, which
dismissed the petition.
ISSUE:
Whether or not the ejectment case will prosper.
28

RULING:
Yes. Court held that the preservation of the status quo agreed
upon by the parties applied only during the period of negotiations for
an amicable settlement and cannot be construed to be effective for the
duration of the pendency of the specific performance case.
An "accion publiciana" does not suspend an ejectment suit. An
action for reconveyance of property or "accion reivindicatoria" also
has no effect on ejectment suits regarding the same property.
In the case at bar, the continued occupation by private
respondent of the leased premises is conditioned upon his right to
acquire ownership over said property.
Contracts that are consensual in nature, like a contract of sale,
are perfected upon mere meeting of the minds. Once there is
concurrence between the offer and the acceptance upon the subject
matter, consideration, and terms of payment, a contract is produced.
The offer must be certain. While it is true that Marcelo informed
Yolanda that he is accepting the latters offer to sell the leased
property, it appears that they did not reach an agreement as to the
extent of the lot subject of the proposed sale. Marcelo did not give his
consent to buy only 413.28 square meters of the leased lot, as he
desired to purchase the whole 490 square-meter-leased premises
which, however, was not what was exactly proposed in petitioners
offer. Therefore, private respondents acceptance of petitioners offer
was not absolute, and will consequently not generate consent that
would perfect a contract.
Considering that the lease contract was not renewed after its
expiration on December 31, 1991, private respondent has no more
right to continue occupying the leased premises. Consequently, his
ejectment therefrom must be sustained.

G.R. No. 157806

November 22, 2007


29

SPOUSES SHEIKDING BOOC and BILY BOOC, petitioners, vs. FIVE


STAR MARKETING CO., INC., respondent.
FACTS:
Five Star Marketing Co., Inc. filed with the MTCC of Iligan City
a Complaint for unlawful detainer against the spouses Sheikding and
Bily Booc. Five Star is the owner of the land and building situated in
Quezon Avenue, Iligan City. That Sps. Booc are the present occupants
of the 3rd floor premises of the building, who were allowed to live
temporarily in the premises for free.
Five Star notified all building occupants that it had withdrawn
the privilege granted (rental free) to them coupled with a notice of
rental rates in each premises concerned, and further required to any
interested occupants to negotiate and sign a lease agreement or
vacate the property otherwise. The petitioners were notified but
ignored the demand and a letter of demand to vacate the premises
was sent to the petitioners.
Sps. Booc contended that Five Star has no cause of action
against them as they are actually the owners of the portion of the
building that they are occupying; that the said property is owned in
common by petitioner Sheikding and his brother, Rufino Booc.
MTCC rendered a judgment in favor of the Sps. Booc and
against Five Star. RTC affirmed with modification the assailed
decision of the MTCC. Aggrieved by the judgment of the RTC,
respondent filed a petition for review with the CA and the petition
was granted.
ISSUE:
Whether or not the ejectment suit will prosper.
RULING:
Yes. Five Star has proved, by preponderance of evidence, its
claim that it is the owner of the disputed properties and, therefore,
has the right of material possession over the same.
Petitioners' claim of co-ownership is anchored on their
assertion that it was petitioner Sheikding together with Rufino who
actually purchased the subject lot; that they were also the ones who
financed the construction of the subject building; and that they paid
30

the taxes due on the subject properties. In claiming that the subject
lot and building were bought and constructed with the money of
petitioner Sheikding and Rufino, petitioners, in effect, aver that
respondent is merely holding the property in trust for them.
As a rule, the burden of proving the existence of a trust is on the
party asserting its existence and such proof must be clear and
satisfactorily show the existence of the trust and its elements. Aside
from the Joint Affidavit, no other competent evidence was presented
to support petitioners' allegation of ownership of the lot in question.
It is settled that a certificate of title is a conclusive evidence of
ownership; it does not even matter if the title is questionable, the
instant action being an ejectment suit. In addition, the age-old rule is
that the person who has a Torrens Title over a land is entitled to
possession thereof.

31

G.R. No. 175561

October 20, 2010

SPOUSES IDA aka "MILAGROS" NIEVES BELTRAN and JOSE


BELTRAN, Petitioners, vs. ANITA R. NIEVES, represented by NELIA
G. MORAN, Respondent.
FACTS:
Nieves is the registered owner of the subject parcel of land as
well as the house thereon. Milagros Beltran is Nieves niece, being the
daughter of Gaston, Nieves brother. In asserting their ownership and
rightful occupation against Nieves, petitioners spouses Beltran claim
that Nieves sold the land and house to Gaston. The deed of sale,
which Nieves disclaims having signed, remains unregistered.
Nieves filed an ejectment suit against Sps. Beltran which seeks
to eject the latter from a 474 square meter parcel of land covered by
Transfer Certificate of Title (TCT) No. T-10963 and the house erected
thereon both registered in the name of petitioner.
Nieves claimed that for a long period of time, she tolerated the
occupation of the subject property by her sister Julieta (Tita) Nieves
and her brother Gaston Nieves and the latters children, respondent
Milagros and Talin, among others. However, despite repeated
demands, respondent Milagros and her husband refused to vacate the
subject premises and surrender its possession to petitioner.
MCTC promulgated its decision in favor of the spouses Beltran
and dismissed Nieves complaint. The MCTC made a provisional
ruling that Gaston Nieves, Milagros Nieves father, owned the subject
lot by virtue of the unregistered deed of sale by Nieves to Gaston. RTC
affirmed the MCTCs decision. The RTC held that the MCTC had no
jurisdiction to entertain the question of just title. Moreover, an action
for unlawful detainer should be filed within one year from the
unlawful withholding of possession. Therefore, the complaints failure
to allege with clarity the specific date that the withholding of
possession became unlawful was a fatal jurisdictional lapse. Nieves
then filed a Petition for Review before the CA.
CA ruled in favor of Nieves. CA ruled that the MCTC and the
RTC erred in declaring that the Spouses Beltran are entitled to
possess the subject property on the basis of the unregistered deed of
sale. The tax declarations presented by the spouses Beltran are not
conclusive evidences of ownership, but are good indicators of
possession in the concept of an owner. The CA further ruled that a
32

certificate of title is conclusive evidence of ownership. Nieves, who


holds title to the land, is thus entitled to possession of the land.
ISSUES:
Won Spouses Beltran have no right over the property based on
an unregistered deed of sale.
RULING:
Court agreed with the CA. Registered owners such as Nieves are
entitled to the possession of the property covered by the title from the
time such title was issued in their favor. The only issue in an
ejectment case is the physical possession of real property possession
de facto and not possession de jure. Court rules upon the issue of
ownership only to determine who between the parties has the better
right of possession.
A person who occupies the land of another at the latters
tolerance or permission, without any contract between them, is
necessarily bound by an implied promise that he will vacate upon
demand, failing which a summary action for ejectment is the proper
remedy against them. Whatever right of possession that the spouses
Beltran may have over the subject property cannot prevail over that of
Nieves for the simple reason that Nieves is the registered owner of the
subject property and the alleged deed of sale, which Nieves disputes,
remains unregistered. Although it is true that the spouses Beltran,
and not Nieves, were in prior physical possession of the subject
property, this argument cannot hold water as prior physical
possession is material only in forcible entry cases.

33

G.R. No. 105760 July 7, 1997


PHILIPPINE NATIONAL BANK, petitioner, vs. COURT OF
APPEALS, HON. JUDGE OF THE REGIONAL TRIAL COURT OF
GAPAN, NUEVA ECIJA, BR. 34, and NILDEFONSO MONTANO,
respondents.
FACTS:
Sps. Crisanto de la Cruz and Pepita Montano mortgaged two
parcels of land to petitioner PNB for a loan of P24,000.00. PNB
extrajudicially foreclosed the mortgage and was the only bidder at the
public auction sale; thus, a Certificate of Sale over said lots was issued
in favor of PNB.
PNB filed before the RTC a Petition for the Issuance of a Writ of
Possession, alleging therein that by virtue of a foreclosure sale
wherein it purchased the subject properties and due to the
mortgagors' (spouses Crisanto de la Cruz and Pepita Montano) failure
to redeem the property within a period of one year, it had become the
absolute owner of the same and is entitled to a Writ of Possession.
The petition was granted by the RTC.
Before implementation of the writ, Montano filed a Motion for
the Dissolution of the Writ of Possession alleging that: (1) he was
instituted as tenant on the subject property even before 1972 by the
former owners of the land; (2) the two lots are the subject matters of
CAR Case; (3) after the foreclosure of the subject land, Montano's
counsel wrote PNB of the pending case between the mortgagors and
private respondent as tenant on the land; (4) the issuance of said Writ
in PNB's favor would work grave injustice to him and violate his
rights under P.D. 27, P.D. 36, P.D. 583, and other laws and legal
issuances on land reform; (5) he was issued a certification by the
Cabiao-San Isidro Agrarian Reform Team that he is an agricultural
lessee in the subject landholding and another certification that he is
an active member of the Samahang Nayon; and (6) in line with the
ruling in "Clapano vs. Gapultos" (132 SCRA 429) that possession of
property is given to a purchaser in Extra-Judicial foreclosure unless a
third-party is actually holding the property adversely to the judgment
debtor, he is to be considered a "third person".
RTC granted Montano's motion to dissolve the writ of
possession. The case was then referred to the CA. CA rendered
judgment in favor of petitioner PNB then reversed its decision when
Montano filed a motion thereby dissolving the Writ of Possession.

34

ISSUE:
WON petitioner PNB is entitled to a Writ of Possession of the
land in question
RULING:
PNB is not entitled to a writ of possession, as the same may be
issued in extrajudicial foreclosure of real estate mortgage only if the
debtor is in possession and no third person had intervened. Such
requisite is evidently lacking in the case at bar, as it has been
established that Montano has been in possession and finally adjudged
as the tenant on the landholding in question.
The decision in CAR proves that Montano is indeed a tenant of
the landholding. The judgment in the agrarian suit is conclusive upon
petitioner PNB.
Petitioner PNB further insists that as absolute owner of the
properties, under Art. 428 and 429 of the New Civil Code, it has the
right to possess and dispose of the same. These very provisions cited,
however, show that the exercise of the rights of ownership is subject
to limitations that may be imposed by law. Under Art. 428 of the Civil
Code, the owner has the right to dispose of a thing without other
limitations than those established by law. As an incident of
ownership, therefore, there is nothing to prevent a landowner form
donating his naked title to the land. However, the new owner must
respect the rights of the tenant.
The Court further held that the agricultural lessee's rights are
enforceable against the transferee or the landowner's successor-ininterest. Therefore, Montano may enforce his right of possession
against petitioner PNB. Petitioner PNB may not, by way of defense,
argue that its right over the land is superior to Montano's claim on
the subject properties since the agricultural lease was not annotated
on the Transfer Certificate of Title and, therefore, it dealt with the
properties in good faith.
Even if the fact of tenancy had not been reflected on the title,
PNB admitted that before they consented to the mortgage, an ocular
inspection was conducted on the landholding on the occasion of
which, PNB's Credit Investigator already found Montano staying on
the land and even interviewed the latter. Upon the interview,
Montano allegedly said that he had been allowed to stay on the
property in question because he was ejected from the adjacent parcel
of land which he used to till.
35

The land being an agricultural one, and considering the ocular


inspection conducted sometime in 1978 when P.D. 27 had been in
effect for some time, petitioner PNB's suspicion that the land was
tenanted should have been aroused by the existence of a farmer on
the land other than the mortgagors themselves. It cannot be denied
that PNB had been put on notice by its actual knowledge of another
person possessing the land, no matter what the given reason may
have been for private respondent Montano's occupancy of the
properties in question.
Furthermore, as purchaser at a public auction, petitioner PNB
was only substituted to and acquired the right, title, interest and
claim of the judgment debtor or mortgagor to the property as of the
time of the levy. In this case, the only remaining right of the
mortgagors (spouses Crisanto de la Cruz and Pepita Montano) at the
time of levy is the right to be paid a reasonable price for the land they
owned as mandated by P.D. 27. That is the only right which petitioner
PNB acquired as the new absolute owner of the land.

36

G.R. No. 175025

February 15, 2012

ROGELIO J. JAKOSALEM and GODOFREDO B.


Petitioners, vs. ROBERTO S. BARANGAN, Respondent.

DULFO,

FACTS:
Col. Roberto S. Barangan purchased a 300 square meter parcel
of land; the old title was cancelled and a new one was issued in his
name. Since then, he has been dutifully paying real property taxes for
the said property. He was not, however, able to physically occupy the
subject property because as a member of the Philippine Air Force, he
was often assigned to various stations in the Philippines.
When he was about to retire from the government service,
Barangan went to visit his property and he discovered that it was
being occupied by petitioner Godofredo Dulfo and his family.
Barangan sent a letter to Dulfo demanding that he and his
family vacate the subject property within 30 days. In reply, petitioner
Atty. Rogelio J. Jakosalem, the son-in-law of petitioner Dulfo, sent a
letter claiming ownership over the subject property.
Barangan commissioned a geodetic engineer to conduct a
relocation survey of the subject property based on the technical
description appearing on respondent Barangans TCT. The relocation
survey revealed that the property occupied by Dulfo and his family is
the same property covered by Barangans title.
Barangan filed a Complaint for Recovery of Possession with the
RTC against Jakosalem. In their answer, Dulfo and Jakosalem
claimed that the subject property was assigned to Jakosalem by Mr.
Nicanor Samson; that they have been in possession of the subject
property since May 8, 1979; and that the property covered by
respondent Barangans title is not the property occupied by petitioner
Dulfo and his family.
RTC issued an order directing the engineer of DENR to conduct
a survey of land; however, it did not push through because the
defense its request for an ocular inspection claiming that it was no
longer necessary.
RTC rendered a decision against Barangan for failure to present
sufficient evidence to prove his claim. The RTC further said that even
37

if the subject property is owned by Barangan, prescription and laches


have already set in; thus, Barangan may no longer recover the same.
CA reversed the findings of the RTC. Hence, the appeal.
ISSUE:
Whether or not the property occupied by Dulfo is the same
property claimed by Barangan;
WON laches and prescription barred the filing of the case.
RULING:
Barangan is entitled to recover the subject property. Article 434
of the Civil Code provides that "in an action to recover, the property
must be identified, and the plaintiff must rely on the strength of his
title and not on the weakness of the defendants claim." In other
words, in order to recover possession, a person must prove (1) the
identity of the land claimed, and (2) his title.
In this case, Barangan was able to prove the identity of the
property and his title. To prove his title to the property, he presented
in evidence the following documents: (1) Land Purchase Agreement;
(2) Deed of Absolute Sale; (3) and a Torrens title registered under his
name, TCT No. N-10772.
Records also show that during the trial, the RTC ordered the
DENR to conduct a resurvey of the subject property; but petitioners
moved that the same be abandoned claiming that the resurvey would
only delay the proceedings. The persistent refusal of petitioners to
participate in the relocation survey does not speak well of their claim
that they are not occupying respondent Barangans property.
The Court held that prescription and laches cannot apply to
registered land covered by the Torrens system" because "under the
Property Registration Decree, no title to registered land in derogation
to that of the registered owner shall be acquired by prescription or
adverse possession."

38

G. R. No. 185124

January 25, 2012

REPUBLIC OF THE PHILIPPINES, represented by the NATIONAL


IRRIGATION ADMINISTRATION (NIA),Petitioner, vs. RURAL
BANK OF KABACAN, INC., LITTIE SARAH A. AGDEPPA, LEOSA
NANETTE AGDEPPA and MARCELINO VIERNES, MARGARITA
TABOADA, PORTIA CHARISMA RUTH ORTIZ, represented by LINA
ERLINDA A. ORTIZ and MARIO ORTIZ, JUAN MAMAC and
GLORIA MATAS, Respondents.
FACTS:
National Irrigation Authority (NIA) is a government-ownedand-controlled corporation and is primarily responsible for irrigation
development and management in the country. NIA was specifically
authorized under P.D. 552 to exercise the power of eminent domain.
NIA needed some parcels of land for the purpose of constructing the
Malitubog-Marigadao Irrigation Project. It filed with the RTC of
Kabacan, Cotabato a complaint for the expropriation of a portion of
three (3) parcels of land.
After filing their amended complaints, NIA prayed that it be
authorized to take immediate possession of the properties after
depositing with the Philippine National Bank the amount of P
19,246.58 representing the provisional value of the properties.
Respondents responded and alleged that NIA had no authority
to expropriate portions of their land because it was not a sovereign
political entity; that it was not necessary to expropriate their
properties, because there was an abandoned government property
adjacent to theirs, where the project could pass through. That NIAs
valuation of their expropriated properties was inaccurate because of
the improvements on the land that should have placed its value at P 5
million; and that NIA never negotiated with the landowners before
taking their properties for the project, causing permanent and
irreparable damages to their properties valued at P 250,000.
RTC issued an order forming a committee tasked to determine
the fair market value of the expropriated properties to establish the
just compensation to be paid to the owners. Lower court issued a Writ
of Possession in favor of NIA. The committee submitted its
computation adding the value of the earth fill excavated from
portions of Lot Nos. 3039 and 3080. Petitioner objected to the
inclusion of the value of the excavated soil in the computation of the
value of the land. RTC adopted the findings of the committee despite
the objections of NIA to the inclusion of the value of the excavated
soil in the computation of the value of the land.
39

NIA, through the Office of the Solicitor General, appealed to the


Court of Appeals (CA) which affirmed with modification the RTCs
decision. CA deleted the value of the soil in determination of
compensation but affirmed RTCs valuation of the improvements
made on the properties.
ISSUE:
Whether or not the value of the excavated soil should be
included in the computation of just compensation.
RULING:
There is no legal basis to separate the value of the excavated soil
from that of the expropriated properties, contrary to what the trial
court did. In the context of expropriation proceedings, the soil has no
value separate from that of the expropriated land. Just compensation
ordinarily refers to the value of the land to compensate for what the
owner actually loses. Such value could only be that which prevailed at
the time of the taking.
In National Power Corporation v. Ibrahim, et al. The SC held
that rights over lands are indivisible. This conclusion is drawn from
Article 437 of the Civil Code which provides: The owner of a parcel of
land is the owner of its surface and of everything under it, and he can
construct thereon any works or make any plantations and excavations
which he may deem proper, without detriment to servitudes and
subject to special laws and ordinances. He cannot complain of the
reasonable requirements of aerial navigation. Thus, the ownership of
land extends to the surface as well as to the subsoil under it.
Hence, the CA correctly modified the trial courts Decision when it
ruled it is preposterous that NIA will be made to pay not only for the
value of the land but also for the soil excavated from such land when
such excavation is a necessary phase in the building of irrigation
projects. As pointed out by the OSG, the law does not limit the use of
the expropriated land to the surface area only. To sanction the
payment of the excavated soil is to allow the landowners to recover
more than the value of the land at the time when it was taken, which
is the true measure of the damages, or just compensation, and would
discourage the construction of important public improvements.

40

G.R. No. 146259

September 13, 2007

FLORENTINO, TROADIO and PEDRO, all surnamed OCHOA,


petitioners, vs. MAURO APETA and APOLONIA ALMAZAN,
respondents.
FACTS:
Ochoa and their predecessors-in-interest have been occupying
Lot No. 1580 consisting of 886 square meters situated in Malaban,
Bian, Laguna since 1910. The lot is covered by a TCT No. T-40624 of
the Registry of Deeds of that province. They built their houses and
apartment building thereon.
Sometime in May 10, 1982, Mauro Apeta and Apolonia Almazan
found that they are the true owners of Lot No. 1580 being occupied by
Ochoas. Apeta filed with the RTC a complaint for recovery of
possession and damages against Ochoa. Respondents alleged in that
they are the lawful owners of Lot No. 1580 covered by Certificate of
Title No. RT-599 (10731) issued by the Registry of Deeds of Laguna.
During the proceedings before the RTC, upon agreement of the
parties, the trial judge commissioned Engr. Romulo Unciano of the
Bureau of Lands of Region IV to conduct a resurvey of the disputed
property. The result of the resurvey shows that Lot No. 1580,
occupied by Ochoas, was registered in the name of Margarita Almada,
Almazans predecessor-in-interest; and that the lot covered by TCT
No. T-40624 is not Lot No. 1580, but Lot No. 1581 registered in the
name of Servillano Ochoa, Ochoas predecessor-in-interest. This lot
has been occupied by Isidro Jasmin.
Trial court rendered a decision in favor of respondents. CA affirmed
the judgment of the RTC. Ochoa filed a Petition for Review on
Certiorari.
ISSUE:
Whether or not petitioners were builders in good faith and be
allowed to take possession of their improvements.
RULING:
Good faith is an intangible and abstract quality with no
technical meaning or statutory definition, and it encompasses, among
other things, an honest belief, the absence of malice and the absence
of design to defraud or to seek an unconscionable advantage. It
41

implies honesty of intention,


circumstances which ought to
essence of good faith lies in an
right, ignorance of a superior
overreach another.

and freedom from knowledge of


put the holder upon inquiry. The
honest belief in the validity of ones
claim and absence of intention to

Applied to possession, one is considered in good faith if he is


not aware that there exists in his title or mode of acquisition any flaw
which invalidates it.
SC is convinced that Ochoa and their predecessors-in-interest
were in good faith when they built their houses and apartment
building on Lot No. 1580 since they were convinced it was covered by
their TCT No. T-40624.
SC applied art. 448, 546, and 548 of the Civil Code. SC held that
the landowner can make a choice - either by appropriating the
building by paying the proper indemnity or obliging the builder to
pay the price of the land. The choice belongs to the owner of the land,
a rule that accords with the principle of accession that the accessory
follows the principal and not the other way around. He must choose
only one.

42

G.R. No. 177703

January 28, 2008

VILMA G. ARRIOLA and ANTHONY RONALD G. ARRIOLA,


petitioners, vs. JOHN NABOR C. ARRIOLA, respondent.
FACTS:
John Nabor C. Arriola filed Special Civil Action with the RTC of
Las Pias City against Vilma G. Arriola and Anthony Ronald G.
Arriola for judicial partition of the properties of decedent Fidel
Arriola. John is the son of decedent Fidel with his first wife Victoria
C. Calabia, while petitioner Anthony is the son of decedent Fidel with
his second wife, petitioner Vilma.
RTC rendered a decision ordering the partition of the parcel of
land left by the decedent Fidel S. Arriola by and among his heirs in
equal shares of one-third (1/3) each without prejudice to the rights of
creditors or mortgagees thereon.
The parties failed to agree on how to partition among them the
land covered by subject land, John sought its sale through public
auction, and petitioners acceded to it. Accordingly, the RTC ordered
the public auction of the subject land. The public auction sale was
scheduled on May 31, 2003 but it had to be reset when petitioners
refused to include in the auction the house (subject house) standing
on the subject land. This prompted respondent to file with the RTC an
Urgent Manifestation and Motion for Contempt of Court, praying that
petitioners be declared in contempt. RTC denied the motion.
John filed with the CA a Petition for Certiorari where he sought
to have the RTC Orders set aside, and prayed that he be allowed to
proceed with the auction of the subject land including the subject
house. CA granted the petition. Hence, petitioners filed a Petition for
Review on Certiorari before the SC.
ISSUE:
Whether or not the public auction should include the subject
house.
RULING:
Yes. SC agrees with the CA that the subject house is covered by
the judgment of partition for reasons that as the deceased owned the
subject land, he also owned the subject house which is a mere
accessory to the land.
43

The right to accession is automatic (ipso jure), requiring no


prior act on the part of the owner or the principal. So that even if the
improvements including the house were not alleged in the complaint
for partition, they are deemed included in the lot on which they stand,
following the principle of accession. Consequently, the lot subject of
judicial partition in this case includes the house which is permanently
attached thereto; otherwise, it would be absurd to divide the
principal, i.e., the lot, without dividing the house which is
permanently attached thereto. While SC treats the subject house as
part of the co-ownership of the parties, it stops short of authorizing
its actual partition by public auction.
Following restrictions on its partition: first, that the heirs
cannot extra-judicially partition it for a period of 10 years from the
death of one or both spouses or of the unmarried head of the family,
or for a longer period, if there is still a minor beneficiary residing
therein; and second, that the heirs cannot judicially partition it
during the aforesaid periods unless the court finds compelling
reasons therefor. No compelling reason has been alleged by the
parties; nor has the RTC found any compelling reason to order the
partition of the family home, either by physical segregation or
assignment to any of the heirs or through auction sale as suggested by
the parties.

44

G.R. No. 170923

January 20, 2009

SULO SA NAYON, INC. and/or PHILIPPINE VILLAGE HOTEL, INC.


and JOSE MARCEL E. PANLILIO,Petitioners, vs. NAYONG
PILIPINO FOUNDATION, Respondent.
FACTS:
In 1975, respondent leased a portion of the Nayong Pilipino
Complex, to petitioner Sulo sa Nayon, Inc. for the construction and
operation of a hotel building, to be known as the Philippine Village
Hotel. The lease was for an initial period of 21 years, or until May
1996. It is renewable for a period of 25 years under the same terms
and conditions upon due notice in writing to respondent of the
intention to renew.
In 1995, petitioners sent respondent a letter notifying the latter
of their intention to renew the contract. July of the same year, parties
agreed to the renewal of the contract for another 25 years, or until
2021. Under the new agreement, petitioner PVHI was bound to pay
the monthly rentals.
Beginning January 2001, petitioners defaulted in the payment
of their monthly rental. Respondent repeatedly demanded petitioners
to pay the arrears and vacate the premises. MeTC rendered its
decision in favor of respondent. RTC modified the ruling of the MeTC.
RTC ordered Nayong Pilipino to submit a written manifestation of the
option or choice it selected, i.e., to appropriate the improvements
upon payment of proper indemnity or compulsory sale of the land
whereon the hotel building of PVHI and related improvements or
facilities were erected.
CA held that the RTC erroneously applied the rules on
accession, as found in Articles 448 and 546 of the Civil Code. CA held
that, By and large, respondents are admittedly mere lessees of the
subject premises and as such, cannot validly claim that they are
builders in good faith in order to solicit the application of Articles 448
and 546 of the Civil Code in their favor.
ISSUE:
Whether or not Sulo sa Nayon as builders have acted in good
faith in order for Art. 448 in relation to Art.546 of the Civil Code may
apply with respect to their rights over improvements.

45

RULING:
Article 448 is manifestly intended to apply only to a case where
one builds, plants, or sows on land in which he believes himself to
have a claim of title and not to lands where the only interest of the
builder, planter or sower is that of a holder, such as a tenant. In the
case at bar, petitioners have no adverse claim or title to the land. In
fact, as lessees, they recognize that the respondent is the owner of the
land. What petitioners insist is that because of the improvements,
which are of substantial value that they have introduced on the leased
premises with the permission of respondent they should be
considered builders in good faith who have the right to retain
possession of the property until reimbursement by respondent.
SC affirms the ruling of the CA that introduction of valuable
improvements on the leased premises does not give the petitioners
the right of retention and reimbursement which rightfully belongs to
a builder in good faith. Otherwise, such a situation would allow the
lessee to easily "improve" the lessor out of its property. We reiterate
the doctrine that a lessee is neither a builder in good faith nor in bad
faith that would call for the application of Articles 448and 546 of the
Civil Code. His rights are governed by Article 1678 of the Civil Code.

46

G.R. No. 157044 October 5, 2005


RODOLFO V. ROSALES, (represented by his heirs, Rodolfo, Jr.,
Romeo Allan, Lillian Rhodora, Roy Victor, Roger Lyle and Alexander
Nicolai, all surnamed Rosales) and LILY ROSQUETA-ROSALES,
Petitioners vs. MIGUEL CASTELLTORT, JUDITH CASTELLTORT,
and LINA LOPEZ-VILLEGAS, assisted by her Attorney-in-Fact, Rene
Villegas, Respondents.
FACTS:
Spouses Rodolfo V. Rosales and Lily Rosqueta-Rosales are the
registered owners of a parcel of land designated as Lot 17, Block 1 of
Subdivision Plan LRC Psd-55244 situated in Los Baos, Laguna.
On August 16, 1995, petitioners discovered that a house was
being constructed on their lot, without their knowledge and consent,
by respondent Miguel Castelltort. It turned out that respondents
Castelltort and his wife Judith had purchased a lot, Lot 16 of the same
Subdivision Plan, from respondent Lina Lopez-Villegas (Lina)
through her son-attorney-in-fact Rene Villegas (Villegas) but that
after a survey thereof by geodetic engineer Augusto Rivera, he
pointed to Lot 17 as the Lot 16 the Castelltorts purchased.
Negotiations began, with Villegas offering a larger lot near
petitioners lot in the same subdivision as a replacement thereof. In
the alternative, Villegas proposed to pay the purchase price of
petitioners lot with legal interest. Both proposals were, however,
rejected by petitioners and directed Castelltort to stop the
construction of and demolish his house and any other structure he
may have built thereon, and desist from entering the lot.
Petitioners subsequently filed a complaint for recovery of
possession and damages with prayer for the issuance of a restraining
order and preliminary injunction against spouses-respondents Miguel
and Judith Castelltort before the RTC of Calamba, Laguna. RTC held
that there is no well-founded belief of ownership by the defendants of
the land upon which they built their house. That they acted in bad
faith and equally guilty of negligence which led to the construction of
the defendants house on plaintiffs property and therefore jointly and
severally liable for all the damages suffered by the plaintiffs.
CA reversed the decision of the trial court. CA held that RTC
ignored the pivotal issue to be resolved in this case, i.e. whether
appellant Miguel is a builder in good faith. Miguel relied on the title
which the intervenor showed to him which, significantly, has no
47

annotation that would otherwise show a prior adverse claim. Thus, as


far as Miguel is concerned, his title over the subject lot, as well as the
title of the intervenor thereto, is clean and untainted by an adverse
claim or other irregularities. Hence, the petition.
ISSUE:
Whether or not the respondent acted in good faith and who has
the right of option under Art. 448?
HELD:
Under the foregoing provision (Art 448), the landowner can
choose between appropriating the building by paying the proper
indemnity or obliging the builder to pay the price of the land, unless
its value is considerably more than that of the structures, in which
case the builder in good faith shall pay reasonable rent. If the parties
cannot come to terms over the conditions of the lease, the court must
fix the terms thereof.
The choice belongs to the owner of the land, a rule that accords
with the principle of accession, i.e., that the accessory follows the
principal and not the other way around. Even as the option lies with
the landowner, the grant to him, nevertheless, is preclusive. The
landowner cannot refuse to exercise either option and compel instead
the owner of the building to remove it from the land.
In the case at bar, Castelltorts good faith ceased on August 21,
1995 when petitioners personally apprised him of their title over the
questioned lot. As held by the CA, should petitioners then opt to
appropriate the house, they should only be made to pay for that part
of the improvement built by Castelltort on the questioned property at
the time good faith still existed on his part or until August 21, 1995.
The commencement of Castelltorts payment of reasonable rent
should start on August 21, 1995 as well, to be paid until such time that
the possession of the property is delivered to petitioners, subject to
the reimbursement of expenses, that is, if such option is for
petitioners to appropriate the house.

48

G.R. No. 175399

October 27, 2009

OPHELIA L. TUATIS, Petitioner, vs. SPOUSES ELISEO ESCOL and


VISMINDA ESCOL; HONORABLE COURT OF APPEALS, 22nd
DIVISION, CAGAYAN DE ORO CITY; REGIONAL TRIAL COURT,
BRANCH 11, SINDANGAN, ZAMBOANGA DEL NORTE; and THE
SHERIFF OF RTC, BRANCH 11, SINDANGAN, ZAMBOANGA DEL
NORTE, Respondents.
FACTS:
Sometime in November 1989, Visminda, as seller, and Tuatis,
as buyer, entered into a Deed of Sale of a Part of a Registered Land by
Installment. The subject matter of said Deed was a piece of real
property situated in Poblacion, Sindangan, Zamboanga del Norte
being known as Lot No. 251.
Tuatis claimed that she had paid the entire purchase price of
P10,000.00 and took possession of the subject property and
constructed a residential building thereon. In 1996, Tuatis requested
Visminda to sign a prepared absolute deed of sale covering the subject
property, but the latter refused, contending that the purchase price
had not yet been fully paid.
The parties tried to amicably settle the case before the Lupon
Barangay, to no avail. Thus, Tuatis filed a Complaint for Specific
Performance with Damages before the RTC. In her answer, Visminda
countered that, except for the P3,000.00 downpayment and
P1,000.00 installment paid by Tuatis on 19 December 1989 and 17
February 1990, respectively, Tuatis made no other payment to
Visminda. Despite repeated verbal demands, Tuatis failed to comply
with the conditions that she and Visminda agreed upon in the Deed of
Sale by Installment for the payment of the balance of the purchase
price for the subject property.
The RTC rendered a decision in favor of Visminda and likewise
granted a motion for issuance of a Writ of Execution. Tuatis moved
that the RTC issue an order allowing her to buy the subject property
from Visminda. While Tuatis indeed had the obligation to pay the
price of the subject property, she opined that such should not be
imposed if the value of the said property was considerably more than
the value of the building constructed thereon by Tuatis. Tuatis alleged
that the value of the building she constructed was considerably more
than that of the subject property. Tuatis maintained that she then had
the right to choose between being indemnified for the value of her
residential building or buying from Visminda the parcel of land
49

subject of the case. Tuatis stated that she was opting to exercise the
second option.
When the Sheriff enforced the Writ of Execution, Tuatis
immediately filed with the CA a petition for the annulment of the RTC
Order. Hence, Tuatis filed a petition to the SC principally arguing that
Article 448 of the Civil Code must be applied to the situation between
her and Visminda.
ISSUE:
Whether or not Art. 448 of the Civil Code must be applied to
this case.
RULING:
Taking into consideration the provisions of the Deed of Sale by
Installment and Article 448 of the Civil Code, Visminda has the
following options:
Under the first option, Visminda may appropriate for herself
the building on the subject property after indemnifying Tuatis for the
necessary and useful expenses the latter incurred for said building, as
provided in Article 546 of the Civil Code. Until Visminda
appropriately indemnifies Tuatis for the building constructed by the
latter, Tuatis may retain possession of the building and the subject
property.
Under the second option, Visminda may choose not to
appropriate the building and, instead, oblige Tuatis to pay the present
or current fair value of the land. The P10,000.00 price of the subject
property, as stated in the Deed of Sale on Installment executed in
November 1989, shall no longer apply, since Visminda will be
obliging Tuatis to pay for the price of the land in the exercise of
Vismindas rights under Article 448 of the Civil Code, and not under
the said Deed. Tuatis obligation will then be statutory, and not
contractual, arising only when Visminda has chosen her option under
Article 448 of the Civil Code.
If the present or current value of the land, the subject property
herein, turns out to be considerably more than that of the building
built thereon, Tuatis cannot be obliged to pay for the subject
property, but she must pay Visminda reasonable rent for the same.
Visminda and Tuatis must agree on the terms of the lease; otherwise,
the court will fix the terms.
50

The Court highlights that the options under Article 448 are available
to Visminda, as the owner of the subject property. There is no basis
for Tuatis demand that, since the value of the building she
constructed is considerably higher than the subject property, she may
choose between buying the subject property from Visminda and
selling the building to Visminda for P502,073.00. Again, the choice of
options is for Visminda, not Tuatis, to make. And, depending on
Vismindas choice, Tuatis rights as a builder under Article 448 are
limited to the following: (a) under the first option, a right to retain the
building and subject property until Visminda pays proper indemnity;
and (b) under the second option, a right not to be obliged to pay for
the price of the subject property, if it is considerably higher than the
value of the building, in which case, she can only be obliged to pay
reasonable rent for the same.

51

G.R. No. 151815

February 23, 2005

SPOUSES JUAN NUGUID AND ERLINDA T. NUGUID, petitioners,


vs. HON. COURT OF APPEALS AND PEDRO P. PECSON,
respondents.
FACTS:
Pedro P. Pecson owned a commercial lot located at 27 Kamias
Road, Quezon City, on which he built a four-door two-storey
apartment building. For failure to pay realty taxes, the lot was sold at
public auction by the City Treasurer of Quezon City to Mamerto
Nepomuceno, who in turn sold it for P103,000 to the spouses Juan
and Erlinda Nuguid.
Pecson challenged the validity of the auction sale before the
RTC. RTC upheld the spouses title but declared that the four-door
two-storey apartment building was not included in the auction sale.
This was affirmed by the Court of Appeals and thereafter by the
Supreme Court. As a result, the Nuguid spouses moved for delivery of
possession of the lot and the apartment building.
The trial court ruled that the Spouses Nuguid were to reimburse
Pecson for his construction cost of P53,000, following which, the
spouses Nuguid were entitled to immediate issuance of a writ of
possession over the lot and improvements. RTC also directed Pecson
to pay the same amount of monthly rentals to the Nuguids as paid by
the tenants occupying the apartment units or P21,000 per month and
allowed the offset of the amount of P53,000 due from the Nuguids
against the amount of rents collected by Pecson from the tenants of
the apartment.
Pecson duly moved for reconsideration but the RTC issued a
Writ of Possession directing the deputy sheriff to put the spouses
Nuguid in possession of the subject property with all the
improvements thereon and to eject all the occupants therein. Pecson
then filed a special civil action for certiorari and prohibition with the
Court of Appeals. The CA affirmed the order of payment of
construction costs but rendered the issue of possession moot on
appeal.
Pecson filed a petition for review before the Supreme Court. SC
remanded the case to the trial court for it to determine the current
market value of the apartment building on the lot. The determined
value shall then be paid by the Sps. Nuguid to Pecson otherwise the
52

latter shall be restored to the possession of the apartment building


until payment of the required indemnity.
The parties have arrived at a compromise agreement that the
value of the said improvement/building is P400,000. The plaintiff
has already received P300,000.00. However, when Sps. Nuguid was
ready to pay the balance of P100,000.00, the Pecson now insists that
there should be a rental to be paid by Sps. Nuguid. The trial court
scheduled for another hearing but directed Nuguid to pay Pecson the
balance of P100,000.
After conducting a hearing, the lower court issued an order
directing the spouses to pay the sum of P1,344,000 as reimbursement
of the unrealized income of Pecson for the period beginning
November 22, 1993 up to December 1997. The sum was based on the
computation of P28,000/month rentals of the four-door apartment.
The Nuguid spouses filed a motion for reconsideration but this
was denied for lack of merit. The Nuguid couple then appealed the
trial courts ruling to the Court of Appeals.
The CA reduced the rentals from P1,344,000 to P280,000 in
favor of the Nuguid couple. The said amount represents accrued
rentals from the determination of the current market value on
January 31, 1997 until its full payment on December 12, 1997. Hence,
Sps. Nuguid filed a petition for review on certiorari with the SC.
Sps. Nuguid argued that after reaching an agreed price of
P400,000 for the improvements, they only made a partial payment of
P300,000. Thus, they contend that their failure to pay the full price
for the improvements will, at most, entitle Pecson to be restored to
possession, but not to collect any rentals.
Pecson, on the other hand, points out that all he asked was that
the value of the fruits of the property during the period he was
dispossessed be accounted for since the prior decision of SC
recognized that he was entitled to the property. In other words, he
was entitled to rental income from the property.

53

ISSUE:
WON the Sps. Nuguid should reimburse Pecson for the benefits
derived from the apartment building.
RULING:
YES. Under Article 448, the landowner is given the option,
either to appropriate the improvement as his own upon payment of
the proper amount of indemnity or to sell the land to the possessor in
good faith. Relatedly, Article 546 provides that a builder in good faith
is entitled to full reimbursement for all the necessary and useful
expenses incurred; it also gives him right of retention until full
reimbursement is made.
The right of retention is considered as one of the measures
devised by the law for the protection of builders in good faith. Its
object is to guarantee full and prompt reimbursement as it permits
the actual possessor to remain in possession while he has not been
reimbursed (by the person who defeated him in the case for
possession of the property) for those necessary expenses and useful
improvements made by him on the thing possessed.
The right of retention, which entitles the builder in good faith to
the possession as well as the income derived therefrom, is already
provided for under Article 546 of the Civil Code.
Sps. Nuguid committed a violation of Pecson's right of retention
when they insisted on dispossessing him by filing for a Writ of
Possession to cover both the lot and the building despite his right of
ownership over the apartment building. They also took advantage of
the situation to benefit from the four-unit apartment building by
collecting rentals thereon, before they paid for the cost of the
apartment building. It was only four years later that they finally paid
its full value to the respondent. Pecson is clearly entitled to payment
by virtue of his right of retention over the said improvement.
Given the circumstances of the instant case where the builder in
good faith has been clearly denied his right of retention for almost
half a decade, SC find that the increased award of rentals by the RTC
was reasonable and equitable. The Nuguid couple had reaped all the
benefits from the improvement introduced by Pecson during said
period, without paying any amount to the latter as reimbursement for
his construction costs and expenses. They should account and pay for
such benefits.
G.R. No. 140798

September 19, 2006


54

MARCELITO D. QUEVADA, petitioner, vs. COURT OF APPEALS and


JUANITO N. VILLAVERDE, respondents.
FACTS:
Juanito is the lessor of a parcel of land with a residential house
in Sampaloc, Manila. Sometime in 1994, he (as a lessor) and Quevada
entered into a Contract of Lease of a portion of the residential house
(consisting of 96 square meters) which is located on the subject
property for the period from August 15, 1994 to August 15, 1995, at a
monthly rental of P2,500.
After expiration of the lease, they entered into another Contract
of Lease, which was an extension of the previous date. After the
expiration of the extended Lease, Marcelito continued possessing the
premises, but without payment of any reasonable compensation.
Juanito made several demands to the petitioner to vacate the
premises but was refused; as a matter of fact, the petitioner refused to
vacate without justifiable reason. Because of the Marcelitos refusal to
vacate the premises, Juanito referred the matter to the barangay
court for conciliation, only for the former to repudiate the "agreement
to vacate as of December 31, 1997."
A notice to vacate the leased property was served upon
Quevada. Quevada answered that he started building the house on the
lot which was finished in 1986 at which time he occupied the house as
his residence.
Juanito in turn "advised" him that he would go ahead and buy
the lot but with an assurance that as soon as Marcelito would be in a
financial position to do payment, the former will transfer the title to
the latter. Thus, a Lease Contract, in the meantime, was executed, for
him to pay the rentals at P2,500 a month, but only with respect to the
land, since the house belonged to him. Juanito did not give him a
chance to pay the purchase price by setting a deadline to do the
payment; similarly, he respondent refused to accept the monthly
rental of the lot for P2,500.
Metropolitan Trial Court (MeTC) ruled in favor of Juanito. RTC
and CA affirmed the decision of MeTC.
ISSUE:
55

Whether or not petitioner ought to be reimbursed for the value


of the house standing on the lot.
RULING:
YES. Marcelito should be paid for the value of the portion of the
house covered by the lease, to be offset against rentals due.
Petitioner is not the owner of or claiming title to the land, but a
mere tenant occupying only a portion of the house on it under the
lease contract between him and private respondent. No supporting
evidence was presented showing that petitioner's construction of the
house was with the consent of the land's previous owner, but good
faith should be presumed, particularly since the lease relationship
was open and in plain view.
Neither is there a showing of bad faith in petitioner's refusal to
vacate the land. On the other hand, there is no indication that private
respondent will oblige petitioner to pay the price of the land. In fact,
the former refuses to sell it to the latter. As a mere tenant, however,
petitioner must pay reasonable rent for the continued use and
occupancy of the leased premises from the time the lease contract
expired until he finally vacates and surrenders it to private
respondent.
It would not be fair for private respondent to receive both the
rent and the portion of the house covered by the lease. As to its
valuation, there is only petitioner's assertion that it cost P500,000. In
order to satisfy the demands of substantial justice, morality,
conscience, and fair dealing, and pursuant to equity and the principle
proscribing unjust enrichment, the value of the portion of the house
covered by the lease should be determined so that compensation of its
value against the rentals due can take effect.

G.R. No. 185020

October 6, 2010
56

FILOMENA
R.
BENEDICTO,
VILLAFLORES, Respondent.

Petitioner,

vs.

ANTONIO

FACTS:
In 1980, Antonio Villaflores bought a portion of a property
owned by her aunt Maria Villaflores and the thereafter took
possession of said portion and thereon, constructed a house. By 1992,
the remaining portion of Marias property was sold to Antonio. Both
the 1980 and the 1992 sale was covered by a Deed of Absolute Sale
but there was failure to register the same with Registry of Deeds
(ROD).
By 1994, Maria resold said property previously sold to Antonio
to Filomena Benedicto of which Filomena registered with the ROD,
thus properly transferring the title of Maria Villaflores in the name of
the Filomena. Thereafter, Filomena proceeded against Antonio for
Accion Publiciana with Cancellation of Notice of Adverse Claim,
Damages and Attorneys Fees against Antonio. She alleged that she
acquired Lot 2-A in 1994 from her grandaunt Maria by virtue of the
Kasulatan ng Bilihang Tuluyan and she was not aware that Antonio
had any claim or interest over the subject property.
Antonio traversed the complaint, asserting absolute ownership
over Lot 2-A. He alleged that he purchased the subject property from
Maria in 1980; and that he took possession of the same and
constructed his house thereon. He came to know of the sale in favor
of Filomena only in 2000 when the latter demanded that he vacate
the property. He averred that Filomena was aware of the sale; hence,
the subsequent sale in favor of Filomena was rescissible, fraudulent,
fictitious, or simulated.
After trial, the RTC rendered a decision sustaining Filomenas
ownership. According to the RTC, Filomena was the one who
registered the sale in good faith; as such, she has better right than
Antonio. It rejected Antonios allegation of bad faith on the part of
Filomena because no sufficient evidence was adduced to prove it.
The RTC found Antonios evidence of ownership questionable
but declared him a builder in good faith. The CA affirmed the RTC for
upholding Filomenas ownership of Lot 2-A and for declaring Antonio
a builder in good faith. However, it remanded the case to the RTC for
further proceedings to determine the respective rights of the parties
under Articles 448 and 546 of the Civil Code, and the amount due
Antonio. Hence, a petition for review on certiorari before the SC.
57

Filomena asserts that Antonio is not entitled to any reimbursement


because he possessed the property by mere tolerance. Maria merely
allowed Antonio to construct his house on a portion of Lot 2-A;
hence, he is not entitled to any reimbursement or retention.
ISSUE:
Whether or not Antonio is a builder in good faith, thus entitled
to reimbursements for the improvements he introduced to the
property?
RULING:
Antonio is a builder in good faith and, as provided under Article
448, a landowner is given the option to either appropriate the
improvement as his own upon payment of the proper amount of
indemnity, or sell the land to the possessor in good faith. Relatedly,
Article 546 provides that a builder in good faith is entitled to full
reimbursement for all the necessary and useful expenses incurred; it
also gives him right of retention until full reimbursement is made.
The objective of Article 546 of the Civil Code is to administer
justice between the parties involved. The said provision was
formulated in trying to adjust the rights of the owner and possessor in
good faith of a piece of land, to administer complete justice to both of
them in such a way as neither one nor the other may enrich himself of
that which does not belong to him. Guided by this precept, it is
therefore the current market value of the improvements which should
be made the basis of reimbursement. A contrary ruling would
unjustly enrich the private respondents who would otherwise be
allowed to acquire a highly valued income-yielding four-unit
apartment building for a measly amount. Consequently, the parties
should therefore be allowed to adduce evidence on the present market
value of the apartment building upon which the trial court should
base its finding as to the amount of reimbursement to be paid by the
landowner.
Thus, the SC denied Filomenas petition and affirmed the CA
ruling.

58

G.R. No. 150666

August 3, 2010

LUCIANO BRIONES and NELLY BRIONES, Petitioners, vs. JOSE


MACABAGDAL, FE D. MACABAGDAL and VERGON REALTY
INVESTMENTS CORPORATION,Respondents.
FACTS:
Respondent-spouses
purchased
from
Vergon
Realty
Investments Corporation (Vergon) Lot No. 2-R, a 325-square-meter
land located in Vergonville Subdivision No. 10 at Las Pias City,
Metro Manila. On the other hand, petitioners are the owners of Lot
No. 2-S, which is adjacent to Lot No. 2-R.
Sometime in 1984, after obtaining the necessary building
permit and the approval of Vergon, petitioners constructed a house
on Lot No. 2-R which they thought was Lot No. 2-S. After being
informed of the mix up by Vergons manager, respondent-spouses
immediately demanded petitioners to demolish the house and vacate
the property. Petitioners, however, refused to heed their demand.
Thus, respondent-spouses filed an action to recover ownership and
possession of the said parcel of land with the RTC of Makati City.
Petitioners insisted that the lot on which they constructed their
house was the lot which was consistently pointed to them as theirs by
Vergons agents. They interposed the defense of being buyers in good
faith and impleaded Vergon as third-party defendant claiming that
because of the warranty against eviction, they were entitled to
indemnity from Vergon in case the suit is decided against them. The
RTC ruled in favor of respondent-spouses and found that petitioners
house was undoubtedly built on Lot No. 2-R and ordered the latter to
immediately vacate the property.
On appeal, the CA affirmed the RTCs finding that the lot upon
which petitioners built their house was not the one which Vergon sold
to them. The CA further ruled that petitioners cannot use the defense
of allegedly being a purchaser in good faith for wrongful occupation
of land.
ISSUE:
Whether or not petitioners were in good faith.

59

RULING:
YES. Article 527 of the Civil Code presumes good faith, and
since no proof exists to show that the mistake was done by petitioners
in bad faith, the latter should be presumed to have built the house in
good faith.
When a person builds in good faith on the land of another,
Article 448 of the Civil Code governs. Said article provides, that, the
owner of the land on which anything has been built, sown or planted
in good faith, shall have the right to appropriate as his own the works,
sowing or planting, after payment of the indemnity provided for in
Articles 546 and 548, or to oblige the one who built or planted to pay
the price of the land, and the one who sowed, the proper rent.
However, the builder or planter cannot be obliged to buy the land if
its value is considerably more than that of the building or trees. In
such case, he shall pay reasonable rent, if the owner of the land does
not choose to appropriate the building or trees after proper
indemnity. The parties shall agree upon the terms of the lease and in
case of disagreement, the court shall fix the terms thereof.
The builder in good faith can compel the landowner to make a
choice between appropriating the building by paying the proper
indemnity or obliging the builder to pay the price of the land. The
choice belongs to the owner of the land, a rule that accords with the
principle of accession, i.e., that the accessory follows the principal and
not the other way around. However, even as the option lies with the
landowner, the grant to him, nevertheless, is preclusive. He must
choose one. He cannot, for instance, compel the owner of the building
to remove the building from the land without first exercising either
option. It is only if the owner chooses to sell his land, and the builder
or planter fails to purchase it where its value is not more than the
value of the improvements, that the owner may remove the
improvements from the land. The owner is entitled to such remotion
only when, after having chosen to sell his land, the other party fails to
pay for the same.
Moreover, petitioners have the right to be indemnified for the
necessary and useful expenses they may have made on the subject
property.

60

G.R. No. 153652

January 16, 2004

ALFREDO YASAY DEL ROSARIO, petitioner, vs. SPS. JOSE E.


MANUEL and CONCORDIA MANUEL, represented by Attorney-infact, PATRICIA ARIOLA,respondents.
FACTS:
On August 12, 1999, spouses Jose and Concordia Manuel,
respondents, filed with the MTC, San Mateo, Rizal a complaint for
unlawful detainer against Alfredo Yasay del Rosario, petitioner. They
alleged that they are the true and lawful owners of a 251 square meter
lot located at Sta. Ana, San Mateo, Rizal. Because of their
compassion, they allowed petitioner, whose house was destroyed by a
strong typhoon, to occupy their lot. They agreed that he could build
thereon a temporary shelter of light materials. But without their
consent, what he constructed was a house of concrete materials.
In 1992, respondents asked petitioner to vacate the lot. This
was followed by repeated verbal demands but to no avail, prompting
them to bring the matter to the barangay. But the parties failed to
reach an amicable settlement. On June 25, 1999, the barangay
chairman issued a Certification to File Action.
In his answer to the complaint, petitioner claimed that
sometime in 1968, respondents allowed him to build his house on the
lot, provided he would guard the premises to prevent land grabbers
and squatters from occupying the area. In 1995, when respondents
visited this country, they agreed verbally to sell the portion on which
his house was constructed. A year later, he made an offer to buy the
60 square meter portion occupied by him and to spend for its survey.
But what respondents wanted to sell was the whole area containing
251 square meters. He then informed them that he would first
consult his children and they said they will wait. Instead, they filed
the instant complaint.
ISSUE:
WON petitioner is a builder in good faith.
RULING:
NO. Petitioner is not a builder in good faith. Considering that he
occupies the land by mere tolerance, he is aware that his occupation
of the same may be terminated by respondents any time.

61

G.R. No. L-57348 May 16, 1985


FRANCISCO DEPRA, plaintiff-appellee, vs. AGUSTIN DUMLAO,
defendant-appellant.
FACTS:
Francisco Depra is the owner of a parcel of land situated in the
municipality of Dumangas, Iloilo, with an area of approximately
8,870 square meters. Agustin Dumlao, owns an adjoining lot with an
approximate area of 231 sq. ms.
When Dumlao constructed his house on his lot, the kitchen
thereof had encroached on an area of thirty four (34) square meters of
Depras property, After the encroachment was discovered in a
relocation survey of Depras lot made on November 2,1972, his
mother, Beatriz Depra after writing a demand letter asking Dumlao to
move back from his encroachment, filed an action for Unlawful
Detainer. After trial, the Municipal Court found that Dumlao was a
builder in good faith, and applying Article 448 of the Civil Code.
Ordering that a forced lease is created between the parties with
the plaintiffs, as lessors, and the defendants as lessees, over the
disputed portion with an area of thirty four (34) square meters, the
rent to be paid is five (P5.00) pesos a month, payable by the lessee to
the lessors within the first five (5) days of the month the rent is due;
and the lease shall commence on the day that this decision shall have
become final.
Neither party appealed but Depra did not accept payment of
rentals so that Dumlao deposited such rentals with the Municipal
Court. Depra filed a Complaint for Quieting of Title against Dumlao
before the then Court of First Instance of Iloilo involving the very
same 34 square meters, which was the bone of contention in the
Municipal Court. Dumlao, in his Answer, admitted the encroachment
but alleged, in the main, that the present suit is barred by res judicata
by virtue of the Decision of the Municipal Court, which had become
final and executory.
Depra claims that the Decision of the Municipal Court was null
and void ab initio because its jurisdiction is limited to the sole issue of
possession, whereas decisions affecting lease, which is an
encumbrance on real property, may only be rendered by Courts of
First Instance.

62

ISSUE:
Whether or not the factual situations of DUMLAO and DEPRA
conform to the juridical positions respectively defined by law, for a
"builder in good faith" under Article 448, a "possessor in good faith"
under Article 526 and a "landowner in good faith' under Article 448.
RULING:
ART. 448. The owner of the land on which anything has been
built sown or planted in good faith, shall have the right to appropriate
as his own the works, sowing or planting, after payment of the
indemnity provided for in articles 546 and 548, or to oblige the one
who built or planted to pay the price of the land, and the one who
sowed, the proper rent. However, the builder or planter cannot be
obliged to buy the land if its value is considerably more than that of
the building or trees. In such case, he shall pay reasonable rent, if the
owner of the land does not choose to appropriate the building or trees
after proper indemnity. The parties shall agree upon the terms of the
lease and in case of disagreement, the court shall fix the terms
thereof.
In the case at bar, Depra has the option either to pay for the
encroaching part of Dumlao's kitchen, or to sell the encroached 34
square meters of his lot to Dumlao. He cannot refuse to pay for the
encroaching part of the building, and to sell the encroached part of
his land, as he had manifested before the Municipal Court. But that
manifestation is not binding because it was made in a void
proceeding.
However, the good faith of Dumlao is part of the Stipulation of
Facts in the Court of First Instance. It was thus error for the Trial
Court to have ruled that Depra is "entitled to possession," without
more, of the disputed portion implying thereby that he is entitled to
have the kitchen removed. He is entitled to such removal only when,
after having chosen to sell his encroached land, Dumlao fails to pay
for the same. In this case, Dumlao had expressed his willingness to
pay for the land, but Depra refused to sell.

63

G.R. No. 160453

November 12, 2012

REPUBLIC OF THE PHILIPPINES, Petitioner, vs. ARCADIO IVAN


A. SANTOS III, and ARCADIO C. SANTOS, JR., Respondents.
FACTS:
Alleging continuous and adverse possession of more than ten
years, Arcadio Ivan A. Santos III, with Arcadio C. Santos, Jr as coapplicant, applied for the registration of property in the RTC. The
property, was bounded in the Northeast by Lot belonging to Arcadio
Jr., in the Southeast by the Paraaque River, in the Southwest by an
abandoned road, and in the Northwest by Lot owned by Arcadio Ivan.
He alleged that the property had been formed through accretion
and had been in their joint open, notorious, public, continuous and
adverse possession for more than 30 years.
The City of Paraaque (the City) opposed the application for
land registration, stating that it needed the property for its flood
control program; that the property was within the legal easement of
20 meters from the river bank; and that assuming that the property
was not covered by the legal easement, title to the property could not
be registered in favor of the applicants for the reason that the
property was an orchard that had dried up and had not resulted from
accretion.
The RTC granted the application for land registration. On
appeal, CA affirmed the RTC.
ISSUE:
Whether or not Article 457 of the Civil Code was applicable
herein.
RULING:
SC declared the disputed lot as exclusively belonging to the
State for being part of the dried-up bed of the Paranaque River.
Article 457 of the Civil Code provides that "to the owners of
lands adjoining the banks of rivers belong the accretion which they
gradually receive from the effects of the currents of the waters." The
application by both lower courts of Article 457 of the Civil Code was
erroneous in the face of the fact that respondents evidence did not
establish accretion, but instead the drying up of the Paraaque River.
64

Applicants for land registration carried the burden of proof to


establish the merits of their application by a preponderance of
evidence, by which is meant such evidence that is of greater weight, or
more convincing than that offered in opposition to it. They would be
held entitled to claim the property as their own and apply for its
registration under the Torrens system only if they established that,
indeed, the property was an accretion to their land.
Accretion is the process whereby the soil is deposited along the
banks of rivers. The deposit of soil, to be considered accretion, must
be: (a) gradual and imperceptible; (b) made through the effects of the
current of the water; and (c) taking place on land adjacent to the
banks of rivers.
The RTC and the CA grossly erred in treating the dried-up river
bed as an accretion that became respondents property pursuant to
Article 457 of the Civil Code. That land was definitely not an
accretion. The process of drying up of a river to form dry land
involved the recession of the water level from the river banks, and the
dried-up land did not equate to accretion.
The State exclusively owned the disputed Lot and may not be
divested of its right of ownership. Article 502 of the Civil Code
expressly declares that rivers and their natural beds are public
dominion of the State. It follows that the river beds that dry up,
continue to belong to the State as its property of public dominion,
unless there is an express law that provides that the dried-up river
beds should belong to some other person.
Since property of public dominion is outside the commerce of
man and not susceptible to private appropriation and acquisitive
prescription, the adverse possession which may be the basis of a grant
of title in the confirmation of an imperfect title refers only to alienable
or disposable portions of the public domain. It is only after the
Government has declared the land to be alienable and disposable
agricultural land that the year of entry, cultivation and exclusive and
adverse possession can be counted for purposes of an imperfect title.

65

G.R. No. 68166 February 12, 1997


HEIRS OF EMILIANO NAVARRO, petitioner, vs. INTERMEDIATE
APPELLATE COURT & HEIRS OF SINFOROSO PASCUAL,
respondents.
FACTS:
Sinforoso Pascual owns a tract of land which lies between
Talisay River on the eastern side and Bulacan River on the western
side. In front of his land, on the northern side, is the disputed land
where there use to lay the Manila Bay. Pascual filed an application to
register and confirm his title to that parcel of land claiming that the
land is an accretion to his property.
Emiliano Navarro, a lessee and in possession of a part of the
subject property, opposed Pascual's application. Navarro claimed that
the land sought to be registered has always been part of the public
domain, it being a part of the foreshore of Manila Bay.
During the pendency of the trial, Emiliano Navarro died and
was substituted by his heirs. Subsequently, Pascual died and was
substituted by his heirs.
On 1975, the court a quo rendered judgment finding the subject
property to be foreshore land and, being a part of the public domain,
it cannot be the subject of land registration proceedings. On appeal,
the CA reversed the findings of the court a quo and granted the
petition for registration of the subject property.
The appellate court explained that the subject land is an
accretion. That the Manila Bay could not have been the cause of the
deposit thereat for in the natural course of things, the waves of the sea
eat the land on the shore. That it is more logical that the two rivers
flowing towards the bay emptied their cargo of sand, silt and clay at
their mouths, thus causing land to accumulate therein.
ISSUE
Whether or not the disputed land is foreshore land.

66

RULING:
SC sided with the conclusion formed by the trial court that the
disputed land is part of the foreshore of Manila Bay and therefore,
part of the public domain.
First, the disputed land used to adjoin, border or front the
Manila Bay and not any of the two rivers whose torrential action is to
account for the accretion on the land. Sulpicio Pascual testified in
open court that the waves of Manila Bay used to hit the disputed land
being part of the bay's foreshore but, after he had planted palapat and
bakawan trees thereon in 1948, the land began to rise. If the accretion
were to be attributed to the action of either or both of the Talisay and
Bulacan Rivers, the alluvium should have been deposited on either or
both of the eastern and western boundaries of Pascual's own tract of
land, not on the northern portion thereof which is adjacent to the
Manila Bay. Clearly lacking, is the third requisite of accretion, which
is, that the alluvium is deposited on the portion of claimant's land
which is adjacent to the river bank.
Accretion as a mode of acquiring property under said Article 457,
requires the concurrence of the following requisites:
1. That the accumulation of soil or sediment be gradual and
imperceptible;
2. That it be the result of the action of the waters of the river; and
3. That the land where the accretion takes place is adjacent to the
bank of the river.
Accretion is the process whereby the soil is deposited, while
alluvium is the soil deposited on the estate fronting the river bank;
the owner of such estate is called the riparian owner. Riparian owners
are, strictly speaking, distinct from littoral owners, the latter being
owners of lands bordering the shore of the sea or lake or other tidal
waters. The alluvium, by mandate of Article 457 of the Civil Code, is
automatically owned by the riparian owner from the moment the soil
deposit can be seen but is not automatically registered property,
hence, subject to acquisition through prescription by third persons.
Second, there is no dispute as to the fact that the disputed land
adjoins the Manila Bay. Manila Bay is obviously not a river. The
disputed property is an accretion on a sea bank, Manila Bay being an
inlet or an arm of the sea.

67

G.R. No. 146616 August 31, 2006


SIAIN ENTERPRISES, INC., Petitioner,vs. F.F. CRUZ & CO., INC.,
Respondent.
FACTS:
On September 18, 1973, Western Visayas Industrial Corporation
(WESVICO) filed a foreshore lease application over the foreshore
land adjacent to Lot 3308, registered in its name located at Loboc,
Lapuz, Lapaz, Iloilo City. The petition for registration was archived by
Court of first instance now RTC because WESVICO could no longer
be contacted. Development Bank of the Philippines (DBP) foreclosed
the lease application where WESVICO ceased to hold its operation.
On July 7, 1983, F.F. Cruz & Co filed with the Bureau of Lands to
lease the application over the foreshore land, which was inspected by
its District Land Officer, that F.F. Cruz occupied as a sanctuary of its
marine equipment used and utilized in the repair and maintenance of
its equipment for loading and unloading in the construction of Iloilo
City Port.
On September 29, 1986, SIAIN Enterprises Inc. purchased from
DBP the properties previously owned by WESVICO including Lot
3309 over to the adjacent foreshore land. Upon knowing that F.F.
Cruz overlapped 130 linear meters, SIAIN filed a protest,
Sangguniang Panglunsod of Iloilo City and Land Management Bureau
favor the mutual interest settling SIAIN to have 70 linear meters
and F.F. Cruz to have 60 linear meters of the disputed area. SIAIN
appealed to DENR, which they ruled that the area is a natural
foreshore, SIAIN is the riparian / littoral owner that should enjoy the
preferential right to lease the entire 130 linear meters adjoining the
property.
ISSUE:
Whether or not the disputed area is reclaimed land or foreshore
land?
Whether or not SIAIN has preferential right to lease the
foreshore area?
RULING:
The Supreme Court held in favor of SIAIN as ruled in par 32 of
Lands Administrative Order No. 7-1 which states that: Preference of
Riparian Owner the owner of the property adjoining foreshore
lands or lands covered with water bordering upon shores or banks of
navigable lakes or rivers, shall be given preference to apply for such
68

lands as may not be needed for public service and in consonance with
Art. 4 Spanish Law of Waters of 1866 which provide that, Lands
added to the shores by accretions and alluvial deposits caused by the
action of the sea form part of the public domain.
In the case at bar, the disputed area is classified as a foreshore
area subject to the preferential right to lease of the littoral owner. The
reason for the preferential right is the same justification for giving
accretions to the riparian owner for the diminutions which his land
suffers by reason of destructive force of the waters. So, he who loses
by the encroachments of the sea should gain by its recession. Such
foreshore area existed even before F.F. Cruz undertook its
reclamation. It was formed by accretions or alluvial deposits due to
the action of the sea, citing the case of Santulan vs. Executive
Secretary. WESVICO does not waived or abandoned its preferential
right to lease the disputed area filed by its application for registration
that was archived due to financial reasons and denied by the trial
court, WESVICO is not barred to file a new foreshore lease
application.
Being part of the public domain, ownership of the area could
not be acquired by WESVICO, but the preferential right remained.
Therefore, the decision of Court of Appeals was reversed and set aside
and reinstated the DENR decision.

69

G.R. No. 116290

December 8, 2000

DIONISIA P. BAGAIPO, petitioner, vs. THE HON. COURT OF


APPEALS and LEONOR LOZANO, respondents.
FACTS:
Petitioner Dionisia P. Bagaipo is the registered owner of Lot No.
415, a 146,900 square meter agricultural land situated in Ma-a, Davao
City. Respondent Leonor Lozano is the owner of a registered parcel of
land located across and opposite the southeast portion of petitioners
lot facing the Davao River. Lozano acquired and occupied her
property in 1962 when his wife inherited the land from her father who
died that year.
Bagaipo filed a complaint for Recovery of Possession with
Mandatory Writ of Preliminary Injunction and Damages against
Lozano for: (1) the surrender of possession by Lozano of a certain
portion of land measuring 29,162 square meters which is supposedly
included in the area belonging to Bagaipo; and (2) the recovery of a
land area measuring 37,901 square meters which Bagaipo allegedly
lost when the Davao River traversed her property. Bagaipo contended
that as a result of a change in course of the said river, her property
became divided into three lots, namely: Lots 415-A, 415-B and 415-C.
Bagaipo also presented Godofredo Corias, a former barangay
captain and long-time resident of Ma-a to prove her claim that the
Davao River had indeed changed its course. Corias testified that the
occurrence was caused by a big flood in 1968 and a bamboo grove
which used to indicate the position of the river was washed away. The
river which flowed previously in front of a chapel located 15 meters
away from the riverbank within Bagaipos property now flowed
behind it. Corias was also present when Magno conducted the
relocation survey in 1988.
For his part, Lozano insisted that the land claimed by Bagaipo is
actually an accretion to their titled property. He asserted that the
Davao River did not change its course and that the reduction in
Bagaipos domain was caused by gradual erosion due to the current of
the Davao River. Lozano added that it is also because of the rivers
natural action that silt slowly deposited and added to his land over a
long period of time. He further averred that this accretion continues
up to the present and that registration proceedings instituted by him
over the alluvial formation could not be concluded precisely because
it continued to increase in size.

70

The trial court conducted an ocular inspection. It concluded


that the applicable law is Article 457 and rendered a decision against
Bagaipo. The CA affirmed the decision of the trial court. Hence, the
appeal.
ISSUE:
Whether or not respondent own Lot 415-C in accordance with
the principle of accretion under Article 457.
RULING:
YES. The trial court and the appellate court both found that the
decrease in land area was brought about by erosion and not a change
in the rivers course. These findings are factual, thus conclusive on the
Court, unless there are strong and exceptional reasons, or they are
unsupported by the evidence on record, or the judgment itself is
based on a misapprehension of facts.
The decrease in petitioners land area and the corresponding
expansion of respondents property were the combined effect of
erosion and accretion respectively. Art. 461 of the Civil Code is
inapplicable. Petitioner cannot claim ownership over the old
abandoned riverbed because the same is inexistent. The riverbeds
former location cannot even be pinpointed with particularity since the
movement of the Davao River took place gradually over an
unspecified period of time, up to the present.
The rule is well-settled that accretion benefits a riparian owner
when the following requisites are present: 1) That the deposit be
gradual and imperceptible; 2) That it resulted from the effects of the
current of the water; and 3) That the land where accretion takes place
is adjacent to the bank of the river. These requisites were sufficiently
proven in favor of respondents. In the absence of evidence that the
change in the course of the river was sudden or that it occurred
through avulsion, the presumption is that the change was gradual and
was caused by alluvium and erosion.
Petitioner did not demonstrate that Lot 415-C allegedly
comprising 29,162 square meters was within the boundaries of her
titled property. The survey plan commissioned by petitioner which
was not approved by the Director of Lands was properly discounted
by the appellate court.

71

G.R. No. 168970

January 15, 2010

CELESTINO BALUS, Petitioner, vs. SATURNINO BALUS and


LEONARDA BALUS VDA. DE CALUNOD, Respondents.
FACTS:
The petitioner and respondents in this case are the children of
Mr. Rufo (the owner of parcel land mortgaged in the bank).
Rufo mortgaged a parcel of land and all the improvement
thereto as a security for a loan he obtained from the bank;
unfortunately Rufo failed to pay his loan. The mortgaged property
was foreclosed and subsequently sold to the bank as the sole bidder at
public auction held for that purpose. The property was not redeemed
within the period allowed by law (two years after the auction) the new
titled issued in favor of the bank. Petitioner and respondents executed
extrajudicial settlement of estate.
Three years after executing the extrajudicial settlement, the
respondents bought the subject property from the bank and the deed
of sale executed under the name of respondents. Meanwhile
petitioner continued possession of the subject lot as a result
respondents filed a complaint for recovery of possession and damages
against petitioner contending that the respondents is the new owner
of the subject property but petitioner still refused to surrender the
possession.
The RTC held that the right of petitioner to purchase from
respondents his share in the disputed property was recognized by the
provision of extrajudicial settlement. Respondents as aggrieved by the
decision of the RTC filed an appeal to the CA.
CA reversed and set aside the decision of the RTC and ordered
petitioner to immediately surrender possession of the subject
property to the respondents. The CA ruled that when petitioner and
respondents did not redeem the subject property within the
redemption period and allowed the consolidation of ownership and
the issuance of a new title in the name of the Bank, their coownership was extinguished.
ISSUE:
Whether or not co- ownership among the petitioner and
respondents over the property continued to exist even after the
transfer of title to the bank.
72

HELD:
The main issue raised by petitioner is whether co-ownership by
him and respondents over the subject property persisted even after
the lot was purchased by the Bank and title thereto transferred to its
name, and even after it was eventually bought back by the
respondents from the Bank.
Petitioner posits that the subject Extrajudicial Settlement is, in
and by itself, a contract between him and respondents, because it
contains a provision whereby the parties agreed to continue their coownership of the subject property by "redeeming" or "repurchasing"
the same from the Bank. This agreement, petitioner contends, is the
law between the parties and, as such, binds the respondents.
SC held that since Rufo lost ownership of the subject property
during his lifetime, it only follows that at the time of his death, the
disputed parcel of land no longer formed part of his estate to which
his heirs may lay claim. Stated differently, petitioner and respondents
never inherited the subject lot from their father.
For petitioner to claim that the Extrajudicial Settlement is an
agreement between him and his siblings to continue what they
thought was their ownership of the subject property, even after the
same had been bought by the Bank, is stretching the interpretation of
the said Extrajudicial Settlement too far.
In the first place, as earlier discussed, there is no co-ownership
to talk about and no property to partition, as the disputed lot never
formed part of the estate of their deceased father.

73

G.R. No. 170829

November 20, 2006

PERLA G. PATRICIO, Petitioner, vs. MARCELINO G. DARIO III and


THE HONORABLE COURT OF APPEALS, Second Division,
Respondents.
FACTS:
On July 5, 1987, Marcelino V. Dario died leaving his wife,
petitioner Perla G. Patricio and their two sons, Marcelino Marc Dario
and private respondent Marcelino G. Dario III as his legal heirs.
Among the properties he left was a parcel of land with a residential
house and a pre-school building built thereon. Petitioner and
Marcelino Marc formally advised private respondent of their
intention to partition the subject property and terminate the coownership. Private respondent refused to partition the property
hence petitioner and Marcelino Marc instituted an action for partition
before the Regional Trial Court of Quezon City. The trial court
ordered the partition of the subject property in the following manner:
Perla G. Patricio, 4/6; Marcelino Marc G. Dario, 1/6; and Marcelino
G. Dario III, 1/6. The trial court also ordered the sale of the property
by public auction wherein all parties concerned may put up their bids.
In case of failure, the subject property should be distributed
accordingly in the aforestated manner.
Private respondent filed a motion for reconsideration which was
denied by the trial court hence he appealed before the Court of
Appeals, which denied the same. However, upon a motion for
reconsideration, the appellate court partially reconsidered. In the
now assailed Resolution, the Court of Appeals dismissed the
complaint for partition filed by petitioner and Marcelino Marc for
lack of merit. It held that the family home should continue despite the
death of one or both spouses as long as there is a minor beneficiary
thereof. The heirs could not partition the property unless the court
found compelling reasons to rule otherwise. The appellate court also
held that the minor son of private respondent, who is a grandson of
spouses Marcelino V. Dario and Perla G. Patricio, was a minor
beneficiary of the family home.
ISSUE:
Whether partition of the family home is proper where one of the
co-owners refuse to accede to such partition on the ground that a
minor beneficiary still resides in the said home.

74

RULING:
The law explicitly provides that occupancy of the family home
either by the owner thereof or by any of its beneficiaries must be
actual.
Article 154 of the Family Code enumerates who are the
beneficiaries of a family home: (1) The husband and wife, or an
unmarried person who is the head of a family; and (2) Their parents,
ascendants, descendants, brothers and sisters, whether the
relationship be legitimate or illegitimate, who are living in the family
home and who depend upon the head of the family for legal support.
To be a beneficiary of the family home, three requisites must concur:
(1) they must be among the relationships enumerated in Art. 154 of
the Family Code; (2) they live in the family home; and (3) they are
dependent for legal support upon the head of the family.
Moreover, Article 159 of the Family Code provides that the
family home shall continue despite the death of one or both spouses
or of the unmarried head of the family for a period of 10 years or for
as long as there is a minor beneficiary, and the heirs cannot partition
the same unless the court finds compelling reasons therefor. This rule
shall apply regardless of whoever owns the property or constituted
the family home.
In the case at bar, private respondents minor son, who is also
the grandchild of deceased Marcelino V. Dario satisfies the first and
second requisites. However, as to the third requisite, Marcelino
Lorenzo R. Dario IV cannot demand support from his paternal
grandmother if he has parents who are capable of supporting him.
There is no showing that private respondent is without means
to support his son; neither is there any evidence to prove that
petitioner, as the paternal grandmother, was willing to voluntarily
provide for her grandsons legal support. On the contrary, herein
petitioner filed for the partition of the property which shows an
intention to dissolve the family home, since there is no more reason
for its existence after the 10-year period ended in 1997. With this
finding, there is no legal impediment to partition the subject
property.

75

G.R. No. 176858

September 15, 2010

HEIRS OF JUANITA PADILLA, represented by CLAUDIO PADILLA,


Petitioners, vs. DOMINADOR MAGDUA, Respondent.
FACTS:
Juanita Padilla (Juanita), the mother of petitioners, owned a
piece of land located in San Roque, Tanauan, Leyte. After Juanitas
death on 23 March 1989, petitioners, as legal heirs of Juanita, sought
to have the land partitioned. Petitioners sent word to their eldest
brother Ricardo Bahia (Ricardo) regarding their plans for the
partition of the land. In a letter dated 5 June 1998 written by Ricardo
addressed to them, petitioners were surprised to find out that Ricardo
had declared the land for himself, prejudicing their rights as co-heirs.
It was then discovered that Juanita had allegedly executed a notarized
Affidavit of Transfer of Real Property (Affidavit) in favor of Ricardo
on 4 June 1966 making him the sole owner of the land. The records
do not show that the land was registered under the Torrens system.
Petitioners filed an action with the RTC of Tacloban City for
recovery of ownership, possession, partition and damages. Petitioners
sought to declare void the sale of the land by Ricardos daughters,
Josephine Bahia and Virginia Bahia-Abas, to respondent Dominador
Magdua (Dominador). The sale was made during the lifetime of
Ricardo. Petitioners alleged that Ricardo, through misrepresentation,
had the land transferred in his name without the consent and
knowledge of his co-heirs.
RTC dismissed the case for lack of jurisdiction. Petitioners filed
a motion; RTC denied the motion for reconsideration and dismissed
the case on the ground of prescription.
ISSUE:
Whether or not the present action is already barred by
prescription.
RULING:
Ricardo and petitioners are co-heirs or co-owners of the land.
Co-heirs or co-owners cannot acquire by acquisitive prescription the
share of the other co-heirs or co-owners absent a clear repudiation of
the co-ownership, as expressed in Article 494 of the Civil Code which
states:

76

Art. 494. No prescription shall run in favor of a co-owner or coheir against his co-owners or co-heirs as long as he expressly or
impliedly recognizes the co-ownership.
Since possession of co-owners is like that of a trustee, in order
that a co-owners possession may be deemed adverse to the cestui que
trust or other co-owners, the following requisites must concur: (1)
that he has performed unequivocal acts of repudiation amounting to
an ouster of the cestui que trust or other co-owners, (2) that such
positive acts of repudiation have been made known to the cestui que
trust or other co-owners, and (3) that the evidence thereon must be
clear and convincing.
In the present case, all three requisites have been met. After
Juanitas death in 1989, petitioners sought for the partition of their
mothers land. The heirs, including Ricardo, were notified about the
plan. Ricardo, through a letter dated 5 June 1998, notified
petitioners, as his co-heirs, that he adjudicated the land solely for
himself. Accordingly, Ricardos interest in the land had now become
adverse to the claim of his co-heirs after repudiating their claim of
entitlement to the land. In order that title may prescribe in favor of
one of the co-owners, it must be clearly shown that he had repudiated
the claims of the others, and that they were apprised of his claim of
adverse and exclusive ownership, before the prescriptive period
begins to run.
However, in the present case, the prescriptive period began to
run only from 5 June 1998, the date petitioners received notice of
Ricardos repudiation of their claims to the land. Since petitioners
filed an action for recovery of ownership and possession, partition
and damages with the RTC on 26 October 2001, only a mere three
years had lapsed. This three-year period falls short of the 10-year or
30-year acquisitive prescription period required by law in order to be
entitled to claim legal ownership over the land. Thus, Dominador
cannot invoke acquisitive prescription.

77

G.R. No. 122047

October 12, 2000

SPOUSES SERAFIN SI AND ANITA BONODE SI, petitioners, vs.


COURT OF APPEALS, SPOUSES JOSE ARMADA and REMEDIOS
ALMANZOR (deceased, and substituted by heirs: Cynthia Armada,
Danilo Armada and Vicente Armada) respondents.
FACTS:
The 340 square meters of land, situated in San Jose District,
Pasay City, the property in dispute, originally belonged to Escolastica,
wife of Severo Armada, Sr. During the lifetime of the spouses, the
property was transferred to their children and the Registry of Deeds,
Pasay City, issued TCT No. 16007 in the names of the three sons, as
follows: DR. CRISOSTOMO R. ARMADA, married to Cresenciana V.
Alejo, 113.34 Square Meters; JOSE R. ARMADA, married to
Remedios Almanzor, 113.33 Square Meters; and DR. SEVERO R.
ARMADA, Jr., single.
Annotated also in the title is the total cancellation of said title
by virtue of the Deed of Sale conveying 113.34 square meters of the
property herein, in favor of ANITA BONODE SI, married to Serafin
D. Si, for the sum of P75,000.00, issuing in lieu thereof Transfer
Certificate of Title.
On April 15, 1980, herein spouses Jose Armada and Remedios
Almanzor, filed a complaint for Annulment of Deed of Sale and
Reconveyance of Title with Damages, against herein petitioners Anita
and Serafin Si and Conrado Isada, brother-in-law of Cresenciana.
Isada brokered the sale. The complaint alleged that Conrado Isada
sold Crisostomos share by making it appear that Cresenciana, the
attorney-in-fact of her husband, is a Filipino citizen. By this time,
Crisostomo and Cresenciana had migrated and were already citizens
of the United States of America.
Petitioners contend that since the property was already three
distinct parcels of land, there was no longer co-ownership among the
brothers. Hence, Jose and Severo, Jr. had no right of redemption
when Crisostomo sold his share to the spouses Si.
The Court of Appeals reversed the decision of the Regional Trial
Court of Pasig City and nullified the sale of the subject lot by the
spouses Crisostomo and Cresenciana Armada to spouses Serafin and
Anita Si.

78

Petitioners filed their motion for new trial under Section 1, Rule
53 of the Revised Rules of Court. Petitioners presented new evidence,
registered in the name of Escolastica de la Rosa, married to Severo
Armada. CA ruling for the new trial, respondent court denied the
motion, reasoning that when the motion was filed, the reglementary
period had lapsed and the decision had become final and executory.
Petitioners motion for reconsideration of said resolution was denied.
ISSUE:
Whether or not private respondents are co-owners who are
legally entitled to redeem the lot under Article 1623 of the Civil Code.
RULING:
No, respondents are not co-owners since the brothers had a
physical division of the lot, the community ownership terminated and
right of redemption for each brother was no longer available.
Rightfully, as early as October 2, 1954, the lot in question had
already been partitioned when their parents executed three (3) deed
of sales in favor of Jose, Crisostomo and Severo, all surnamed
Armada. Notably, every portion conveyed and transferred to the three
sons was definitely described and segregated and with the
corresponding technical description. In short, this is what we call
extrajudicial partition. Moreover, every portion belonging to the three
sons has been declared for taxation purposes.
The fact that the three portions are embraced in one certificate
of title does not make said portions less determinable or identifiable
or distinguishable, one from the other, nor that dominion over each
portion less exclusive, in their respective owners. Hence, no right of
redemption among co-owners exists. After the physical division of the
lot among the brothers, the community ownership terminated, and
the right of preemption or redemption for each brother was no longer
available.
Under Art. 484 of the Civil Code, there is co-ownership
whenever the ownership of an undivided thing or right belongs to
different persons. There is no co-ownership when the different
portions owned by different people are already concretely determined
and separately identifiable, even if not yet technically described.

79

G.R. No. 125233

March 9, 2000

Spouses ALEXANDER CRUZ and ADELAIDA CRUZ, petitioners, vs.


ELEUTERIO LEIS, RAYMUNDO LEIS, ANASTACIO L. LAGDANO,
LORETA L. CAYONDA and the HONORABLE COURT OF APPEALS,
respondents.
FACTS:
Adriano and Gertrudes were married on 19 April 1923. On 27
April 1955, Gertrudes acquired from the then DANR a parcel of land
with an area of one hundred (100) square meters, situated at Bo. Sto.
Nio, Marikina, Rizal. The Deed of Sale described Gertrudes as a
widow. On 2 March 1956, TCT No. 43100 was issued in the name of
"Gertrudes Isidro," who was also referred to therein as a "widow." On
2 December 1973, Adriano died and it does not appear that he
executed a will before his death.
On 5 February 1985, Gertrudes obtained a loan from
petitioners, the spouses Alexander and Adelaida Cruz, in the amount
of P15,000.00. The loan was secured by a mortgage over the property
covered by TCT No. 43100. Gertrudes, however, failed to pay the loan
on the due date.
Unable to pay her outstanding obligation after the debt became
due and payable, Gertrudes executed two contracts in favor of
petitioner Alexander Cruz. The first is denominated as "Kasunduan"
which the parties concede is a pacto de retro sale, granting Gertrudes
one year within which to repurchase the property. The second is a
"Kasunduan ng Tuwirang Bilihan," a Deed of Absolute Sale covering
the same property for the price of P39,083.00, the same amount
stipulated in the "Kasunduan."
For failure of Gertrudes to repurchase the property, ownership
thereof was consolidated in the name of Alexander Cruz and a new
TCT title was issued in his name. When Gertrudes Isidro died, her
heirs, herein private respondents, received demands to vacate the
premises from petitioners, the new owners of the property. Private
respondents responded by filing a complaint seeking the nullification
of the contracts of sale over a lot executed by Gertrudes Isidro before
the RTC.
The RTC rendered a decision in favor of private respondents.
The RTC held that the land was conjugal property since the same was
acquired during the marriage of the spouses and that Adriano
contributed money for the purchase of the property. Thus, the court
80

concluded, Gertrudes could only sell to petitioner spouses her onehalf share in the property. Petitioners appealed to the CA in vain. The
Court of Appeals affirmed the decision of the RTC.
ISSUE:
Whether or not a co-owner may acquire exclusive ownership
over the property held in common.
RULING:
No. The right of repurchase may be exercised by a co-owner
with respect to his share alone. While the records show that petitioner
redeemed the property in its entirety, shouldering the expenses
therefor, that did not make him the owner of all of it. In other words,
it did not put to end the existing state of co-ownership. There is no
doubt that redemption of property entails a necessary expense. The
result is that the property remains to be in a condition of coownership. It is conceded that, as a rule, a co-owner such as
Gertrudes could only dispose of her share in the property owned in
common. Article 493 of the Civil Code provides:
Art. 493. Each co-owner shall have the full ownership of his
part of the fruits and benefits pertaining thereto, and he may
therefore alienate, assign or mortgage it, and even substitute
another person in its enjoyment, except when personal rights
are involved. But the effect of the alienation or the mortgage,
with respect to the co-owners, shall be limited to the portion
which may be allotted to him in the division upon the
termination of the co-ownership.
Unfortunately for private respondents, the property was
registered solely in the name of "Gertrudes Isidro, widow." Where a
parcel of land, forming past of the undistributed properties of the
dissolved conjugal partnership of gains, is sold by a widow to a
purchaser who merely relied on the face of the certificate of title
thereto, issued solely in the name of the widow, the purchaser
acquires a valid title to the land even as against the heirs of the
deceased spouse. The rationale for this rule is that "a person dealing
with registered land is not required to go behind the register to
determine the condition of the property. He is only charged with
notice of the burdens on the property which are noted on the face of
the register or the certificate of title. To require him to do more is to
defeat one of the primary objects of the Torrens system.

81

G.R. No. 168943

October 27, 2006

IGLESIA NI CRISTO, petitioner, vs. HON. THELMA A.


PONFERRADA, in her capacity as Presiding Judge, Regional Trial
Court, Br. 104, Quezon City, and HEIRS OF ENRIQUE G. SANTOS,
respondents.
FACTS:
Enrique Santos was the alleged owner with his TCT No. 57272
issued by the Register of Deeds on July 27, 1961. After his death, he
was survived by his wife, Alicia Santos, and their children Alfredo,
Roberto, Enrique and Susan. Thereafter, they took peaceful and
adverse possession of the property.
In February 1996, Iglesia ni Cristo (INC) was claiming
ownership over the property based on TCT No. 321744 issued on
September 18, 1984. Santoses had the property fenced but INC
deprived them of the final use and enjoyment of their property.
On 2001, the Santoses, represented by Enrique G. Santos, filed
a complaint for Quieting of Title and/or Accion Reinvindicatoria
before the RTC of Quezon City against the INC. INC moved to dismiss
complaint on the grounds that the action (either Quieting of Title or
Accion Reinvindicatoria) had prescribed, having been filed only on
October 24, 2001 beyond the statutory ten-year period.
The case involved more than one plaintiff but the verification
and certification against forum shopping incorporated in the
complaint was signed only by Enrique Santos; there having no
showing that Enrique Santos was, authorized to so represent the
other plaintiffs to file the complaint and to sign the verification and
certification of non-forum shopping.
The trial court denied INCs motion to dismiss. Unsatisfied,
INC filed a Petition for Certiorari and Prohibition before the CA. The
CA dismissed the petition.
ISSUE:
Whether or not Enrique Santos can represent his co-owners
with or without authority in an action for the recovery of
possession of real property.
Whether or not an action to quiet title prescribe from
dispossession.
82

RULING:
First issue: Yes. As heirs, the Santoses, they are considered coowners pro indiviso of the whole property since no specific portion
yet has been adjudicated to any of them. Consequently, as one of the
heirs and principal party, the lone signature of Enrique G. Santos in
the verification and certification is sufficient for the RTC to take
cognizance of the case. The commonality of their interest gave
Enrique G. Santos the authority to inform the RTC on behalf of the
other plaintiffs therein that they have not commenced any action or
claim involving the same issues in another court or tribunal, and that
there is no other pending action or claim in another court or tribunal
involving the same issues.
As co-owners, each of the heirs may properly bring an action for
ejectment, forcible entry and detainer, or any kind of action for the
recovery of possession of the subject properties. Thus, a co-owner
may bring such an action, even without joining all the other coowners as co-plaintiffs, because the suit is deemed to be instituted for
the benefit of all.
Second issue: No. An action for quieting of title is
imprescriptible until the claimant is ousted of his possession. The
owner of a real property, as plaintiff, is entitled to the relief of
quieting of title even if, at the time of the commencement of his
action, he was not in actual possession of real property. Under Article
477 of the New Civil Code, the owner need not be in possession of the
property.

83

G.R. No. 122904

April 15, 2005

ADORACION E. CRUZ, THELMA DEBBIE E. CRUZ, GERRY E.


CRUZ and NERISSA CRUZ-TAMAYO, Petitioner, vs. THE
HONORABLE COURT OF APPEALS, SUMMIT FINANCING CORP.,
VICTOR S. STA. ANA, MAXIMO C. CONTRERAS, RAMON G.
MANALASTAS, and VICENTE TORRES, Respondents.
FACTS:
Petitioners and Arnel Cruz were co-owners of a parcel of land
situated in Taytay, Rizal. Yet the property was registered only in the
name of Arnel Cruz. According to petitioners, the property was
among the properties they and Arnel Cruz inherited upon the death of
Delfin Cruz, husband of Adoracion Cruz.
Petitioners and Arnel Cruz executed a Deed of Partial Partition,
distributing to each of them their shares consisting of several lots
previously held by them in common. Among the properties
adjudicated to defendant Cruz was the parcel of land covered at the
time by TCT No. 495225 which is the subject of this case.
Subsequently, the same parties to the Deed of Partial Partition
agreed in writing to share equally in the proceeds of the sale of the
properties although they had been subdivided and individually titled
in the names of the former co-owners pursuant to the Deed of Partial
Partition.
On June 4, 1980, a Real Estate Mortgage was constituted on the
disputed property then covered by TCT No. 495225 to secure the loan
obtained by Arnel Cruz thru Nelson Tamayo from respondent
Summit. Since the loan had remained outstanding on maturity,
Summit instituted extrajudicial foreclosure proceedings, and at the
foreclosure sale it was declared the highest bidder.
A Certificate of Sale was issued to respondent Summit, which
more than a year later, consolidated its ownership of the foreclosed
property. In their complaint before the RTC, petitioners asserted that
they co-owned the properties with Arnel Cruz and argued that the
mortgage was void since they did not consent to it.
The trial court rendered a judgment in favor of the plaintiffs.
The CA reversed the trial court's decision.
84

ISSUE:
Whether or not the real estate mortgage on the property then
covered by TCT No. 495225 is valid.
RULING:
YES. Resolution of the issue in turn depends on the
determination of whether the mortgaged property was the exclusive
property of Arnel Cruz when it was mortgaged.
Co-ownership is terminated upon judicial or extra-judicial
partition of the properties owned in common. Partition, in general, is
the separation, division and assignment of a thing held in common
among those to whom it may belong. Every act which is intended to
put an end to indivision among co-heirs and legatees or devisees is
deemed to be a partition, although it should purport to be a sale, an
exchange, a compromise, or any other transaction.
In the aforesaid deed, the shares of petitioners and Arnel Cruz's
in the mass of co-owned properties were concretely determined and
distributed to each of them. Since a partition legally made confers
upon each heir the exclusive ownership of the property adjudicated to
him, it follows that Arnel Cruz acquired absolute ownership over the
specific parcels of land assigned to him in the Deed of Partial
Partition, including the property subject of this case. As the absolute
owner thereof then, Arnel Cruz had the right to enjoy and dispose of
the property, as well as the right to constitute a real estate mortgage
over the same without securing the consent of petitioners.

85

G.R. No. 121069

February 7, 2003

BENJAMIN CORONEL AND EMILIA MEKING VDA. DE CORONEL,


petitioners,
vs.
FLORENTINO
CONSTANTINO,
AUREA
BUENSUCESO, AND THE HONORABLE COURT OF APPEALS,
respondents.
FACTS:
The property involves two adjacent parcels of land situated in
Sta. Monica, Hagonoy, Bulacan. The property is originally owned by
Honoria Aguinaldo. One-half (1/2) of it was inherited by Emilia
Meking Vda. de Coronel together with her sons Benjamin, Catalino
and Ceferino, all surnamed Coronel. The other half was inherited by
Florentino Constantino and Aurea Buensuceso.
In 1991, Constantino and Buensuceso filed a complaint for
declaration of ownership, quieting of title and damages with prayer
for writ of mandatory and/or prohibitory injunction with the
Regional Trial Court against the Coronels. They allege that Jess C.
Santos and Priscilla Bernardo purchased the property belonging to
Coronels by virtue of a deed of sale signed by Emilia alone. Santos
and Bernardo sold the same to Constantino and Buensuceso by virtue
of a compromise agreement in a Civil Case. As such, Constantino and
Buensuceso claim that they are the owners of the subject property.
The RTC decided in favor of Constantino and Buensuceso. It
was appealed but the CA affirmed the decision of the lower court.
ISSUE:
Whether or not the contract of sale executed by a parent-coowner, in her own behalf, is enforceable with respect to the shares of
her co-heirs-children.
RULING:
NO. In the "Kasulatan ng Bilihang Patuluyan" which is a private
document, not having been duly notarized, it clearly shows that only
the share of Emilia in the subject property was sold because Benjamin
did not sign the document and the shares of Ceferino and Catalino
were not subject of the sale. Therefore, it is unenforceable against the
three of them. Emilia executed the instrument in her own behalf and
not in representation of her three children.

86

Article 493 of the Civil Code provides that "Each co-owner shall
have the full ownership of his part and of the fruits and benefits
pertaining thereto, and he may therefore alienate, assign or mortgage
it, and even substitute another person in its enjoyment, except when
personal rights are involved. But the effect of the alienation or the
mortgage, with respect to the co-owners, shall be limited to the
portion which may be allotted to him in the division upon the
termination of the co-ownership." Consequently, the sale of the
subject property made by Emilia in favor of Santos and Bernardo is
limited to the portion which may be allotted to her upon the
termination of her co-ownership over the subject property with her
children.
The sale is valid insofar as the share of Emilia is concerned.
Hence, Santos and Bernardo, who purchased the share of Emilia,
became co-owners of the subject property together with Benjamin
and the heirs of Ceferino and Catalino. As such, Santos and Bernardo
could validly dispose of that portion of the subject property
pertaining to Emilia in favor of Constantino and Buensuceso.
However, the particular portions properly pertaining to each of the
co-owners are not yet defined and determined as no partition in the
proper forum or extrajudicial settlement among the parties has been
effected among the parties.

87

G.R. No. 120864

October 8, 2003

MANUEL T. DE GUIA, petitioner, vs. COURT OF APPEALS (Former


Sixth Division) and JOSE B. ABEJO, represented by his Attorney-inFact, Hermenegilda Abejo-Rivera, respondents.
FACTS:
Abejo alleged that he is the owner of the undivided portion of
a property used as a fishpond ("FISHPOND") situated in
Meycauayan, Bulacan. He alleged ownership over approximately
39,611 square meters out of the fishponds total area of 79,220 square
meters. Abejo further averred that De Guia continues to possess and
use the fishpond without any contract and without paying rent to
Abejos damage and prejudice. Abejo also complained that De Guia
refuses to surrender ownership and possession of the fishpond
despite repeated demands to do so after De Guias sublease contract
over the fishpond had expired.
De Guia, a lawyer by profession, alleged that the complaint does
not state a cause of action and has prescribed. He claimed that the
fishpond was originally owned by Maxima Termulo who died
intestate with Primitiva Lejano as her only heir. According to him,
Abejo is not the owner of the entire fishpond but the heirs of
Primitiva Lejano who authorized him to possess the entire fishpond.
He assailed ABEJOs ownership of the undivided portion of the
FISHPOND as void and claimed ownership over an undivided half
portion of the FISHPOND for himself. DE GUIA sought payment of
damages and reimbursement for the improvements he introduced as
a builder in good faith.
The trial court ruled in favor of Abejo. The case was raised to
the CA which affirmed the trial courts decision.
ISSUE:
Whether or not a co-owner can file an action of recovery of
possession against a co-owner.
RULING:
Yes. Under Article 484 of the Civil Code, "there is co-ownership
whenever the ownership of an undivided thing or right belongs to
different persons."

88

Any co-owner may file an action under Article 487 not only
against a third person, but also against another co-owner who takes
exclusive possession and asserts exclusive ownership of the property.
In the latter case, however, the only purpose of the action is to obtain
recognition of the co-ownership. The plaintiff cannot seek exclusion
of the defendant from the property because, as co-owner, he has a
right of possession. The plaintiff cannot recover any material or
determinate part of the property.
Abejo and De Guia are owners of the whole and over the whole,
they exercise the right of dominion. However, they are at the same
time individual owners of a portion, which is truly abstract because
until there is partition, such portion remains indeterminate or
unidentified. As co-owners, Abejo and De Guia may jointly exercise
the right of dominion over the entire fishpond until they partition the
fishpond by identifying or segregating their respective portions.
Since a co-ownership subsists between Abejo and De Guia,
judicial or extra-judicial partition is the proper recourse. An action to
demand partition is imprescriptible and not subject to laches. Each
co-owner may demand at any time the partition of the common
property unless a co-owner has repudiated the co-ownership under
certain conditions. Neither Abejo nor De Guia has repudiated the coownership under the conditions set by law.

89

G.R. No. 205867

February 23, 2015

MARIFLOR T. HORTIZUELA, represented by JOVIER TAGUFA,


Petitioner, vs. GREGORIA TAGUFA, ROBERTO TAGUFA and
ROGELIO LUMABAN, Respondents.
FACTS:
The property involved in this case is a parcel of land containing
an area of 539 square meters. By virtue of the special power of
attorney executed by Mariflor Tagufa Hortizuela, Jovier Tagufa
instituted the instant case against the defendants praying for the
peaceful surrender of the above-described property unto them and
further ordering defendant Gregoria Tagufa to reconvey in plaintiffs
favor the same property which was titled under her name via fraud.
Before it was titled in the name of Defendant Tagufa, said
property was originally owned by plaintiffs parents, Spouses Epifanio
Tagufa and Godofreda Jimenez. Although untitled, the spouses
mortgaged the property with the Development Bank of the
Philippines (DBP). For failure to redeem the property, DBP
foreclosed the same and sold it to Atty. Romulo Marquez who, in
turn, sold it back to Runsted Tagufa, husband of defendant Gregoria
Tagufa, on April 4, 2002 using the fund sent by plaintiff Hortizuela
who was in America and with the agreement that Runsted will
reconvey the said property to her sister when demanded.
However, plaintiff discovered that the same unregistered
property was titled in the name of Gregoria Tagufa. Investigating
further, plaintiff discovered that Gregoria Tagufa was able to title the
said property by virtue of a free patent application before the
Department of Environment and Natural Resources (DENR) and the
execution of a Deed of Extrajudicial Settlement of the Estate of the
late Spouses Leandro Tagufa and Remedios Talosig dated May 9,
2003. Plaintiff now seeks to recover possession of the said property
which is presently occupied by Gregoria Tagufa and her codefendants and have the same be reconveyed unto them.
MCTC dismissed the complaint for lack of merit. Not in
conformity, Hortizuela appealed to the RTC. RTC reversed the MCTC
ruling and granted the appeal of the plaintiff, ordering the defendant
Gregorio Tagufa to reconvey to the plaintiff Mariflor Tagufa
Hortizuela the land described in the complaint; it also ordered the
defendants to vacate the same land and to surrender the peaceful
possession thereof to the plaintiff. CA reversed the decision.

90

ISSUE:
Whether or not an action for reconveyance and recovery of
possession constitutes an indirect collateral attack on the validity of
the subject Certificate of Title which is proscribed by law.
RULING:
The Supreme Court granted the petition and reversed the
decision of the CA. The Court is not unmindful of the principle of
indefeasibility of a Torrens title and Section 48 of P.D. No. 1528
where it is provided that a certificate of title shall not be subject to
collateral attack. A Torrens title cannot be altered, modified or
cancelled except in a direct proceeding in accordance with law. When
the Court says direct attack, it means that the object of an action is to
annul or set aside such judgment, or enjoin its enforcement. On the
other hand, the attack is indirect or collateral when, in an action to
obtain a different relief, an attack on the judgment or proceeding is
nevertheless made as an incident thereof.
The complaint of Hortizuela was not a collateral attack on the
title warranting dismissal. As a matter of fact, an action for
reconveyance is a recognized remedy, an action in personam,
available to a person whose property has been wrongfully registered
under the Torrens system in anothers name. In an action for
reconveyance, the decree is not sought to be set aside. It does not
seek to set aside the decree but, respecting it as incontrovertible and
no longer open to review, seeks to transfer or reconvey the land from
the registered owner to the rightful owner. Reconveyance is always
available as long as the property has not passed to an innocent third
person for value.
The fact that petitioner was able to secure a title in her name
did not operate to vest ownership upon her of the subject land.
Registration of a piece of land under the Torrens System does not
create or vest title, because it is not a mode of acquiring ownership. A
certificate of title is merely an evidence of ownership or title over the
particular property described therein. It cannot be used to protect a
usurper from the true owner; nor can it be used as a shield for the
commission of fraud; neither does it permit one to enrich himself at
the expense of others. Its issuance in favor of a particular person
does not foreclose the possibility that the real property may be coowned with persons not named in the certificate, or that it may be
held in trust for another person by the registered owner.

91

G.R. No. 154415. July 28, 2005


GASPAR CALACALA, BALTAZAR CALACALA, MELCHOR
CALACALA, SOLOMON CALACALA, FELICIDAD CALACALA,
PETRONILA CALACALA and SALOME CALACALA, Petitioners, vs.
REPUBLIC OF THE PHILIPPINES, represented by the Solicitor
General, and SHERIFF JUAN C. MARQUEZ,Respondents.
FACTS:
The predecessors-in-interest, then Spouses Camilo and
Conchita Calacala, of herein petitioners, used their property to secure
a bond for the temporary liberty of an accused in a criminal case. For
failure of such accused to face trial, the bond was forfeited in favor of
the government and the same was ultimately levied upon and an
annotation was made on the title and registered with the ROD.
An auction was held with the Republic winning the bid, thus a
Sheriffs Deed of Sale was issued in favor of the Republic and the
same was registered with the ROD in 1982. A one year redemption
period was set for the owners to redeem the property but to no avail
and such non-redemption event went beyond their delivery to their
respective graves, in 1988 and 1994.
The Calacalas, as legal heirs of the deceased spouses, petitioners
filed a complaint for Quieting of Title and Cancellation of
Encumbrance on TCT No. T-21204 against respondents Republic and
Sheriff Juan C. Marquez. In their complaint, petitioners prayed for
the cancellation of entries on the title subject of this case or the
declaration of said entries as null and void.
The trial court granted the Republics motion to dismiss and
accordingly dismissed petitioners complaint.
A Motion for
Reconsideration was filed and was likewise dismissed.
Thus, a petition for review on certiorari under Rule 45 of the
Rules of Court, was lodged by petitioners to the SC asking to annul
and set aside the resolutions of the Regional Trial Court which
respectively dismissed petitioners complaint and denied their motion
for reconsideration.

92

ISSUE:
Whether or not the trial courts dismissal of petitioners
complaint for Quieting of Title proper
RULING:
YES, the Dismissal is proper. Citing Article 476 and 477 of the
New Civil Code, the Court said that the remedy of quieting a title may
be availed of only when, by reason of any instrument, record, claim,
encumbrance or proceeding, which appears valid but is, in fact,
invalid, ineffective, voidable or unenforceable, a cloud is thereby casts
on the complainants title to real property or any interest therein and
that the party who may bring an action to quiet title must have legal
or equitable title to, or interest in the real property which is the
subject-matter of the action. He need not be in possession of said
property.
Thus, the Court ruled that for an action to quiet title to prosper,
two (2) indispensable requisites must concur, namely: (1) the plaintiff
or complainant has a legal or an equitable title to or interest in the
real property subject of the action; and (2) the deed, claim,
encumbrance or proceeding claimed to be casting cloud on his title
must be shown to be in fact invalid or inoperative despite its prima
facie appearance of validity or legal efficacy.
Absent the two requisites in the petition of the Calacalas, the
Court upheld the decision of the RTC.

93

G.R. No. 148943

August 15, 2002

AGNES GAPACAN, EUGENIA GAPACAN-KIAKI and MARILYN


GAPACAN, petitioners, vs. MARIA GAPACAN OMIPET, respondent.
FACTS:
Antonio Gapacan and Maria Gapacan Omipet are children of
Paicat Gapacan. Paicat left an unregistered land without will upon
his death. In the adult years of the siblings life, Antonio Gapacan left
home to find life outside their homestead while Maria remained to
cultivate the land and took care of their aging father until his death
during the World War II.
Antonio, now married, retired and with his wife and children,
returned to his place of birth and there established his family by
farming the land owned by their parents. However, instead of just
farming, Antonio, in 1954, executed an Affidavit of Transfer of Real
Property showing that the property of their father had been
transferred to him by his sister Maria, making him in effect the legal
owner of the property in question. The Affidavit of Transfer of Real
Property was allegedly thumbmarked by Maria's husband, Pedro
Omipet, in her behalf. Thus, by virtue of the Affidavit of Transfer of
Real Property, Antonio had the property in question declared in his
name for taxation purposes in 1954. Since then, Agnes Gapacan and
her daughters Eugenia and Marilyn had been occupying and
cultivating the three (3) parcels of rice land and a parcel devoted to
camote subject matter of the present controversy.
As such, when Maria took time to cultivate said land, she was
faced by opposition from Antonios family which landed in the courts
in a case of forcibly entry, depriving Maria her better share on the
property.
Given the situation, Maria moved for a quieting of title, alleging
in her complaint that the disputed land was part of her inheritance
from her deceased parents which she in fact had declared in her name
for taxation purposes in 1948 although the area was only 1,188 square
meters for which Tax Declaration No. A-0808 was issued in her
name. She further contended that she merely lent the parcels of rice
land to petitioners when Antonio Gapacan returned to Abatan after
his retirement.
The trial court dismissed the complaint. On appeal, CA
rendered a decision declaring the property to be the common
property of both the petitioner and respondent.
94

ISSUE:
Whether or not the heirs of Antonio Gapacan has a better right
than Maria Gapacan Omipet given their actual possession of the land
and the tax declarations issued on formers favor?
RULING:
Article 476 of the Civil Code provides that an action to quiet title
may be brought when there exists a cloud on the title to real property
or any interest therein. In the case of Bautista v. Exconde, we held
that a property owner whose property rights were being disturbed
may ask a competent court for a proper determination of the
respective rights of the party-claimants, not only to place things in
their proper place, that is, to require the one who has no right to
refrain from acts injurious to the peaceful enjoyment of the property
not only of the rightful owner but also for the benefit of both with the
view of dissipating any cloud of doubt over the property.
It goes without saying therefore that the appellate court in
resolving the present controversy is well within its authority to
adjudicate on the respective rights of the parties, that is, to pass upon
the ownership of the subject property; hence to declare the same as
common property of the party-litigants. Besides, private respondent
Maria Gapacan Omipet instituted the present action for the purpose
of asking the court to pass judgment upon the issue of ownership of
the disputed property with the hope that she would be declared its
rightful owner.
Thus, as the Court noted, while the Heirs of Antonio proved
actual possession, such possession was a possession in bad faith, and
while Maria has a tax declaration to show her claim to be valid, tax
declarations in themselves do not vest absolute ownership of the
property upon the declarant, nor do declarations of ownership for
taxation purposes constitute adequate evidence of ownership or of the
right to possess realty.
It is therefore proper for the CA to declare co-ownership on the
juridical concept that co-ownership is unity of the object or property
and plurality of subjects x x x x Each co-owner, jointly with the other
co-owners, is the owner of the whole property, but at the same time of
the undivided aliquot part x x x x Each co-owner has the right to sell,
assign or dispose of his share, unless personal rights are involved x x
x and therefore, he may lose such rights to others, as by prescription
thereof by a co-owner x x x x"

95

G.R. No. 144208

September 11, 2007

EFREN TANDOG, FELIX TANDOG, FELIPE TANDOG, JOSEFINO


TANDOG, HELEN TANDOG, CATALINA TANDOG, ROMEO
TANDOG, DOMINGO TANDOG, CATALINA SANTOS, MARIA
BAUTISTA CATANYAG, ARTEMIO CATANYAG, ANGELES
CATANYAG, APOLONIA CATANYAG, ADORACION CATANYAG,
ARCELY CATANYAG, and AMPARO CATANYAG, all represented by
EFREN TANDOG, petitioners, vs. RENATO MACAPAGAL, SPOUSES
ALFONSO and MARINA CALDERON, and the LANDS
MANAGEMENT BUREAU, respondents.
FACTS:
The subject of the controversy is a land consisting of 147,991
square meters situated at Sitio Inarawan, Barangay Inuman, San
Isidro, Antipolo City.
Petitioners claim that they and their predecessors-in-interest
have been in actual, open, continuous, exclusive, and notorious
possession of the land since time immemorial. They trace their rights
to Casimiro Policarpio, unmarried, who died in 1945. He was survived
by his nephews and nieces, now deceased, except Maria Bautista
Catanyag. She and Casimiros grandnieces and grandnephews (herein
petitioners) have continued possessing and cultivating the land.
When petitioners decided to apply for the judicial registration of the
property, they found that portions of the land have been occupied by
spouses Alfonso and Marina Calderon and Renato Macapagal,
respondents.
Respondent Marina Calderon, in her answer, specifically denied
petitioners allegations in their complaint. She alleged that she and
her husband bought their property in 1958 and, since then, have been
in possession of the same. They planted trees and crops thereon. Also,
they have been paying the corresponding realty taxes. She does not
know petitioners who are all strangers in the place.
After petitioners had presented their evidence, spouses
Calderon filed a demurrer to evidence. The trial court granted their
motion and dismissed the complaint. On appeal by petitioners, the
Court of Appeals rendered a decision affirming the order of the trial
court dismissing their complaint.

96

ISSUE:
Whether or not the allegations of spouses Calderon that they
purchased their property and Macapagals claim that he applied for a
Free Patent are judicial admissions which the petitioners consider as
cloud upon their interest in the disputed property.
RULING:
As a general rule, a cloud which may be removed by suit to quiet
title is not created by mere verbal or parol assertion of ownership of
or an interest in property. This rule is subject to qualification, where
there is a written or factual basis for the asserted right. Thus, a claim
of right based on acquisitive prescription or adverse possession has
been held to constitute a removable cloud on title.
While petitioners alleged that respondents claim of adverse
possession is a cloud on their (petitioners) interest in the land,
however, such allegation has not been proved. The alleged falsified
documents relied upon by respondents to justify their possession
were merely marked as exhibits but were never formally offered in
evidence by petitioners. We have consistently ruled that documents
which may have been marked as exhibits during the hearing, but
which were not formally offered in evidence, cannot be considered as
evidence, nor can they be given any evidentiary value.
It is important that petitioners must first establish their legal or
equitable title to, or interest in the real property which is the subject
matter of the action. Petitioners failed to do so. Parenthetically, they
did not present any evidence to prove that Casimiro Policarpio
existed and that he is their predecessor-in-interest.
Records show that petitioners failed to establish by evidence
any or all the above requisites.

97

G.R. No. 157852

December 15, 2010

HEIRS OF DOMINGO VALIENTES, Petitioners, vs. Hon. REINERIO


(Abraham) B. RAMAS, Acting Presiding Judge, RTC, Branch 29, 9th
Judicial Region, San Miguel, Zamboanga del Sur and Vilma V. Minor,
Respondents.
FACTS:
Petitioners claim that they are the heirs of Valientes who, before
his death, was the owner of a parcel of land in Zamboanga del Sur. In
1939, Domingo Valientes mortgaged the subject property to secure
his loan to the spouses Leon Belen and Brigida Sescon.
In the 1950s, the Valientes family purportedly attempted, but
failed, to retrieve the subject property from the spouses Belen.
Through an allegedly forged document captioned VENTA
DEFINITIVA purporting to be a deed of sale of the subject property
between Domingo Valientes and the spouses Belen, the latter
obtained Transfer Certificate of Title (TCT) No. T-5,427 in their
name.
On February 28, 1970, Maria Valientes Bucoy and Vicente
Valientes, legitimate children of the late Domingo Valientes, had their
Affidavit of Adverse Claim duly entered in the Memorandum of
Encumbrances at the back of TCT No. T-5,427. Upon the death of the
spouses Belen, their surviving heirs Brigida Sescon Belen and Maria
Lina Belen executed an extra-judicial settlement with partition and
sale in favor of private respondent Vilma Valencia-Minor, the present
possessor of the subject property.
On June 20, 1979, Minor then filed with the Court of First
Instance a petition for cancellation of memorandum of encumbrance
appearing in TCT No. T-5,427. On July 31, 2000, the RTC granted
Minors prayer to allow the Register of Deeds to have the title to the
subject property transferred to her name.
In the meantime, on August 20, 1998, petitioners filed a
complaint for the cancellation of TCT NO. T-5,427. The Complaint
was docketed as Civil Case No. 98-021.
Private respondent Minor filed an Omnibus Motion to Dismiss
Civil Case No. 98-021 on the grounds of forum shopping and litis
pendentia. On August 3, 2000, the RTC issued an order in open court
ruling that forum shopping does not apply. On September 22, 2000,
98

private respondent Minor filed a Motion for Reconsideration of the


August 3, 2000 Order.
On May 7, 2001, the RTC issued an Order granting the Motion
for Reconsideration by dismissing Civil Case No. 98-021 on the
ground of forum shopping. Petitioners filed a Motion for
Reconsideration on May 30, 2001, but the same was denied by the
RTC in its Order dated September 18, 2001.
On November 12, 2001, petitioners filed with the Court of
Appeals a Petition for Certiorari assailing the RTC Orders dated May
7, 2001 and September 18, 2001.
Petitioners raised the sole issue of whether the trial court was
correct in finding that Civil Case No. 98-021 constitutes forum
shopping, litis pendentia or res judicata with SPL Case No. 186. The
Petition was docketed as CA-G.R. SP No. 68501.
The Court of Appeals rendered its assailed Decision on said
petition on August 16, 2002. Despite agreeing with petitioners that
there was no forum shopping, litis pendentia or res judicata in the
filing of Civil Case No. 98-021, the Court of Appeals, asserting that it
has the discretion to review matters not otherwise assigned as errors
on appeal if it finds that their consideration is necessary at arriving at
a complete and just resolution of the case, held that Civil Case No. 98021 cannot prosper on the grounds of prescription and laches. Hence,
the Petition for Certiorari.
ISSUE:
Whether or not prescription or laches has already set in to bar
the filing of the case at hand.
HELD:
YES. The case cannot prosper because an action for
reconveyance is a legal remedy granted to a landowner whose
property has been wrongfully or erroneously registered in anothers
name, which must be filed within ten years from the issuance of the
title since such issuance operates as a constructive notice (Declaro vs.
Court of Appeals, 346 SCRA 57).

99

Where a party has neglected to assert his rights over a property


in question for an unreasonably long period, he is estopped from
questioning the validity of another persons title to the property. Long
inaction and passivity in asserting ones rights over a disputed
property precludes him from recovering said property (Po Lam vs.
Court vs. Court of Appeals, 347 SCRA 86).
In conclusion, petitioners cause of action has already
prescribed and now heavily infirmed with laches.
The cause of action of petitioners in Civil Case No. 98-021,
wherein they claim that private respondent Minors predecessor-ininterest acquired the subject property by forgery, can indeed be
considered as that of enforcing an implied trust. In particular, Article
1456 of the Civil Code provides:
Art. 1456. If property is acquired through mistake or fraud, the
person obtaining it is, by force of law, considered a trustee of an
implied trust for the benefit of the person from whom the property
comes.
However, the Court made a clear distinction in Olviga: when the
plaintiff in such action is not in possession of the subject property, the
action prescribes in ten years from the date of registration of the deed
or the date of the issuance of the certificate of title over the property.
When the plaintiff is in possession of the subject property, the action,
being in effect that of quieting of title to the property, does not
prescribe. In the case at bar, petitioners (who are the plaintiffs in Civil
Case No. 98-021) are not in possession of the subject property.
If the Civil Case No. 98-021 were to be considered as that of
enforcing an implied trust, it should have therefore been filed within
ten years from the issuance of TCT No. T-5,427 on December 22,
1969. It was, however, filed on August 20, 1998, which was way
beyond the prescriptive period.
Under the Torrens System as enshrined in P.D. No. 1529, the
decree of registration and the certificate of title issued become
incontrovertible upon the expiration of one year from the date of
entry of the decree of registration, without prejudice to an action for
damages against the applicant or any person responsible for the
fraud.

100

SC have allowed actions for reconveyance based on implied trusts


even beyond such one-year period, for such actions respect the decree
of registration as incontrovertible.
Yet, the right to seek reconveyance based on an implied or
constructive trust is not absolute nor is it imprescriptible. An action
for reconveyance based on an implied or constructive trust must
perforce prescribe in ten years from the issuance of the Torrens title
over the property.
It took petitioners 28 before filing this case. This period is
unreasonably long for a party seeking to enforce its right to file the
appropriate case. Thus, petitioners claim that they had not slept on
their rights is patently unconvincing.

101

G.R. No. 141964

June 30, 2006

SPOUSES EDESITO and CONSORCIA RAGASA, Petitioners, vs.


SPOUSES GERARDO and RODRIGA ROA and the EX-OFFICIO
SHERIFF OF QUEZON CITY, Respondents.
FACTS:
On May 10, 1989, Edesito and Consorcia Ragasa entered into a
contract with Oakland Development Resources Corporation for the
purchase in installments of a piece of property. Immediately
thereafter, they took possession of the property covered by TCT No.
27946 of the Registry of Deeds for Quezon City. From May of 1989 up
to the present date, plaintiffs were in continuous and notorious
possession of the property.
Sps. Ragasa were able to fully pay for the agreed purchase price
of the property and a Deed of Absolute Sale was executed and the
original owners copy of TCT No. 27946 was turned over to them.
Upon learning that Oakland Development Resources Corporation was
no longer functional as a corporate entity, Consorcia decided to cause
the transfer of registration herself.
She was thus surprised to learn from the Registry of Deeds for
Quezon City that the property in question was sold by Ex-Officio
Sheriff of Quezon City to defendants Sps. Roa as the highest bidder
for the price and consideration of P511,000.00 as shown in the
Sheriffs Final Deed of Sale.
Sps. Ragasa filed a case to RTC and instead of filing an answer,
respondents moved for the dismissal of the complaint on the grounds
of prescription and laches. RTC granted the motion. Characterizing
the suit as an action "upon an injury to the rights of the plaintiff"
which, according to Article 1146 of the Civil Code, must be filed within
four years, the RTC held that petitioners action was barred by
prescription for having been filed more than four years after the
registration of the execution sale.
Petitioners proceeded to the SC with the petition for review on
certiorari raising only a pure question of law and seeking a reversal of
the trial courts order.

102

ISSUE:
Whether or not prescription or laches has already set in to bar
the filing of the case at hand.
HELD
NO. SC held that the premise of RTC was erroneous. The
allegations in petitioners complaint reveal that the action was
essentially one for quieting of title to real property under Article 476
of the Civil Code which states that:
Whenever there is a cloud on title to real property or any
interest therein, by reason of any instrument, record, claim,
encumbrance or proceeding which is apparently valid or
effective but is in truth and in fact invalid, ineffective, voidable,
or unenforceable, and may be prejudicial to said title, an action
may be brought to remove such cloud or to quiet the title.
An action may also be brought to prevent a cloud being cast
upon title to real property or any interest therein.
To make out an action to quiet title, allegations should show
that (1) the plaintiff has "title to real property or any interest therein"
and (2) the defendant claims an interest therein adverse to the
plaintiffs arising from an "instrument, record, claim, encumbrance,
or proceeding which is apparently valid or effective but is in truth and
in fact invalid, ineffective, voidable, or unenforceable."
Thus, the averments in petitioners complaint that (1) they
acquired ownership of a piece of land by tradition or delivery as a
consequence of sale and (2) private respondents subsequently
purchased the same piece of land at an allegedly void execution sale
were sufficient to make out an action to quiet title under Article 476.
It is also an established rule that actions to quiet title to
property in the possession of the plaintiff are imprescriptible.
Accordingly, petitioners action was not subject to prescription.

103

G.R. No. 178609

August 04, 2010

MANUEL P. NEY AND ROMULO P. NEY V. SPOUSES CELSO P.


QUIJANO AND MINA N. QUIJANO
FACTS:
Petitioners Manuel P. Ney and Romulo P. Ney are the registered
owners of a residential lot located at 1648 Main Street, Paco Manila.
A three (3) door apartment was constructed on the subject lot - one
for Manuel, the other for Romulo; and the last one for their sister
Mina N. Quijano and her husband Celso Quijano.
On October 8, 1999, Quijano filed with the RTC of Manila a suit
for reconveyance, partition and damages against petitioners. They
averred that they are co-owners of the subject property having paid
part of its purchase price; that Celso's name was inadvertently
omitted as one of the buyers in the execution of the deed of sale.
Consequently, TCT No. 122489 covering the subject property was
issued only in the names of Manuel and Romulo.
To obtain a separate certificate of title, they requested from
petitioners the segregation of the portion allotted to them, but the
latter refused. They later discovered that the entire property was
mortgaged with Metropolitan Bank & Trust Company, prompting
them to execute and register their adverse claim with the Register of
Deeds; and to file the instant complaint,
Petitioners averred that Celso Quijano was not a vendee of the
subject lot; thus, his name did not appear on the title. They asserted
that respondents cannot validly maintain an action against them
because the latter possessed the property by mere tolerance; and even
assuming that respondents had a valid cause of action, the same had
already been barred by prescription and/or laches. Petitioners,
therefore, prayed for the dismissal of the complaint.
RTC sustained petitioners' assertion that respondents possessed
part of the property through mere tolerance; and that their cause of
action, if any, already prescribed. The RTC thus ruled that
respondents can no longer demand the segregation or reconveyance
of the claimed portion of the property. CA reversed the decision of
RTC. The CA considered respondents' complaint as one for quieting
of title which is imprescriptible; and granted to respondents the
reliefs that they prayed for.

104

ISSUE:
Whether or not respondents are entitled for reconveyance and if
the latters cause of action has been prescribed.
RULING:
Yes, respondents are entitled for their claim and the complaint
is one for quieting title which is imprescriptible.
These allegations make out a case for reconveyance. That
reconveyance was one of the reliefs sought was made abundantly
clear by respondents in their prayer. Undoubtedly, respondents did
not only seek the partition of the property and the delivery of the title,
but also the reconveyance of their share which was inadvertently
included in petitioners' TCT.
An action for reconveyance is one that seeks to transfer
property, wrongfully registered by another, to its rightful and legal
owner. Indeed, reconveyance is an action distinct from an action for
quieting of title, which is filed whenever there is a cloud on title to
real property or any interest therein, by reason of any instrument,
record, claim, encumbrance or proceeding which is apparently valid
or effective but is in truth and in fact, invalid, ineffective, voidable, or
unenforceable, and may be prejudicial to said title for purposes of
removing such cloud or to quiet title. However, the SC finds nothing
erroneous in the CA's ruling treating respondents' action for
reconveyance as an action to quiet title.
An action for reconveyance based on an implied trust prescribes
in 10 years. The reference point of the 10-year prescriptive period is
the date of registration of the deed or the issuance of the title. The
prescriptive period applies only if there is an actual need to reconvey
the property as when the plaintiff is not in possession of the property.
However, if the plaintiff, as the real owner of the property also
remains in possession of the property, the prescriptive period to
recover title and possession of the property does not run against him.
In such a case, an action for reconveyance, if nonetheless filed, would
be in the nature of a suit for quieting of title, an action that is
imprescriptible.

105

G.R. No. 157812 November 22, 2005


RODOLFO SANTOS, Petitioner, vs. RONALD C. MANALILI as Heir
or Representative of Deceased Defendants NOLI BELEN C.
MANALILI and REYNALDO MANALILI & BOARD OF
LIQUIDATORS, Respondents.
FACTS:
Reynaldo Manalili, predecessor-in-interest of respondent
Ronald C. Manalili, filed with the BOL an application to purchase a
4,608 square-meter parcel of land. The application was granted and
Manalili paid the downpayment of 10% of the purchase price as
required and thereafter declared the land for taxation purposes.
After the lapse of 9 years, Rodolfo Santos wrote an undated
letter to the BOL protesting Manalilis application. A formal
investigation was then conducted by BOL and eventually came out
with a report that Santos "was not actually occupying the lot and that
he only hired certain Abalahin and Lumaad to plant bananas and
coconut trees and maintain a vegetable garden thereon presumably to
establish a bona-fide occupancy over the lot." Accordingly, BOL
recommended the dismissal of Santos' protest and the approval of the
sale of Manalili.
Santos filed a complaint for reconveyance and damages. Trial
court adjudged against Santos ordering him to immediately vacate
the land and deliver the possession to Manalili.
CA upheld the findings of the BOL land examiner that Santos
protest was unfounded and was only meant to disturb the sale of the
subject land to respondent Manalili. To the appellate court, the BOLs
findings were duly supported by evidence, as in fact the sale of the
land to Manalili was approved by no less than the Office of the
President. Hence, they filed a petition for review on certiorari on the
SC.
Santos submits that he has clearly established a better right of
possession over the subject property. Per his testimony, the land in
dispute was originally occupied by one Col. Agsalud in 1956 up to
1959. Later, Col. Agsalud transferred his rights in favor of one Ernesto
Abalahin who continuously occupied the land and from whom Santos
acquired the property sometime in February 1969. He then
introduced various improvements thereon and continuously occupied
the same up to the present.
ISSUE:
106

Who has better right of possession over the lot in question?


RULING:
The Manalilis had a better right of possession thereto.
Preponderant evidence of respondent have sufficiently established
that as early as 1970, Reynaldo Manalili, respondents predecessor-ininterest, had already filed an Affidavit of Occupancy with the BOL,
the government agency tasked to administer it; that the Manalilis
administered the land before they left for Manila in 1972; that after
they moved to Manila they appointed an administrator to oversee the
land and the improvements and crops they have planted thereon,
such as bananas and coconut trees; and that the Manalilis have been
paying the real estate taxes for the subject land even before the sale
thereof to them.
Possession may be exercised in ones own name or in that of
another. It is not necessary that the owner or holder of the thing
exercise personally the rights of possession. Rights of possession may
be exercised through agents. So, the circumstances that after the sale,
the Manalilis resided in Manila and Pangasinan are of no moment.
Santos' claim of having bought the land from a certain Ernesto
Abalahin who, in turn, bought it from one Col. Agsalud, allegedly a
guerrilla veteran who occupied the lot from 1956 to 1959, is without
basis. No proof has been presented as to the alleged title or the
transfer of any rights. The supposed Deed of Absolute Sale between
petitioner and Ernesto Abalahin does not even sufficiently identify
the lot which was the subject of the sale and, worse, that same deed is
not notarized and is unregistered. A sale of a piece of land appearing
in a private deed cannot be considered binding on third persons if it is
not embodied in a public instrument and recorded in the Registry of
Deeds.
Santos' evidence do not also support his allegation that the
BOLs sale of the land to Reynaldo Manalili was fraudulent. It is wellsettled that fraud must be established by clear and convincing
evidence. Santos failed in this venture.
It is a settled rule in this jurisdiction that factual findings of an
administrative agency deserve utmost respect, more so, when, as
here, they are supported by substantial evidence, albeit such evidence
may not be overwhelming or merely preponderant.
G.R. No. 164823 August 31, 2005
107

MARIA CARLOS, represented by TERESITA CARLOS VICTORIA,


Petitioners, vs. REPUBLIC OF THE PHILIPPINES, Respondent.
FACTS:
Maria Carlos, represented by her daughter, Teresita Carlos
Victoria, filed an application for registration and confirmation of title
over a parcel of land with an area of 3,975 square meters located at
Pusawan, Ususan, Taguig, Metro Manila. Petitioner alleged, among
others, that she is the owner of said parcel of land which she openly,
exclusively and notoriously possessed and occupied since July 12,
1945 or earlier. Petitioner further claimed that she has been in
possession of the subject land in the concept of an owner; that her
possession has been peaceful, public, uninterrupted and continuous
since 1948 or earlier; and tacking her possession with that of her
predecessors-in-interest, petitioner has been in possession of the land
for more than 50 years.
Upon the demise of Maria Carlos, Victoria took possession of
the property with the consent of her brothers and sisters. However,
they have agreed to undertake the titling of the property and
promised to deliver the certificate of title to Ususan Development
Corporation which bought the property from Maria Carlos. Victoria
admitted that her mother had sold the land to Ususan Development
Corporation in 1996 but failed to deliver the title.
The Republic of the Philippines, represented by the Director of
Lands, filed an opposition to petitioners application. The trial court
granted her application. On appeal, the Court of Appeals reversed and
set aside the decision of the trial court.
CA noted that at the time she filed her application for
registration of title, the applicant was no longer in possession and
occupation of the land in question since the applicants mother and
predecessor-in-interest sold the subject land to Ususan Development
Corporation.
ISSUE:
WON Victoria still has the possession in the concept of an
owner when she filed an application for registration.

RULING:
108

SC affirmed the ruling of the appellate court. Applicants for


confirmation of imperfect title must prove the following: (a) that the
land forms part of the disposable and alienable agricultural lands of
the public domain; and (b) that they have been in open, continuous,
exclusive, and notorious possession and occupation of the same
under a bona fide claim of ownership either since time immemorial or
since June 12, 1945.
The applicant must show that he is in actual possession of the
property at the time of the application. Actual possession of a land
consists in the manifestation of acts of dominion over it of such a
nature as a party would naturally exercise over his own property.
It is clear in the case at bar that the applicant, Maria Carlos, no
longer had possession of the property at the time of the application
for the issuance of a certificate of title. Victoria admitted during the
hearing that her mother had sold the property to Ususan
Development Corporation in 1996. They also presented as evidence
the deed of absolute sale executed by and between Maria Carlos and
Ususan Development Corporation. This contradicts petitioners claim
that she was in possession of the property at the time that she applied
for confirmation of title.
Even if it were true that it was petitioner who had actual
possession of the land at that time, such possession was no longer in
the concept of an owner. Possession may be had in one of two ways:
possession in the concept of an owner and possession of a holder. A
possessor in the concept of an owner may be the owner himself or one
who claims to be so. On the other hand, one who possesses as a mere
holder acknowledges in another a superior right which he believes to
be ownership, whether his belief be right or wrong.16 Petitioner
herein acknowledges the sale of the property to Ususan Development
Corporation in 1996 and in fact promised to deliver the certificate of
title to the corporation upon its obtention. Hence, it cannot be said
that her possession since 1996 was under a bona fide claim of
ownership. Under the law, only he who possesses the property under
a bona fide claim of ownership is entitled to confirmation of title.

G.R. No. 181812

June 8, 2011
109

FELICIANO GAITERO and NELIA GAITERO, Petitioners, vs.


GENEROSO ALMERIA and TERESITA ALMERIA, Respondents.
FACTS:
A land registration court issued an original certificate of title to
Rosario O. Tomagan (Tomagan) covering a 10,741 square-meter land,
designated as Lot 9960. Tomagan subdivided the lot awarded to her
into four: Lot 9960-A3; Lot 9960-B; Lot 9960-C4; and Lot 9960-D.
Tomagan waived her rights over Lots 9960-A and 9960-C in favor of
petitioner Feliciano Gaitero (Gaitero) and Lot 9960-B in favor of
Barangay Ysulat, Tobias Fornier. She retained Lot 9960-D.
Lot 9960-A that went to Gaitero adjoined Lot 9964 which
belonged to respondent spouses Almerias and was covered by OCT P14556. In June 2000, the Almerias commissioned a relocation survey
of their lot and were astonished to find that Gaitero, who owned
adjoining Lot 9960-A, intruded into their lot by as much as 737 sq m
(the disputed area).
Apparently to settle the dispute, the Almerias waived their
rights over a 158 sq m portion of the disputed area in Gaiteros favor
but maintained their claim over the remaining 579 sq m.
Subsequently, however, Gaitero filed an affidavit of adverse claim on
the Almerias title over the remaining 579 sq m.
Gaitero claimed that he was the registered owner of Lot 9960-A,
that he inherited the same from his mother, Maria Obay, who in turn
inherited it from her father, Bonifacio Obay; that before the cadastral
survey, Lot 9960-A was erroneously lumped with Lot 9960 in
Tomagans name; that, acknowledging the mistake, Tomagan
subdivided Lot 9960 into four lots and waived her rights over Lots
9960-A and 9960-C in Gaiteros favor; that the Almerias claimed a
portion of Lot 9960-A by virtue of a relocation survey and fenced it
close to Gaiteros house, obstructing the latters passageway; and that
while the Almerias returned 158 sq m of the disputed portion, they
refused to return to him the remaining 579 sq m.
Almerias alleged that they bought Lot 9964 in 1985 by virtue of
an Extra-Judicial Settlement of Estate and Sale; that it was Gaitero
who unlawfully encroached on the 737 sq m portion of Lot 9964; and
that, while they had waived a portion of the disputed area, Gaiteros
incessant claim to the remaining 579 sq m prompted them to cancel
their previous waiver of the 158 sq m. The Almerias prayed for the
dismissal of the complaint and the award of damages in their favor.
110

MTC rendered a decision dismissing the complaint and ordering


Gaitero to pay the Almerias damages and held that the Almerias were
entitled to the possession of the disputed area considering that it is
included in the technical description of their registered title. Further,
the MCTC held that Gaitero acknowledged the true boundaries of
9960-A when Lot 9960 was subdivided in 1993. Indeed, the
subdivision plan clearly shows that the disputed area is excluded from
9960-A.
RTC reversed the decision of the MCTC due to laches. Although
it recognizes the Almerias as rightful owners, laches prevented them
from asserting their right over the same since it took them 15 years
before they do so.
CA reversed the decision of the RTC and reinstated that of the
MCTC. The CA held that the Almerias owned the disputed area since,
between a registered title and a verbal claim of ownership, the former
must prevail. The CA did not consider the Almerias in laches since no
one had lodge a claim of ownership against their title to the disputed
property.
ISSUE:
Whether or not the CA erred in holding that the Almerias are
entitled to th e possession of the disputed area as against Gaitero?
RULING:
Possession is an essential attribute of ownership. Necessarily,
whoever owns the property has the right to possess it. Here, between
the Almerias registered title of ownership and Gaiteros verbal claim
to the same, the formers title is far superior.
As the MCTC, the RTC, and the CA found, the disputed area
forms part of the Almerias registered title. Upon examination, this
fact is also confirmed by the subdivision plan which partitioned
Tomagans original Lot 9960. Since Gaitero was unable to prove that
fraud attended the titling of the disputed area, the Almerias right
over the same became indefeasible and incontrovertible a year from
registration.
Gaitero's cannot collaterally attack registered titles based on his
alleged continuous possession and claim of ownership of the disputed
area. Gaiteros action before the MCTC is one for recovery of
111

possession of the disputed area. . A registered title cannot be


impugned, altered, changed, modified, enlarged, or diminished,
except in a direct proceeding permitted by law. Otherwise, reliance on
registered titles would be lost. Gaiteros action is prohibited by law
and should be dismissed.
Gaiteros theory of laches cannot vest on him the ownership of
the disputed area. To begin with, laches is a consideration in equity
and therefore, anyone who invokes it must come to court with clean
hands, for he who has done inequity shall not have equity. Here,
Gaiteros claim of laches against the Almerias can be hurled against
him. When the lot that the Almerias acquired (Lot 9964) was
registered in 1979, Gaitero had constructive, if not actual, notice that
the cadastral survey included the disputed area as part of the land
that Leon Asenjo claimed. Yet, neither Gaitero nor his mother
complained or objected to such inclusion.
When Gaitero saw the subdivision plan covering Tomagans
original Lot 9960 in 1993, it showed that the disputed area fell
outside the boundaries of Lot 9960-A which he claimed. Still, Gaitero
did nothing to correct the alleged mistake. He is by his inaction
clearly estopped from claiming ownership of the disputed area. He
cannot avail himself of the law of equity.
SC dismissed the petition and affirmed the decision of the CA.

G.R. No. 152423

December 15, 2010

112

SPOUSES MARCOS R. ESMAQUEL and VICTORIA SORDEVILLA,


Petitioners, vs. MARIA COPRADA, Respondent.
FACTS:
Sps. Marcos Esmaquel and Victoria Sordevilla (Victoria) filed
an ejectment case against respondent Maria V. Coprada. In 1945,
Coprada was able to persuade the spouses to allow her and her family
to use and occupy the land for their residence, under the condition
that they will vacate the premises should petitioners need to use the
same. Respondent and her family were allowed to construct their
residential house. Since then, the petitioners never made an attempt
to drive them away out of pity, knowing that respondent and her eight
children have no other place to live in. Also, respondent and her
family have been occupying the subject premises free of rent,
including payment of realty taxes.
When Coprada's present circumstances have completely
improved, the spouses verbally demanded that they vacate the subject
land but the former refused. Thus, petitioners were forced to send a
demand letter dated August 22, 1996, giving respondent until
November 30, 1996 to vacate the subject premises. Respondent still
ignored said demand; hence, the spouses lodged an ejectment case
against Coprada.
Coprada admitted that petitioners are the registered owners of
the subject land. However, she averred that in 1945, it was Victoria
Sordevilla's mother and not the petitioners who gave permission to
her late husband Brigido Coprada to use the subject lot.
Respondent alleged that sometime in the early 1960's,
petitioner Victoria offered the said lot for sale for P2,000.00 to
respondent, who readily agreed. The purchase price was paid in
installments and was fully paid in 1962. Due to their close
relationship, the agreement was never reduced to writing.
Coprada further maintained that since the execution of the oral
sale of the subject lot, she has been the one paying the realty taxes
due on the property. After the sale, respondent built on the subject
land a semi-concrete structure. Respondent stated that petitioners'
claim is barred by laches. Even granting, without admitting, that
respondent's claim of ownership over the property is improper
because petitioners are the registered owners thereof, respondent
argued that she is a builder in good faith, because she was able to

113

build the structure on the subject lot with the prior permission of the
owner.
MCTC rendered judgment dismissing the complaint. It held
that laches had already set in which prevented petitioners from
questioning the validity of the purported sale between Victoria and
Maria.
On appeal, the RTC reversed the MCTCs judgment. The RTC
ruled that respondent's occupation of the subject property was by
virtue of petitioners' tolerance and permission. Hence, respondent is
bound by an implied promise that she will vacate the property upon
demand. respondent also failed to prove the alleged oral sale and that
petitioners have adequately proven that they are entitled to the
possession of the subject land as registered owners thereof.
The CA granted respondent's petition, reversed the Decision of
the RTC and affirmed in toto the Decision of the MCTC. Hence, the
instant petition with the SC.
ISSUE:
Whether or not petitioners have a valid ground to evict
respondent from the subject property.
RULING:
Court held that respondent failed to present evidence to
substantiate her allegation that a portion of the land was sold to her
in 1962. In fact, when petitioners sent a letter to the respondent,
demanding her to vacate the subject property, the respondent, in
reply to the said letter, never mentioned that she purchased the
subject land in 1962. If the sale really took place, the respondent
should have immediately and categorically claimed that in her letter
response. As against the respondent's unproven claim that she
acquired a portion of the property from the petitioners by virtue of an
oral sale, the Torrens title of petitioners must prevail. Moreover, the
age-old rule is that the person who has a Torrens title over a land is
entitled to possession thereof.
The validity of petitioners' certificate of title cannot be attacked
by respondent in this case for ejectment. A certificate of title shall not
be subject to collateral attack. It cannot be altered, modified or
canceled, except in a direct proceeding for that purpose in accordance
with law. Whether or not the respondent has the right to claim
114

ownership over the property is beyond the power of the trial court to
determine in an action for unlawful detainer.
It is settled that a Torrens Certificate of title is indefeasible and
binding upon the whole world unless and until it has been nullified by
a court of competent jurisdiction. As the registered owner, petitioner
had a right to the possession of the property, which is one of the
attributes of ownership.
The doctrine that a registered owner may lose his right to
recover its possession by reason of laches is not applicable here.
Laches is the failure or neglect, for an unreasonable and unexplained
length of time, to do that which, by exercising due diligence, could or
should have been done earlier.
Respondent first acquired possession of the subject lot by mere
tolerance. From 1945 until the filing of the complaint for ejectment in
1997, the nature of that possession has never changed. Petitioners
allowed the respondent to possess the property with the knowledge
that the respondent will vacate the same upon demand. Hence, until
such demand to vacate was communicated by the petitioners to the
respondent, petitioners are not required to do any act to recover the
subject land, precisely because they knew of the nature of the
respondent's possession, i.e., possession by mere tolerance. Thus, it
cannot be said that petitioners are guilty of failure or neglect to assert
a right within a reasonable time.
In contrast, respondent, who is claiming that a portion of the
property was sold to her in 1962, has herself failed within a long
period of time to have that portion transferred in her name. As
registered owners of the lots in question, the private respondents
have a right to eject any person illegally occupying their property.
This right is imprescriptible.
Since respondent's occupation of the subject property was by
mere tolerance, she has no right to retain its possession under Article
448 of the Civil Code. She is aware that her tolerated possession may
be terminated any time and she cannot be considered as builder in
good faith.

115

G.R. No. 157044 October 5, 2005


RODOLFO V. ROSALES, (represented by his heirs, Rodolfo, Jr.,
Romeo Allan, Lillian Rhodora, Roy Victor, Roger Lyle and Alexander
Nicolai, all surnamed Rosales) and LILY ROSQUETA-ROSALES,
Petitioners vs. MIGUEL CASTELLTORT, JUDITH CASTELLTORT,
and LINA LOPEZ-VILLEGAS, assisted by her Attorney-in-Fact, Rene
Villegas, Respondents.
FACTS:
Spouses-petitioners Rodolfo V. Rosales and Lily RosquetaRosales are the registered owners of a parcel of land with an area of
approximately 315 square meters designated as Lot17. Petitioners
discovered that a house was being constructed on their lot, without
their knowledge and consent, by respondent Miguel Castelltort.
Castelltort and his wife Judith had purchased a lot, Lot 16 of the
same Subdivision Plan, from respondent Lina Lopez-Villegas (Lina)
through her son-attorney-in-fact Rene Villegas (Villegas) but that
after a survey thereof by geodetic engineer Augusto Rivera, he
pointed to Lot 17 as the Lot 16 the Castelltorts purchased.
Negotiations for the settlement of the case thus began, with
Villegas offering a larger lot near petitioners lot in the same
subdivision as a replacement thereof. In the alternative, Villegas
proposed to pay the purchase price of petitioners lot with legal
interest. Both proposals were rejected.
To the complaint, the Castelltorts claimed in their Answer with
Counterclaim that they were builders in good faith.
Lina alleged that the Castelltorts acted in good faith in
constructing the house on petitioners lot as they in fact consulted her
before commencing any construction thereon, they having relied on
the technical description of the lot sold to them, Lot 16, which was
verified by her officially designated geodetic engineer.
RTC ruled out good faith. CA granted the appeal and set aside
the RTC decision.
ISSUE:
Whether appellant Miguel is a builder in good faith.
116

RULING:
Miguel relied on the title which the intervenor showed to him
which, significantly, has no annotation that would otherwise show a
prior adverse claim. Thus, as far as appellant Miguel is concerned, his
title over the subject lot, as well as the title of the intervenor thereto,
is clean and untainted by an adverse claim or other irregularities.
Miguels failure to secure a building permit from the Municipal
Engineers Office on their construction on Lot 17 does not impinge on
his good faith. Miguel, in good faith, built the house on appellees
land without knowledge of an adverse claim or any other
irregularities that might cast a doubt as to the veracity of the
assurance given to him by the intervenor. Miguel cannot be faulted
for having relied on the expertise of the land surveyor who is more
equipped and experienced in the field of land surveying.
The appellants house erroneously encroached on the property
of the appellees due to a mistake in the placement of stone
monuments as indicated in the survey plan, which error is directly
attributable to the fault of the geodetic engineer who conducted the
same. This fact alone negates bad faith on the part of appellant
Miguel.
A builder in good faith is one who builds with the belief that the
land he is building on is his or that by some title one has the right to
build thereon, and is ignorant of any defect or flaw in his title.
Article 527 of the Civil Code provides that good faith is always
presumed, and upon him who alleges bad faith on the part of a
possessor rests the burden of proof. The certified true copy bore no
annotation indicating any prior adverse claim on Lot 16. The records
indicate that at the time Castelltort began constructing his house on
petitioners lot, he believed that it was the Lot 16 he bought and
delivered to him by Villegas.
In the case at bar, Castelltorts good faith ceased on August 21,
1995 when petitioners personally apprised him of their title over the
questioned lot. As held by the CA, should petitioners then opt to
appropriate the house, they should only be made to pay for that part
of the improvement built by Castelltort on the questioned property at
the time good faith still existed on his part or until August 21, 1995.

117

G.R. No. 158929

August 3, 2010

ROSARIO P. TAN, Petitioner, vs. ARTEMIO G. RAMIREZ, MOISES


G. RAMIREZ, RODRIGO G. RAMIREZ, DOMINGO G. RAMIREZ,
and MODESTA RAMIREZ ANDRADE, Respondents.
FACTS:
Rosario Tan filed a case for recovery of ownership of a parcel of
land against respondents Ramirez, et. al. Rosario alleges that her
great-grandfather, Catalino, acquired said property under a Tax
Declaration in 1915. The respondents trace their ownership of said
land through a compromise agreement and deed of absolute sale from
Gavino who has been cultivating the said property. Gavino is the
husband of one of the daughters of Catalino.
The appellate court ruled that the respondents are the true
owners of the property, there being acquisitive prescription. The deed
of absolute sale and compromise agreement constitute just title, thus,
their 24 years of possession was more than enough to meet the 10year possession required by law.
ISSUE:
Whether or not the compromise agreement and deed of sale
were enough bases for good faith and just title, thus allowing ordinary
acquisitive prescription to take place.
RULING:
NO. A compromise agreement is aimed to put an end to
litigation; it does not create or transmit ownership rights over a
property. A deed of sale could not also be used as a basis for showing
good faith in cases of acquisitive prescription. Good faith, or the want
of it, can be ascertained only from the acts of the one claiming it, as it
is a condition of mind that can only be judged by actual or fancied
token or signs.
The respondents in the case at bar were aware of the pending
litigation over the property, as evidenced by the compromise
agreement. Thus, they cannot be considered to be in good faith in
acquiring said property by prescription. The respondents did not
acquire the property through ordinary acquisitive prescription since
they were not in good faith. Likewise, they did not acquire such by
extraordinary acquisitive prescription for holding it only for 24 years,
short of the 30-year period mandated by law.

118

G.R. No. 172217

September 18, 2009

SPOUSES LYDIA FLORES-CRUZ and REYNALDO I. CRUZ,


Petitioners, vs. SPOUSES LEONARDO and ILUMINADA GOLICRUZ, SPOUSES RICO and FELIZA DE LA CRUZ, SPOUSES BOY
and LANI DE LA CRUZ, ZENAIDA A. JACINTO and ROGELIO DE
LOS SANTOS, Respondents.
FACTS:
Petitioner spouses Lydia Flores-Cruz and Reynaldo I. Cruz
purchased a 5,209-sq. m. lot situated in Pulong Yantok, Angat,
Bulacan5 from Lydias siblings, namely, Teresita, Ramon and Daniel
(all surnamed Flores).
After the death of Lydia's father, petitioners found out that
respondent spouses Leonardo and Iluminada Goli-Cruz et al. were
occupying a section of the land. Initially, petitioner Lydia talked to
respondents and offered to sell them the portions they were
occupying but the talks failed as they could not agree on the price.
Respondents countered that their possession of the land ranged
from 10 to 20 years. According to respondents, the property was
alienable public land.
Petitioners filed a complaint for recovery of possession of the
land in the RTC. RTC rendered a decision in favor of petitioners.
CA ruled that RTC had no jurisdiction over the action for
recovery of possession because petitioners had been dispossessed of
the property for less than a year.
ISSUE:
The issue for resolution is whether the RTC had jurisdiction
over this case.
RULING:
That the nature of the action on which depends the question
of whether a suit is within the jurisdiction of the court is
determined solely by the allegations in the complaint and the law at
the time the action was commenced.

119

SC agree with the CA that considering that petitioners claimed that


respondents were possessors of the property by mere tolerance only
and the complaint had been initiated less than a year from the
demand to vacate, the proper remedy was an action for unlawful
detainer which should have been filed in the MTC.
Based on the allegations in petitioners complaint, it is apparent
that such is a complaint for unlawful detainer based on possession by
tolerance of the owner. It is a settled rule that in order to justify such
an action, the owners permission or tolerance must be present at the
beginning of the possession. Such jurisdictional facts are present
here.
There is another reason why petitioners complaint was not a
proper action for recovery of possession cognizable by the RTC. It is
no longer true that all cases of recovery of possession or accion
publiciana lie with the RTC regardless of the value of the property.
When the case was filed in 2001, Congress had already
approved Republic Act No. 769122 which expanded the MTCs
jurisdiction to include other actions involving title to or possession of
real property (accion publiciana and reinvindicatoria) where the
assessed value of the property does not exceed P20,000 (or P50,000,
for actions filed in Metro Manila).
Since petitioners complaint made out a case for unlawful
detainer which should have been filed in the MTC and it contained no
allegation on the assessed value of the subject property, the RTC
seriously erred in proceeding with the case. The proceedings before a
court without jurisdiction, including its decision, are null and void.34
It follows that the CA was correct in dismissing the case.

120

G.R. No. 182673

October 5, 2009

AQUALAB PHILIPPINES,
MARCELINO PAGOBO

INC.,

Petitioner,

vs.

HEIRS

OF

FACTS:
Subject of the complaint initiated by respondents are Lots 6727Q and 6727-Y. Lot 6727-Q and Lot 6727-Y used to form part of Lot
6727 owned by respondents great grandfather, Juan Pagobo
containing an area of 127,436 square meters. Lot 6727 was once
covered by Juan Pagobos homestead application. Shortly after
homestead patent was issued, the lots were subsequently sold to
Tarcela de Espina; to Rene Espina; to Anthony Gaw Kache; and
finally to Aqualab. TCT was issued.
Respondents held that they are absolute and legal owners and
rightful possessors of Lot [no.] 6727-Q and Lot no. 6727-Y. These are
ancestral lands which are part of a bigger parcel of land. Ownership
and Possession by plaintiffs predecessors-in-interest, and plaintiffs
herein, respectively, over the said land, have been peaceful,
continuous [sic] open, public and adverse, since the year 1936 or even
earlier. Their peaceful possession was disturbed only in 1991.
RTC granted Aqualabs motion and dismissed respondents
complaint. In granting Aqualabs motion to dismiss, the trial court
ruled that prescription has set in. Moreover, the trial court held that
Aqualab is an innocent purchaser for value and, thus, its rights are
protected by law. Finally, it concluded that legal redemption or
reconveyance was no longer available to respondents.
CA reversed the decision of the RTC declaring the sale of subject
as null and void.
The CA resolved the following issues: (1) the propriety of the
dismissal of the complaint by the RTC; and, (2) whether respondents
have the right to redeem subject lots. The CA ruled that the trial court
erred in dismissing the complaint as the sale of subject lots to Tarcela
de Espina was void, thus making the subsequent conveyances
ineffective and no titles were validly transferred. Moreover, it ruled
that Aqualab is not an innocent purchaser for value, and held that
respondents, as heirs of the homestead grantee, never lost their valid
title to the subject lots.

121

ISSUE:
Whether the action of respondents is barred by prescription;
Whether Aqualab is an innocent purchaser for value.
RULING:
Respondents aver that they are the absolute and lawful owners
of subject properties over which they have had actual possession since
1936 or earlier until sometime in 1991, when Aqualab disturbed such
possession. While the records show that respondents did not have in
their names the certificate of titles over subject lots, the factual
assertion of open, peaceful, public, and adverse possession is
hypothetically admitted by Aqualab.
Moreover, respondents allege that the conveyances of subject
lots were fraudulently made in violation of the restrictions on
alienation of homesteads under CA 141, and that said conveyances
were made without their knowledge and, thus, asserting their right to
redeem the subject properties in line with the policy of CA 141 that
the homestead should remain with the grantee and his family. The
alleged fraudulent conveyances were likewise hypothetically admitted
by Aqualab.
Respondents have duly averred continuous possession until
1991 when such was allegedly disturbed by Aqualab. Being in
possession of the subject lotshypothetically admitted by Aqualab
respondents right to reconveyance or annulment of title has not
prescribed or is not time-barred.
Verily, an action for annulment of title or reconveyance based
on fraud is imprescriptible where the plaintiff is in possession of the
property subject of the acts.
A buyer of real property that is in the possession of a person
other than the seller must be wary, and a buyer who does not
investigate the rights of the one in possession can hardly be regarded
as a buyer in good faith.
Having hypothetically admitted respondents possession of
subject lots, Aqualab cannot be considered, in the context of its
motion to dismiss, to be an innocent purchaser for value or a
purchaser in good faith. Moreover, the defense of indefeasibility of a
Torrens title does not extend to a transferee who takes it with notice
of a flaw in the title of his transferor.
122

G.R. No. 178906

February 18, 2009

ELVIRA T. ARANGOTE, petitioner, vs. SPS. MARTIN MAGLUNOB


and LOURDES S. MAGLUNOB, and ROMEO SALIDO, Respondents.
FACTS:
Elvira T. Arangote, herein petitioner married to Ray Mars E.
Arangote, is the registered owner of the subject property.
Respondents Martin (Martin II) and Romeo are first cousins and the
grandnephews of Esperanza Maglunob-Dailisan (Esperanza), from
whom petitioner acquired the subject property.
The Petition stems from a complaint filed by petitioner and her
husband against the respondents for Quieting of Title, Declaration of
Ownership and Possession, Damages with Preliminary Injunction,
and Issuance of Temporary Restraining Order before the MCTC. The
complaint alleged that Esperanza inherited the subject property from
her uncle Victorino Sorrosa. Esperanza executed a Last Will and
Testament bequeathing the subject property to petitioner and her
husband, but it was never probated.
On 9 June 1986, Esperanza executed another document, an
Affidavit, in which she renounced, relinquished, waived and
quitclaimed all her rights, share, interest and participation
whatsoever in the subject property in favor of petitioner and her
husband. In 1989, petitioner and her husband constructed a house on
the subject property.
Respondents averred that they co-owned the subject property
with Esperanza. The subject property was co-owned by Esperanza,
respondent Martin II (together with his wife Lourdes), and
respondent Romeo, each holding a one-third pro-indiviso share
therein. Thus, Esperanza could not validly waive her rights and
interest over the entire subject property in favor of the petitioner.
Respondents also asserted in their Counterclaim that petitioner
and her husband, by means of fraud, undue influence and deceit were
able to make Esperanza, who was already old and illiterate, affix her
thumbmark to the Affidavit dated 9 June 1986, wherein she
renounced all her rights and interest over the subject property in
favor of petitioner and her husband. Respondents thus prayed that
the OCT issued in petitioners name be declared null and void insofar
as their two-thirds shares are concerned.

123

The MCTC rendered a decision declaring petitioner and her


husband as the true and lawful owners of the subject property. RTC
reversed the MCTC decision. The Court of Appeals likewise rendered
a Decision denying the Petition for Review of petitioner and her
husband and affirming the RTC Decision.
ISSUE:
Whether or not the petitioners are builders in good faith.
RULING:
Court held that the findings of fact of the MCTC as regards the
origin of the subject property are in conflict with the findings of fact
of both the RTC and the Court of Appeals. SC affirms the findings of
both the RTC and the Court of Appeals as regards the origin of the
subject property and the fact that respondents, with their grand aunt
Esperanza, were co-heirs and co-owners of the subject property. It is
clear from the records that the subject property was not Esperanzas
exclusive share, but also that of the other heirs of her father, Martin I.
Esperanza expressly affixed her thumbmark to the Deed of
Extrajudicial Settlement of July 1981 not only for herself, but also on
behalf of the other heirs of Martin I.
Further, it bears emphasis that the Partition Agreement was
executed by and among the son, grandsons, granddaughters and
cousins of Victorino. Esperanza was neither the granddaughter nor
the cousin of Victorino, as she was only Victorinos grandniece. The
cousin of Victorino is Martin I, Esperanzas father. In effect,
therefore, the subject property allotted to Esperanza in the Partition
Agreement was not her exclusive share, as she holds the same for and
on behalf of the other heirs of Martin I, who was already deceased at
the time the Partition Agreement was made.
Esperanzas Affidavit is, in fact, a Donation. Esperanzas real
intent in executing the said Affidavit was to donate her share in the
subject property to petitioner and her husband.
Art. 749 of the Civil Code provides, "In order that the donation
of an immovable may be valid, it must be made in a public document,
specifying therein the property donated and the value of the charges
which the donee must satisfy."
Three requisites for the validity of a simple donation of a real
property: (1) it must be made in a public instrument; (2) it must be
124

accepted, which acceptance may be made either in the same Deed of


Donation or in a separate public instrument; and (3) if the acceptance
is made in a separate instrument, the donor must be notified in an
authentic form, and the same must be noted in both instruments.
In the present case, the said Affidavit, which is tantamount to a
Deed of Donation, met the first requisite, as it was notarized; thus, it
became a public instrument. Nevertheless, it failed to meet the
aforesaid second and third requisites.
Possession in good faith ceases from the moment defects in the
title are made known to the possessor by extraneous evidence or by a
suit for recovery of the property by the true owner. Every possessor in
good faith becomes a possessor in bad faith from the moment he
becomes aware that what he believed to be true is not so.
In the present case, when respondents came to know that an
OCT over the subject property was issued and registered in
petitioners name, respondents brought a complaint before the Lupon
of Barangay Maloco, Ibajay, Aklan, challenging the title of petitioner
to the subject property on the basis that said property constitutes the
inheritance of respondent, together with their grandaunt Esperanza,
so Esperanza had no authority to relinquish the entire subject
property to petitioner. From that moment, the good faith of the
petitioner had ceased.
In this case, the subject property waived and quitclaimed by
Esperanza to the petitioner and her husband in the Affidavit was only
covered by a tax declaration in the name of Esperanza. Petitioner did
not even bother to look into the origin of the subject property and to
probe into the right of Esperanza to relinquish the same. Thus, when
petitioner and her husband built a house thereon in 1989 they cannot
be considered to have acted in good faith as they were fully aware that
when Esperanza executed an Affidavit relinquishing in their favor the
subject property the only proof of Esperanzas ownership over the
same was a mere tax declaration. Settled is the rule that a tax
declaration does not prove ownership. It is merely an indicium of a
claim of ownership. Payment of taxes is not proof of ownership; it is,
at best, an indicium of possession in the concept of ownership.
Hence, the petitioner is not entitled to the rights under Article 448
and 546 as the petitioner is not a builder and possessor in good faith.
SC affirmed the RTC decision and declared the respondents as the
lawful owners and possessors of the subject property.
125

G.R. No. 162886

August 11, 2008

HEIRS OF THE DECEASED SPOUSES VICENTE S. ARCILLA and


JOSEFA ASUNCION ARCILLA, namely: Aida Arcilla Alandan, Rene
A. Arcilla, Oscar A. Arcilla, Sarah A. Arcilla, and Nora A. Arcilla, now
deceased and substituted by her son Sharmy Arcilla, represented by
their attorney-in-fact, SARAH A. ARCILLA, petitioners, vs. MA.
LOURDES A. TEODORO, respondent.
FACTS:
Ma. Lourdes A. Teodoro (respondent) initially filed with the
RTC of Virac, Catanduanes an application for land registration of two
parcels of land located at Barangay San Pedro, Virac, Catanduanes.
Respondent alleged that, with the exception of the commercial
building constructed thereon, she purchased the subject lots from her
father, Pacifico Arcilla (Pacifico), as shown by a Deed of Sale and that,
prior thereto, Pacifico acquired the said lots by virtue of the partition
of the estate of his father, Jose Arcilla evidenced by a document
entitled Extrajudicial Settlement of Estate. Respondent also
presented as evidence an Affidavit of Quit-Claim in favor of Pacifico,
executed by herein petitioners as Heirs of Vicente Arcilla (Vicente),
brother of Pacifico.
In their opposition, petitioners contended that they are the
owners pro-indiviso of the subject lots including the building and
other improvements constructed thereon by virtue of inheritance
from their deceased parents, spouses Vicente and Josefa Arcilla;
contrary to the claim of respondent, the lots in question were owned
by their father, Vicente, having purchased the same from a certain
Manuel Sarmiento sometime in 1917; Vicente's ownership is
evidenced by several tax declarations attached to the record;
petitioners and their predecessors-in-interest had been in possession
of the subject lots since 1906.
MTC rendered a decision declaring Lourdes A. Teodoro as the
valid owner of the subject property. RTC dismissed the appeal of
petitioners for lack of merit. CA dismissed the same.
ISSUE:
Whether or not the petitioners have right of possession and
ownership to subject property.

126

RULING:
NO. The Court agrees with the CA in its finding that petitioners
failed to present any substantial evidence, such as a deed of sale, to
prove their claim that their predecessor, Vicente Arcilla, bought the
disputed property from Sarmiento. Petitioners were only able to
present tax declarations in Vicente's name to prove their allegation
that Vicente became the owner of the subject property. The tax
declarations presented in evidence by petitioners are not supported
by any other substantial proofs.
Tax declarations do not prove ownership but are at best an
indicium of claims of ownership. Payment of taxes is not proof of
ownership, any more than indicating possession in the concept of an
owner. Neither a tax receipt nor a declaration of ownership for
taxation purposes is evidence of ownership or of the right to possess
realty when not supported by other effective proofs.
In addition, the Court agrees with the CA when it held that if
Vicente, in fact, owned the disputed properties, his widow, Josefa,
would not have agreed to include said lots among those partitioned in
the Extrajudicial Settlement of the Estate of Jose.
On the other hand, respondent's claim of ownership is not only
backed up by tax declarations but also by other pieces of evidence
such as the subject Extrajudicial Settlement, Affidavit of Quitclaim,
and Deed of Sale.

127

G.R. No. 146550

March 17, 2006

FELIPA DELFIN, GINA MAALAT, SHIRLEY TAMAYO, RECIO


DAOS, and ROBERTO DELFIN, Petitioners, vs. PRESENTACION
D. BILLONES, ROSARIO D. DEMONARCA (accompanied by
husband Pedro and Demonarca), WENEFREDO DEGALA
(representing Pedro Degala), RAMON DELA CRUZ (representing his
deceased wife Maria Daradar dela Cruz), TERESITA DALIVA
DEVIENTE (daughter of Esperanza Daradar Daliva), and JOLLY
DATAR (representing his deceased mother Trinidad D. Datar) and
the COURT OF APPEALS, Respondents.
FACTS:
A Deed of Absolute Sale over Lot No. 213 of the Cadastral
Survey of Panitan, Capiz, was executed by Teresa Daos, Esperanza
Daradar, Estrella Daradar and Maria Daradar, with the marital
consent of Cipriano Degala, husband of Teresa Daos, in favor of the
spouses Rodolfo Delfin and Felipa Belo (spouses Delfin). The
document, so it appears, bore the signatures of Esperanza and
Estrella, as well as the thumb marks of Teresa, Maria, and Cipriano,
and was acknowledged before a notary public. The spouses Delfin
registered the sale with the ROD and a new title was issued under
their name.
Respondents, claiming to be the heirs of the former owners of
Lots No. 213 and No. 3414, filed an action for annulment,
reconveyance, recovery of ownership and possession and damages.8
According to them, it was only in 1989 when they discovered that
Teresa Daos, sick and in dire need of money, was constrained to
mortgage the one-half (1/2) portion of Lot No. 3414 to the spouses
Delfin for P300.00 sometime in 1965.
Taking advantage of her condition, the spouses Delfin made her
sign a document purporting to be a mortgage, but which turned out to
be an extrajudicial partition with deed of absolute sale. As to Lot No.
213, respondents averred that the Deed of Sale covering the property
was fictitious and the signatures and thumb marks contained therein
were all forged because three (3) of the signatories therein died before
the alleged sale in 1960, namely: Estrella Daradar, who died in 1934,
and Esperanza Daradar and Cipriano Degala, who both died in 1946.
Petitioners alleged that respondents action was already barred
by prescription and laches. Further, they argued that the spouses
Delfin, as well as the subsequent owners of the subject properties, are
innocent purchasers for value and in good faith, whose titles to the
128

lots at the time of the purchase were all clean and free from liens and
encumbrances.
Giving credence to the claims of petitioners, the trial court ruled
that respondents claim of ownership over the subject properties was
not established by a preponderance of evidence. Compared to
respondents verbal claims of ownership, the spouses Delfin were able
to prove that they bought the properties from the original owners.
The trial court held that the deeds of sale being duly executed notarial
and public documents; they enjoy the presumption of regularity
which can only be contradicted by clear and convincing evidence. In
addition, respondents claims based on fraud were barred by
prescription, having been filed more than four (4) years from the time
the instruments were registered with the Register of Deeds, and they
are estopped from annulling the documents by reason of laches, the
action having been filed 15 years after the deeds were registered. The
trial court also denied respondents claims for damages.
The CA reversed the decision of the trial court. The Court of
Appeals ruled that while an action for reconveyance based on implied
or constructive trust prescribes in ten (10) years from the date of the
issuance of the certificate of title over the property, such prescriptive
period does not apply if the person claiming to be the owner of the
property is in possession thereof, such as respondents in this case.
ISSUE:
Whether or not the petitioners have a right to possession on the
subject property.
RULING:
YES. When ones property is registered in anothers name
without the formers consent, an implied trust is created by law in
favor of the true owner. Implied trusts are those which, without being
expressed, are deducible from the nature of the transaction by
operation of law as matters of equity, independently of the particular
intention of the parties. Meanwhile, constructive trusts are created in
order to satisfy the demands of justice and prevent unjust
enrichment. They arise against one who, by fraud, duress or abuse of
confidence, obtains or holds the legal right to property which he
ought not, in equity and good conscience, to hold.
An action for reconveyance based upon an implied or
constructive trust prescribes in ten (10) years from the registration of
the deed or from the issuance of the title, registration being
129

constructive notice to all persons. However, an action for


reconveyance based on fraud is imprescriptible where the plaintiff is
in possession of the property subject of the acts.
Court held that respondents failed to prove that fraud attended
the sale of Lots No. 213 and No. 3414. A duly executed contract
carries with it the presumption of validity. The party who impugns its
regularity has the burden of proving its simulation. A notarized
document is executed to lend truth to the statements contained
therein and to the authenticity of the signatures. Notarized
documents enjoy the presumption of regularity which can be
overturned only by clear and convincing evidence. Hence, parties who
have the burden of proof must produce such quantum of evidence,
with plaintiffs having to rely on the strength of their own evidence,
not on the weakness of the defendants.

130

G. R. No. 147074

July 15, 2005

Spouses RODRIGO PADERES and SONIA PADERES , Petitioners, vs.


The Hon. COURT OF APPEALS,1 Hon. CARLOTA P. VALENZUELA,
in her capacity as the Liquidator of Banco Filipino Savings and
Mortgage Bank,2 Respondents.
FACTS:
Manila International Construction Corporation (MICC)
executed a real estate mortgage over 21 registered parcels of land
including the improvements thereon in favor of Banco Filipino
Savings and Mortgage Bank (Banco Filipino) in order to secure a
loan. The mortgage was registered with the Registry of Deeds of Pasay
City and annotated on the corresponding transfer certificates of title
(TCTs).
The 21 mortgaged properties included two lots covered by TCT
Nos. 610626 and 61078. Subsequently MICC sold the lot covered by
TCT No. 61078, together with the house thereon, to the petitioners in
the first case, the Paderes spouses. MICC sold the house built on the
lot covered by TCT No. 61062 to the petitioners in the second case,
the Bergado spouses. Neither sale was registered, however.
For failure of MICC to settle its obligations, Banco Filipino filed
a verified Petition for the extrajudicial foreclosure of MICCs
mortgage; at the auction sale of the foreclosed properties. Banco
Filipino was declared the highest bidder.
A writ of possession was granted by RTC, copies of which
addressed to MICC "and/or All persons claiming rights under them"
to voluntarily vacate the premises within 7 days from receipt thereof,
were served on petitioners. Instead of vacating the two lots, however,
petitioners filed separate petitions before the Court of Appeals,
docketed as C.A. G.R. Numbers 42470 and 42471 which were later
consolidated, assailing the validity of the Writ of Possession. On
appeal, the CA dismissed the consolidated petitions for lack of merit
and uphold the validity of the writ of possession.
Petitioners Motion for Reconsideration of the appellate courts
decision having been denied by Resolution of February 16, 2001, they
jointly come before this Court arguing that: (1) having purchased
their respective properties in good faith from MICC, they are third
parties whose right thereto are superior to that of Banco Filipino; (2)
they are still entitled to redeem the properties and in fact a binding
agreement between them and the bank had been reached; (3) their
131

respective houses should not have been included in the auction sale of
the mortgaged properties; (4) on the contrary, as builders in good
faith, they are entitled to the benefits of Article 448 of the Civil Code;
and (5) the writ of possession issued by the RTC in 1996 had already
lost its validity and efficacy.
ISSUE:
Whether or not the writ of possession is effective against
petitioners.
RULING:
YES. That petitioners purchased their properties from MICC in
good faith is of no moment. The purchases took place after MICCs
mortgage to Banco Filipino had been registered. As such, a real right
or lien in favor of Banco Filipino had already been established,
subsisting over the properties until the discharge of the principal
obligation, whoever the possessor(s) of the land might be.
Sale or transfer cannot affect or release the mortgage. A
purchaser is necessarily bound to acknowledge and respect the
encumbrance to which is subjected the purchased thing and which is
at the disposal of the creditor "in order that he, under the terms of the
contract, may recover the amount of his credit therefrom." For, a
recorded real estate mortgage is a right in rem, a lien on the property
whoever its owner may be. Because the personality of the owner is
disregarded; the mortgage subsists notwithstanding changes of
ownership; the last transferee is just as much of a debtor as the first
one; and this, independent of whether the transferee knows or not the
person of the mortgagee. So it is, that a mortgage lien is inseparable
from the property mortgaged. All subsequent purchasers thereof
must respect the mortgage, whether the transfer to them is with or
without the consent of the mortgagee. For, the mortgage, until
discharge, follows the property.
Petitioners are also not entitled to redeem the foreclosed
properties. The debtor in extra-judicial foreclosures under Act No.
3135, or his successor-in-interest, has, one year from the date of
registration of the Certificate of Sale with the Registry of Deeds, a
right to redeem the foreclosed mortgage; petitioners failed to do so.

132

G.R. No. L-30817

September 29, 1972

DOMINADOR DIZON, doing business under the firm name


"Pawnshop of Dominador Dizon", petitioner, vs. LOURDES G.
SUNTAY, respondent.
FACTS:
A diamond ring was turned over to a certain Clarita R. Sison,
for sale on commission, along with other pieces of jewelry of
respondent Suntay. It was then pledged to petitioner. Since what was
done was violative of the terms of the agency, there was an attempt on
her part to recover possession thereof from petitioner, who refused.
She had to file an action then for its recovery. She was successful, as
noted above, both in the lower court and thereafter in the Court of
Appeals. She prevailed as she had in her favor the protection
accorded by Article 559 of the Civil Code.
Suntay is the owner of a three-carat diamond ring valued at
P5,500.00. She and Clarita R. Sison entered into a transaction
wherein the ring was delivered to Clarita R. Sison for sale on
commission. Upon receiving the ring, Clarita R. Sison executed and
delivered to Suntay the receipt ... . Suntay had already previously
known Clarita R. Sison as the latter is a close friend of the
respondent's cousin and they had frequently met each other at the
place of the respondent said cousin.
After the lapse of a considerable time without Clarita R. Sison
having returned to Suntay the latter's ring, the plaintiff made
demands on Clarita R. Sison for the return of her ring but the latter
could not comply with the demands because, without the knowledge
of the respondent, three days after the ring above-mentioned was
received by Clarita R. Sison from the respondent, said ring was
pledged by Melia Sison, niece of the husband of Clarita R. Sison,
evidently in connivance with the latter, with the defendant's
pawnshop for P2,600.00.
Sison finally delivered to the former the pawnshop ticket ...
which is the receipt of the pledge with the defendant's pawnshop of
the respondent's ring. Suntay filed a case of estafa and filed an action
with the RTC for the recovery of the said ring. The lower court issued
the writ of replevin and later on rendered judgment declaring that
Suntay had the right to the possession of the ring in question.
CA denied the appeal of Dizon.
RULING:
133

Article 559 of the Civil Code provides "The possession of


movable property acquired in good faith is equivalent to a title.
Nevertheless, one who has lost any movable or has been unlawfully
deprived thereof may recover it from the person in possession of the
same. If the possessor of a movable lost of which the owner has been
unlawfully deprived, has acquired it in good faith at a public sale, the
owner cannot obtain its return without reimbursing the price paid
therefor."
If the possessor of a movable lost of which the owner has been
unlawfully deprived, has acquired it in good faith at a public sale, the
owner cannot obtain its return without reimbursing the price paid
therefor. The only exception the law allows is when there is
acquisition in good faith of the possessor at a public sale, in which
case the owner cannot obtain its return without reimbursing the
price.
The common law principle that were one of two innocent
persons must suffer by a fraud perpetrated by another, the law
imposes the loss upon the party who, by his misplaced confidence,
has enabled the fraud to be committed, cannot be applied in a case
which is covered by an express provision of the new Civil Code,
specifically Article 559. Between a common law principle and a
statutory provision, the latter must prevail in this jurisdiction.
Petitioner invoked the principle of estoppel. Estoppel has its
roots in equity. Good faith is its basis. It is a response to the demands
of moral right and natural justice. For estoppel to exist though, it is
indispensable that there be a declaration, act or omission by the party
who is sought to be bound.
In the present case, not only has the ownership and the origin of
the jewels misappropriated been unquestionably proven but also that
the accused, acting fraudulently and in bad faith, disposed of them
and pledged them contrary to agreement, with no right of ownership,
and to the prejudice of the injured party, who was thereby illegally
deprived of said jewels; therefore, in accordance with the provisions
of article 464, the owner has an absolute right to recover the jewels
from the possession of whosoever holds them.
Petitioner ought to have been on his guard before accepting the
pledge in question. Estoppel certainly cannot be justly invoked.
G.R. No. L-20264

January 30, 1971


134

CONSUELO S. DE GARCIA and ANASTACIO GARCIA, petitioners,


vs. HON. COURT OF APPEALS, ANGELINA D. GUEVARA and
JUAN B. GUEVARA, respondents.
FACTS:
Mrs. Guevara owned a pretty diamond ring with white gold
mounting, 2.05 diamond-solitaire, and 4 brills. Sometime in February
1952, the ring was stolen from her house. Luckily, on October 1953
(barely a year after), she found it at a restaurant, La Bulakena, on the
finger of the restaurant owner, Consuelo De Garcia.
Guevara asked De Garcia where she bought it and explained to
her how she had lost it. When the ring was handed to her by De
Garcia, it fitted her perfectly. The next time around, she brought her
husband and Rebullida, the person whom she bought the ring from,
to verify the identity of the ring. Rebullida examined the ring with the
aid of high power lens and his 30 years of experience. He concluded
that it was the very ring that he had sold to the Guevaras. After that,
Guevara sent a written request for the ring, but De Garcia did not
deliver it. When the sheriff tries to serve a writ of seizure, De Garica
likewise refused to deliver the ring.
According to De Garcia, she bought the ring from her kumare
who got it from another Miss who in turn got it from the owner, a
certain Aling Petring. Aling Petring however, was nowhere to be
found. She boarded three months at the first buyers house but left a
week after her landlady bought the ring. The first buyer did not even
know Aling Petrings last name nor her forwarding address. De Garcia
claims to be a holder in good faith and for value. She says her
possession is equivalent to title.
ISSUE:
Who has a better right of possession.
RULING:
Respondent Angelina D. Guevara, having been unlawfully
deprived of the diamond ring in question, was entitled to recover it
from petitioner Consuelo S. de Garcia who was found in possession of
the same.
The controlling provision is Article 559 of the Civil Code. It
reads thus: "The possession of movable property acquired in good
faith is equivalent to a title. Nevertheless, one who has lost any
movable or has been unlawfully deprived thereof may recover it from
135

the person in possession of the same. If the possessor of a movable


lost of which the owner has been unlawfully deprived, has acquired it
in good faith at a public sale, the owner cannot obtain its return
without reimbursing the price paid therefor."
The only exception the law allows is when there is acquisition in
good faith of the possessor at a public sale, in which case the owner
cannot obtain its return without reimbursing the price.

G.R. Nos. 154391-92

September 30, 2004


136

Spouses ISMAEL and TERESITA MACASAET, petitioners, vs.


Spouses VICENTE and ROSARIO MACASAET, respondents.
FACTS:
The present case involves a dispute between parents and
children. The children were invited by the parents to occupy the
latters two lots, out of parental love and a desire to foster family
solidarity. Unfortunately, an unresolved conflict terminated this
situation. Out of pique, the parents asked them to vacate the
premises. Thus, the children lost their right to remain on the
property. They have the right, however, to be indemnified for the
useful improvements that they constructed thereon in good faith and
with the consent of the parents. In short, Article 448 of the Civil Code
applies.
Petitioners Ismael and Teresita Macasaet and Respondents
Vicente and Rosario Macasaet are first-degree relatives. Ismael is the
son of respondents, and Teresita is his wife. the parents filed with the
Municipal Trial Court in Cities (MTCC) of Lipa City an ejectment suit
against the children. Respondents alleged that they were the owners
of two (2) parcels of land; that by way of a verbal lease agreement,
Ismael and Teresita occupied these lots in March 1992 and used them
as their residence and the situs of their construction business; and
that despite repeated demands, petitioners failed to pay the agreed
rental of P500 per week.
Ismael and Teresita denied the existence of any verbal lease
agreement. They claimed that respondents had invited them to
construct their residence and business on the subject lots in order
that they could all live near one other, employ Marivic (the sister of
Ismael), and help in resolving the problems of the family. They added
that it was the policy of respondents to allot the land they owned as
an advance grant of inheritance in favor of their children.
The MTCC ruled in favor of respondents and ordered
petitioners to vacate the premises. On appeal, the RTC upheld the
findings of MTCC and allowed respondents to appropriate the
building and other improvements introduced by petitioners after
payment of the indemnity provided for by Article 448 in relation to
Articles 546 and 548 of the Civil Code.
The CA sustained the finding of the two lower courts that
Ismael and Teresita had been occupying the subject lots only by the
tolerance of Vicente and Rosario. Thus, possession of the subject lots
137

by petitioners became illegal upon their receipt of respondents letter


to vacate it.
In ascertaining the right of petitioners to be reimbursed for the
improvements they had introduced on respondents properties, the
appellate court applied the Civil Codes provisions on lease. The CA
modified the RTC Decision by declaring that Article 448 of the Civil
Code was inapplicable. The CA opined that under Article 1678 of the
same Code, Ismael and Teresita had the right to be reimbursed for
one half of the value of the improvements made.
ISSUE:
Whether or not Article 1678 of the Civil Code should apply to
the case on the matters of improvements, or is it Article 447 of the
Civil Code in relation to the Article 453 and 454 thereof that should
apply, if ever to apply the Civil Code.
RULING:
In the present case, petitioners failed to justify their right to
retain possession of the subject lots, which respondents own. Since
possession is one of the attributes of ownership, respondents clearly
are entitled to physical or material possession.
This Court has consistently held that those who occupy the land
of another at the latters tolerance or permission, without any
contract between them, are necessarily bound by an implied promise
that the occupants will vacate the property upon demand. A summary
action for ejectment is the proper remedy to enforce this implied
obligation. The unlawful deprivation or withholding of possession is
to be counted from the date of the demand to vacate.
Court ruled out the finding of possession by mere tolerance.
Petitioners were able to establish that respondents had invited them
to occupy the subject lots in order that they could all live near one
other and help in resolving family problems. The occupancy of the
subject lots by petitioners was not merely "something not wholly
approved of" by respondents. Neither did it arise from what Tolentino
refers to as "neighborliness or familiarity." In point of fact, their
possession was upon the invitation of and with the complete approval
of respondents, who desired that their children would occupy the
premises. It arose from familial love and a desire for family solidarity,
which are basic Filipino traits.
When persistent conflict and animosity overtook the love and
solidarity between the parents and the children, the purpose of the
138

agreement ceased. Thus, petitioners no longer had any cause for


continued possession of the lots. Their right to use the properties
became untenable. It ceased upon their receipt of the notice to vacate.
And because they refused to heed the demand, ejectment was the
proper remedy against them. Their possession, which was originally
lawful, became unlawful when the reason therefor -- love and
solidarity -- ceased to exist between them.
The right of petitioners to inherit from their parents is merely
inchoate and is vested only upon the latters demise. Indisputably,
rights of succession are transmitted only from the moment of death of
the decedent.
Article 447 is not applicable, because it relates to the rules that
apply when the owner of the property uses the materials of another. It
does not refer to the instance when a possessor builds on the property
of another, which is the factual milieu here. Article 1678 of the Civil
Code is likewise inapplicable.
On the other hand, when a person builds in good faith on the
land of another, the applicable provision is Article 448. The
established facts of this case show that respondents fully consented to
the improvements introduced by petitioners. In fact, because the
children occupied the lots upon their invitation, the parents certainly
knew and approved of the construction of the improvements
introduced thereon. Thus, petitioners may be deemed to have been in
good faith when they built the structures on those lots.
Respondents have the right to appropriate -- as their own -- the
building and other improvements on the subject lots, but only after
(1) refunding the expenses of petitioners or (2) paying the increase in
value acquired by the properties by reason thereof. They have the
option to oblige petitioners to pay the price of the land, unless its
value is considerably more than that of the structures -- in which case,
petitioners shall pay reasonable rent.

G.R. No. 141325

July 31, 2006


139

PELBEL MANUFACTURING CORPORATION, Substituted by


Pelagia Beltran, and Virginia Malolos, petitioners, vs. HON. COURT
OF APPEALS, and THE REPUBLIC OF THE PHILIPPINES,
respondents.
FACTS:
Pelbel Manufacturing Corporation, Aladdin Trinidad and
Virginia Malolos are the original applicants sought to register two
parcels of land situated in San Juan, Taytay, Rizal. Initial hearing was
scheduled; there being no formal opposition on record, an order of
general default was issued and applicants were allowed to present
evidence ex-parte. Laguna Lake Development Authority filed a
manifestation stating that the subject lots are situated below the
elevation of 12.50 meters, thus forming part of the bed of the Laguna
Lake.
Opposition was filed by the Office of the Solicitor General
alleging that neither the applicants nor their predecessors-in-interest
have been in open, continuous, exclusive and notorious possession
and occupation of the land since June 12, 1945 or prior thereto; that
the applicants' claim of ownership in fee simple on the basis of
Spanish Title or grant can no longer be availed of for failure to file the
appropriate application for registration within six (6) months from
February 16, 1976 as required by P.D. No. 892; and that applicant
Pelbel Manufacturing Corporation is disqualified, being a private
corporation, to hold lands of the public domain except by lease
pursuant to Section 11, Article XIV of the 1973 Constitution.
Pelbel filed a motion to substitute Pelagia P. Beltran in its place
as applicant with respect to 17,500 square meters of the lot applied
for and was granted by the lower court. The lower court rendered the
assailed decision adjudicating the parcels of land applied for in favor
of the following: Pelagia Beltran 17,500 square meters; Aladdin
Trinidad 2,500 square meters; Virginia Malolos 10,251 square
meters.
A Motion to Amend Order of General Default and Set Aside
Decision was filed by Laguna Lake Development Authority on the
ground that LLDA had already established by preliminary
investigations that the lots are below elevation of 12.50 meters, hence
are of public dominion. The lower court directed the Office of the
Solicitor General to file comment on the motion. The OSG filed its
Comment supporting the LLDA's position that lakes and their beds
such as the lots sought to be registered are considered public domain.
OSG further argued that applicant Pelbel, being a corporation, is
140

disqualified from acquiring lands of the public domain and that


applicants are not entitled to registration for lack of the requisite
number of years of possession before June 12, 1945.
The trial court rendered a decision in favor of the petition. On
appeal, CA reversed the decision of the lower court. It dismissed the
applications for land registration of petitioners Pelagia Beltran,
Aladdin F. Trinidad and Virginia Malolos.
ISSUES:
Whether or not the subject lots are public land.
If they are not public land, whether applicants-petitioners have
registrable title to the land.
RULING:
The controlling act in this case is the Public Land Act. Under
the Public Land Act, there is a presumption that the land applied for
belongs to the state, and that the occupants and possessors can only
claim an interest in the land by virtue of their imperfect title or
continuous, open, and notorious possession thereof for a period
prescribed by law. The basic doctrine is that all lands not otherwise
appearing to be clearly within private ownership are presumed to
belong to the State.
Any applicant for judicial confirmation of an imperfect title has
the burden of proving, by incontrovertible evidence, that the (a) land
applied for is alienable and disposable public land; and, (b) the
applicant, by himself or through his predecessors-in-interest had
occupied and possessed the land, in the concept of owner, openly,
continuously, exclusively, and adversely since June 12, 1945, or
earlier.
Court held that petitioners failed to show that the parcels of
land subject of their application are alienable and disposable.
The following are of public dominion: (1) Rivers and their
natural beds; (2) Continuous or intermittent waters of springs and
brooks running in their natural beds and the beds themselves; (3)
Waters rising continuously or intermittently on lands of public
dominion; (4) Lakes and lagoons formed by Nature on public lands,
and their beds.

141

Petitioners invoke the case of Bautista v. Court of Appeals,


claiming that the inundation was merely due to the rains, and that the
water elevation should be determined from the highest ordinary
depth during dry season. It is contended that the measurement of
Laguna Lake Development Authority Geodetic Engineer Merida of
12.19 meters as the highest observed elevation of the subject lots was
made in November, which is still rainy season. We disagree for while
November is not part of the summer season, it is not part of the rainy
season either. It still is part of the dry season during which the waters
are at their "highest ordinary depth."
In Ledesma v. Municipality of Iloilo, this Court held that
"simple possession of a certificate of title, under the Torrens System,
does not make the possessor the true owner of all the property
described therein. If a person obtains a title, under the Torrens
System, which includes by mistake or oversight land which cannot be
registered under the Torrens System, he does not, by virtue of said
certificate alone, become the owner of the lands illegally included." It
is basic principle that prescription does not run against the
government.
Possession is open when it is visible and apparent to a common
observer. Continuous possession consists of uninterrupted acts of
nonpermissive possession of property by the current occupants and
their predecessors. To be notorious, possession must be so
conspicuous that it is generally known and talked of by the public or
at least by the people in the vicinity of the premises. Mere possession
of land and the making of vague assertions to the public that a
possessor is claiming the land are not sufficient to satisfy the
requirement of open and notorious possession. Bernardo failed to
show that his alleged possession and occupation were of the nature
and duration required by law.

142

You might also like